X



トップページ数学
1002コメント421KB

分からない問題はここに書いてね454

レス数が1000を超えています。これ以上書き込みはできません。
0002132人目の素数さん
垢版 |
2019/07/06(土) 23:16:28.69ID:DdjFMnYe
ここは分からない問題を書くスレです。
お願いごとをするスレでも分からない問題に答えてもらえるスレでもありません。
0003132人目の素数さん
垢版 |
2019/07/06(土) 23:27:14.34ID:qfvtguSN
答えが分かってる問題を書く場所でもないがな
0004132人目の素数さん
垢版 |
2019/07/06(土) 23:33:42.68ID:Q3OOWfSD
自然数の複雑度を流行らせたい
ある自然数nを1と+と×と()を用いて表すとき最小の1の個数をω(n)とする。
自然数n,kについてω(n^k)=kω(n)か?
素数pに対してω(p)=ω(p-1)+1か?
0005132人目の素数さん
垢版 |
2019/07/06(土) 23:48:25.95ID:Q3OOWfSD
ちなみに、集合W(n)をW(1)={1}、W(2)={1,2}、W(3)={1,2,3}、n≧3について
W(n+1)={∀a∈W(n),∀d(dはaが合成数のときのaの約数),∀e(eはaが素数のときのa-1の約数)/
a, a+a/d, a+(a-1)/e}
とすると、ある自然数kに対して、kが集合W(n)に含まれて、i≦kを満たす全ての自然数iに対して
kがW(i)に含まれないことと、ω(k)=nであることは同値です
0006132人目の素数さん
垢版 |
2019/07/07(日) 00:44:49.01ID:VoskLKfF
>>4
ω(n^k)=kω(n)の反例
ω(11)=8 ;11=(1+1)×(1+1+1+1+1)+1
ω(11^2)=15 ;11^2=(1+1)×(1+1)×(1+1)×(1+1+1)×(1+1+1+1+1)+1
0007132人目の素数さん
垢版 |
2019/07/07(日) 01:25:08.29ID:zXPpwQqV
>>6
反例ありがとうございます 因みにn=3のときには成り立っていることは証明されています数学セミナーにありました
0008132人目の素数さん
垢版 |
2019/07/07(日) 01:53:15.88ID:zXPpwQqV
>>5の追記です
d≠aかつe≠a-1です
0009 【底辺】
垢版 |
2019/07/07(日) 10:58:05.44ID:yiXPjGyl
やっぱり部分積分じゃない。
変数がrとθの2つあるでなぁ。
f(x)g(x)みたいに一つの変数xじゃなくて変数が2つある場合だから、この式じゃないと思う。
∬[r=0〜 ][θ=0°〜 ]――drdθみたいな感じ。
0010132人目の素数さん
垢版 |
2019/07/07(日) 11:13:55.05ID:D6DPOF4d
定積分I[x] = ∫[1 to x] 1/{xln(x)} dx に対し、
不等式m < I[n] < m+1を満たす自然数nの個数をa[m]とおく。
mがm=1,2,...と正の整数値をとるとき、
b[m]=a[m+1]/a[m]で定義される数列b[m]について、
その増減、m→∞としたときの極限
をそれぞれ述べよ。
0011132人目の素数さん
垢版 |
2019/07/07(日) 13:48:06.99ID:QiINekeW
I[x] = ∫[1 to x] 1/{xln(x)} dx =
= lim{ε→+0} ∫[1+ε to x] 1/{xln(x)} dx = ln( ln(x) ) - lim{ε→+0} ln( ln(1+ε) )
という理解でOK? これ発散しますよね?
0012132人目の素数さん
垢版 |
2019/07/07(日) 17:45:01.21ID:NQih2PzA
>>9
h(r,θ) = 2(1 - r・cosθ),
を入れると
v/2 = ∫[0〜1] ∫[0〜60゚] h(r,θ) dθ rdr
 = ∫[0〜1] ∫[0〜π/3] 2(1-r・cosθ) dθ rdr
 = ∫[0〜1] 2r dr・∫[0〜π/3] dθ - ∫[0〜1] rr dr・∫[0〜π/3] 2cosθ dθ
 = (π/3) - (1/3)[ 2sinθ ](0→π/3)
 = (π-√3)/3,
部分積分しなくても出る。

>>10
J[x] = ∫[e to x] 1/{x・ln(x)} dx = [ ln(ln(x)) ](e to x) = ln(ln(x)),
は収束。
0013132人目の素数さん
垢版 |
2019/07/07(日) 18:16:20.38ID:D6DPOF4d
長方形ABCD(AB<AD)の内角∠Aの2等分線をLとする。
L上に点Pを、直線PCと対角線BDが交点を持つようにとる。
このとき、以下を示せ。
「PC=BDとなるための必要十分条件は、PCとBDが直交することである。」
0018132人目の素数さん
垢版 |
2019/07/07(日) 20:47:14.80ID:h0Y2HNP2
スレチだったらすみません。

ベッセル関数が定常波、ハンケル関数が進行波を示すという意味がよくわからないのですが、どういった風に考えれば良いですか?
0021132人目の素数さん
垢版 |
2019/07/07(日) 22:23:26.07ID:csdahXcm
あんま興味ある人ないかもだが集合論に関して教えてくれ
ラッセルのパラドックスを生む「集合」Rを定義すると矛盾が導かれることは有名だ
背理法を使った証明において、何らかの「集合」Xを導入して矛盾を示したとする。
この時、XがRと同じように定義するだけで矛盾を導く「集合」ではないことを証明する必要があると思うんだが、
これを行うにはどういう手順を踏むのが普通なのか教えてくれ
0022132人目の素数さん
垢版 |
2019/07/07(日) 22:42:58.23ID:UZ6/i4IF
>>21
そのXが利用してる公理的集合論の公理から正しく存在が保証されてるか確認すればいい。
0023132人目の素数さん
垢版 |
2019/07/07(日) 22:56:57.22ID:csdahXcm
>>22
ありがとう。やっぱりそれしかないか
空集合か少なくとも一つ集合を含むことを言えればOKとかそういう便利なのがあればよかったんだが。
公理に戻るのは大変なので、専門書から存在証明がされている集合の定理を探してみるよ
0024132人目の素数さん
垢版 |
2019/07/07(日) 23:08:11.74ID:NQih2PzA
>>13
 A(0,0) B(b,0) C(b,d) D(0,d) E(b+d,b+d)
とおく。
Cを中心とし半径BDの円周をKとおく。
AC = BD, CE=BD より A,E ∈ K
∴  K∩L = {A,E}   ← 円と直線の交点は高々2つ

P∈L とすると
 PC=BD ⇔ P∈(K∩L) ⇔ P∈{A,E}
0025132人目の素数さん
垢版 |
2019/07/07(日) 23:33:29.35ID:NQih2PzA
ハンケル関数は、円筒波(または2次元空間の波)を表わす解であり、内側/外側に伝搬する波を表わす。
0026132人目の素数さん
垢版 |
2019/07/08(月) 00:02:24.63ID:1pXLvBhA
>>25
ありがとうございます。
ただ、円筒波を表すと言われても、あの形でなぜ波?となるのですが。
ベッセル関数とノイマン関数の線形結合させたのがハンケル関数のはずですがそもそもなぜ、ハンケル関数を導入したのかと。
0027イナ ◆/7jUdUKiSM
垢版 |
2019/07/08(月) 13:32:00.19ID:K2B6PKC6
>>9
>>12
1-rcosθがどこかわかればわかると思うんです。
1-rcosθの1は単位円の半径ですか?
rcosθは中心から距離rの地点の中心からの距離ですか?
これを1から引くということは、
1-rcosθは中心からr、扇形の端からθの地点の単位円周上からの距離ですか。
これを2倍して、
2(1-rcosθ)は高さですか? つまり高さ1の単位円柱が正四面体PABCの側面で外に削り取られた蒲鉾外の高さ。
でも断面が1:2:√5の直角三角形になるのはθ=π/3のときだけのはず。
h(r,θ)の地点で2倍してるのはなぜでしょう?
0028132人目の素数さん
垢版 |
2019/07/08(月) 14:06:28.97ID:5fGgcQug
f(x)=3xx /(2xx +1)とする。
0<z<1、a(1)=z、a(n+1)=f( a(n) )で定める時、
zの値に応じたlim(n→∞) a(n)を求めよ。

これお願いします 
某大学の入試ですが、f'(x)が1を越えることがあるので普通の平均値の定理を使った解法では解けません
発想の順序も教えてもらえたら嬉しいっす
0029132人目の素数さん
垢版 |
2019/07/08(月) 16:06:08.32ID:3v+A3NFN
0<a≤b、0<R、とする。
また領域D、領域Eを以下のように定める。

領域D:
O(0,0),A(a,0),B(a,b),C(0,b)を頂点とする長方形OABCの周上および内部

領域E:
O(0,0)を中心とする半径Rの円の周上および内部

(1)2つの円C_1とC_2を、領域Dからはみ出ないように、かつC_1とC_2が互いに外接するか外部にあるように置く。
C_1とC_2の面積の合計が最大になるような置き方を述べよ。

(2)2つの円K_1とK_2を、(1)と同様に領域E内に置く置き方を述べよ。
0030132人目の素数さん
垢版 |
2019/07/08(月) 18:50:01.07ID:0A6xzhiD
微分についての質問です

x=sinθとおいたとき両辺をθで微分すると
dx/dθ=cosθとなります
ここで、記号的には「xを微分する」という操作で「xの前にdをつけて、さらに分母にdθをつける」
ことを行いました
その後dx=cosθdθとおいて積分する方法がよく用いられますが、これが気持ち悪くて仕方ありません
dx/dθにはxをθで微分するという意味が与えられているだけで、分数の概念ではありません。
それなのに分数と扱って分子と分母を都合よく切り分けて計算してよいのか疑問です。
0031132人目の素数さん
垢版 |
2019/07/08(月) 19:02:46.57ID:FZN1I4Vd
>>30
高木貞治先生が解析概論でこの記号は割り算と書いています
微小量に対する微小量の変化を見ているのが本義なので実際本質的意味は割り算です
0032132人目の素数さん
垢版 |
2019/07/08(月) 19:22:09.85ID:MbYtBCrY
Aが0でない定数のとき、
exp(x)=A(1+x)
上記の方程式解ける方いたら、教えてください
0033132人目の素数さん
垢版 |
2019/07/08(月) 20:24:52.09ID:U4VP0NGu
>>26
なぜ波か? それは
2次元波動方程式:
 { (∂x)^2+(∂y)^2 - (∂ct)^2 } Ψ = { r^2.(∂r)^2+ r.(∂r) + (1/r).(∂φ) + k^2 } Ψ = 0
 Ψ ∝ exp(-iωt), k=ω/c
この一般解(極座標表示) が
 Ψ(r,φ,t) = Σ ( a[n]. J_n(kr)+ b[n]. Y_n(kr) ). exp(inφ). exp(-iωt)  
のように表せるからです。
J_n, Y_n は定常波 (https://imgur.com/a/CKaxlNo)
H1_n = J_n + i Y_n は外向き進行波 ( https://imgur.com/a/Lrq5Lua )
H2_n = J_n - i Y_n は内向き進行波になっています。
おまけ ( https://imgur.com/a/YPSIGd0 )

3次元波動方程式の場合
 { (∂x)^2+(∂y)^2+(∂z)^2 - (∂ct)^2 } Ψ = { r^2.(∂r)^2+ r.(∂r) + (1/r)(∂φ) + k^2 } Ψ = 0
 Ψ ∝ exp(ihz).exp(-iωt), k^2 = (ω/c)^2 - h^2
一般解(円筒座標表示): Ψ = Ψ(r, φ,z, t) = (略)
と、まあ 2次元極座標とほぼ同じ扱いになりますが
k^2 < 0 になる場合は変形ベッセル方程式となります. (解は I_n, K_n で表せる)
0034132人目の素数さん
垢版 |
2019/07/08(月) 20:28:48.06ID:U4VP0NGu
訂正
誤: (1/r).(∂φ)
正: (1/r^2).(∂φ)^2

∂x, ∂ct, ∂φ, etc. は ∂/∂x , ∂/(c∂t) , ∂/∂φ, etc. の略記です。
0035132人目の素数さん
垢版 |
2019/07/08(月) 20:36:24.68ID:dfIKQ8dH
>>30
記号が意味そのものではないとは確かなことなんですけど、長い微分積分の歴史の中でそういう表記が生き残ってきたのにはやはり理由があるんですね

分数と考えても良いのですよ
003633
垢版 |
2019/07/08(月) 20:45:44.48ID:U4VP0NGu
追加
Y_n の定常波 ( https://imgur.com/a/LoX4UdK )

plot は python のmatplotlib を使用した。
ほんの40行くらいで 3次元プロット, latex形式でタイトル入れ, gif アニメ生成までやってくれた。
0038132人目の素数さん
垢版 |
2019/07/08(月) 21:18:11.66ID:9/nEljcK
>>30
高校生だろうから深く突っ込む必要はないけど、微分形式というものを考えるとdyやdxなどを単独で扱うことができます
で、ポアンカレの補題というもの(の特殊な場合)から適当な領域上でdy=F(x)dxとなる関数Fが存在することが知られています
実際にはこの関数F(x)が導関数dy/dxですから、この記号dy/dxを「dy=F(x)dxをdxで割ったもの、つまり割り算」と見て差し支えありません

一方で微分を割り算として捉えるとまずい場合もありますが、それは偏微分(多変数関数の各変数に関する微分)というもので浮き彫りになると思います
0039132人目の素数さん
垢版 |
2019/07/08(月) 21:34:21.01ID:AotJujlr
>>30
分母を払うのはただのニーモニックで、実際には置換積分をやっただけだよ
割り算とか分数とかは忘れな
004133
垢版 |
2019/07/09(火) 01:29:02.81ID:QOcWt3fr
再訂正
誤: { r^2.(∂r)^2+ r.(∂r) + (1/r).(∂φ) + k^2 } Ψ = 0
正: { (∂r)^2+ (1/r).(∂r) + (1/r)^2.(∂φ)^2 + k^2 } Ψ = 0

r^2 を掛けて
{ r^2.(∂r)^2+ r.(∂r) + (∂φ)^2 + (kr)^2 } Ψ = 0
Ψ = f(kr) exp(inφ) exp(-iωt) と変数分離(一価性からの要請により n は整数)、ξ=kr と置いて
ベッセル方程式: { ξ^2.(∂ξ)^2+ ξ.(∂ξ) + (ξ^2 - n^2) } f(ξ) = 0
が得られます。
二階の常微分方程式なので独立な解は 二つ、
それが J_n, Y_n の組 (或いは H1_n , H2_n の組) です。
Y_n の解は r → 0 で発散しますが、 中心部( r = 0 )を含まない境界条件の時に活躍してくれます。
0042132人目の素数さん
垢版 |
2019/07/09(火) 03:12:02.73ID:96Oo9tpn
>>32

 -(1+x)・e^{-(1+x)} = -1/(Ae),
より
 -(1+x) = W{-1/(Ae)},
 x = -W{-1/(Ae)} -1,
 W は Lambert のW函数。
0043132人目の素数さん
垢版 |
2019/07/09(火) 03:32:45.06ID:XQMJVwCR
2^n (n=1,2,3,...)の一桁目の数は2,4,8,6の周期4で変化する。
一方最上位の数に注目すると2,4,8,1,3,6,1,2,5,1,2,...etcと非周期的に変化して分布も知られています(ベンフォードの定理)
それではその中間の二桁目の数、三桁目の数 、、、最上位の一つ下の数 etc 他の桁の数は周期的に変化するのでしょうか??
それとも非周期的なんでしょうか?
0046132人目の素数さん
垢版 |
2019/07/09(火) 11:21:48.17ID:ngjOKblw
分布は決定できるけどね。
最上位からみて二桁目が3である割合とか。
いわゆる一様分布定理の応用。
0048132人目の素数さん
垢版 |
2019/07/09(火) 15:29:32.82ID:96Oo9tpn
>>28

0<z<1/2 のとき
 g(x) = f(x)/x とおく。
 g(x) = 3x/(2xx+1) = 1 - (1-x)(1-2x)/(2xx+1) <1,
 a(n) は単調減少。a(n)<z<1/2,
 また g(x) は |x| < 0.707107 で単調増加。
 a(n+1)/a(n) = g(a(n)) ≦ g(z) < g(1/2) = 1,
 0 < a(n+1) < z・g(z)^n → 0  (n→∞)

1/2<z<1 のとき
 h(x) = [1-f(x)]/(1-x) とおく。
 h(x) = (x+1)/(2xx+1) = 1 - x(2x-1)/(2xx+1) < 1,
 a(n) は単調増加。a(n)>z>1/2,
 また また h(x) は x > 0.224745 で単調減少。
 [1-a(n+1)]/[1-a(n)] = h(a(n)) < h(z) < h(1/2) = 1,
 0 < 1 - a(n+1) < (1-z)・h(z)^n → 0  (n→∞)

z=1/2 のとき
 a(n) = 1/2.
0049132人目の素数さん
垢版 |
2019/07/09(火) 15:54:25.80ID:T5vxXPqs
>>48
ありがとうございます!
実戦で25分以内にその関数を見つけられるようにするためには、どういう思考の道筋を辿ると良いと思われますか?
0050132人目の素数さん
垢版 |
2019/07/09(火) 16:28:44.61ID:AYJbNafv
>>49
発想としては極限値をグラフの交点から推測する時と同じ
あとそれを論述する式変形をする
0051132人目の素数さん
垢版 |
2019/07/09(火) 18:21:09.21ID:m7NCgSpi
平均値の定理方式なら、(必要なら)最初の何項かを間引いて平均値の定理が使うといい
25分もあればなんとかなるでしょ
0052132人目の素数さん
垢版 |
2019/07/09(火) 19:51:51.09ID:8hyZVRLK
そうそう。
どんな初項から始めてもならちょっと考えないといけない時もあるけど、具体的に初項指定されてたら大概二三項めから考えればなんとかなるからな。
0054132人目の素数さん
垢版 |
2019/07/09(火) 20:03:43.04ID:XugGftlG
ウエメセでできる人にのっかって適当いいたいだけのヤカラが結構いるな。
問題くらいは読め
0055132人目の素数さん
垢版 |
2019/07/09(火) 20:10:21.09ID:AFWmOWgm
ちゃんと 0<z<1、a(1)=z と指定されてるぞ甘ったれハゲ
0056132人目の素数さん
垢版 |
2019/07/09(火) 20:18:41.96ID:XugGftlG
そもそもf'(x)=1の解が0.19221…と0.74767…あたりなんだが
最初2-3項調べて一体何をどうする気なんだかw
問題触りすらしない一言居士見ると笑っちまうな
0057132人目の素数さん
垢版 |
2019/07/09(火) 20:35:28.84ID:QOcWt3fr
グラフ描けば一目瞭然じゃーん、終わり。
としたいけど、それじゃダメなんですよね。
どうしたらいいのか分からんけど、
この種の「一目瞭然」を統一的に扱うにはどうしたらいいのかね?
毎回、>>48 みたいな工夫をしないといけないの?
0058132人目の素数さん
垢版 |
2019/07/09(火) 21:28:57.26ID:8hyZVRLK
まず1未満の正の数rを選ぶ。
交点を通り傾きが±rの直線を引く。
何項か計算して(a(n),a(n+1))がその二本の直線に挟まれたエリアに入ったら完。
0059132人目の素数さん
垢版 |
2019/07/09(火) 22:10:06.80ID:d6MHMyN4
>>57
これ試験に書いても満点はもらえないとされてるようですね。部分点はくれるでしょうが。

しかしこれで…うーん…
どういう理屈で説明すれば完璧になるんでしょうね?
0060132人目の素数さん
垢版 |
2019/07/09(火) 22:39:56.31ID:/q+BEn65
>>40
反例ありがとうございます。 そうでしたか。結局全部の予想が外れてて残念です
でも複雑度は結構面白いので皆さんも考えてみては?
0061132人目の素数さん
垢版 |
2019/07/10(水) 01:38:58.00ID:miBhNnEC
>>27
 h は 平面PBC (正三角錐PABCの一面) の高さです。
 (x,y) 座標では h = 2(1+y) です。(y=0 で 2, y=-1 で 0 だから)
 (r,θ) 座標では h = 2(1-r・cosθ) です。
0062132人目の素数さん
垢版 |
2019/07/10(水) 02:12:37.49ID:0SG7ubvB
>>59

0<|z|<1/2の時
第2項は0<x<1/2
r=a(2)とおくと2以上のnで(a(n),a(n+1))は領域0<x<1、|y|<|rx|
以下ry

1/2<|z|<1の時
第2項は1/2<x<1
r=|a(2)-1|とおくと2以上のnで(a(n),a(n+1))は領1/2<x<1、|y-1|≦r|x-1|
以下ry

|z|>1/2の時
第2項は1/2<x<1
r=f'(1)とおくと2以上のnで(a(n),a(n+1))は領1/2<x<1、|y-1|≦r|x-1|
以下ry
0063132人目の素数さん
垢版 |
2019/07/10(水) 05:18:45.21ID:gOXX67fF
(1)等式 3^n = k^2 - 40 を満たす自然数の組(n,k)を全て求めよ。

mは1以上99以下の整数とする。

(2)等式3^n = k^2 - m を満たす自然数の組(n,k)が存在しないようにmを定めることができる。このことを示せ。

(3)(2)のようなmを全て求めよ。
006457
垢版 |
2019/07/10(水) 11:01:16.05ID:oDIzpewt
>>28 のタイプの問題を 統一的に扱う件、少し考えてみた
y=f(x) は単調増加な連続関数とする

■1 a[1] < f( a[1] ) = a[2] の場合
開区間族 S := { (a[1],t) ; x ∈ (a[1],t) → x < f(x) }
合併区間 U:= ∪ S = (a[1], q) (非有界なら q=+∞)
α := f(q)
とする。

連続性により, 任意のε(>0) に対し δ(>0)が存在して |τ-q|≦δ → |f(τ)-α|≦ε
Uの定義より、あるτについて q<τ<q+δ かつ f(τ) ≦ τ (単調増加より f(τ)=τ である)
α-ε ≦ f(τ) = τ < q+δ ≦ f(q+δ) ≦ α+ε
ε→0 により q=α が得られる.

(具体的な αはグラフで交点を求めて、数式で Uの性質を満たす事を示せばよい )

●1.1  a[n]∈U かつ a[n+1] not∈ U となる n が存在する場合
あるτについて a[n] < τ ≦ a[n+1] かつ τ=f(τ) (∵中間値の定理)
a[n] < τ ≦ a[n+1]=f(a[n]) ≦ f(τ)=τ (∵単調増加)
よって a[n+1]=τ
αの定義より α≦τ , 単調増加より τ=f(a[n]) ≦ f(α)=α よって τ=α
α=a[n+1] = f(α) = ff(α) = f...f(α)
つまり lim a[n]=α

●1.2 a[n]∈U かつ a[n+1] not∈ U となる n が存在しない場合
a[1] < a[2] < ... < α である。
lim a[n] = β ≦ α (上に有界の場合) とする. (収束しなければ lim a[n]= +∞ = α である)
任意のε(>0) に対し ある N が存在して n>N → β-ε < a[n]<a[n+1]=f(a[n]) < β
よって lim{ f(a[n]) - a[n] }= f(β) - β = 0, 前と同様の論法で β=α
つまり lim a[n]=α

■2 a[1] > f( a[1] ) = a[2] の場合
1と同様の論法で処理

■3 a[1] = f( a[1] ) = a[2] の場合
明らか
0065132人目の素数さん
垢版 |
2019/07/10(水) 11:30:15.12ID:loHHg0S2
AB=1、AD=aの長方形ABCDと相似な長方形PAQDがあり、5点A,B,C,D,Pは同一円周上にある。
aの値を求めよ。
0066132人目の素数さん
垢版 |
2019/07/10(水) 11:39:35.34ID:oGsCS2S2
>>65
そんな図形あり得る?
ADは長方形ABCDの外接円の直径にはなり得ない
Pも同一円周上にあるなら∠APDは直角にはなり得ないのでPAQDという長方形は作れないんじゃ?
006757
垢版 |
2019/07/10(水) 11:58:47.54ID:oDIzpewt
>>64
> 連続性により, 任意のε(>0) に対し δ(>0)が存在して |τ-q|≦δ → |f(τ)-α|≦ε ...
(ここは以下のように変更可能 )
∀ε ∃τ (q-ε < τ < q) ∧ ( τ < f(τ) )
q-ε < τ < f(τ) ≦ f(q) = α ∴ q ≦ α

∀ε ∃τ ( q ≦ τ < q+ε ) ∧ ( f(τ) ≦ τ )
α= f(q)≦ f(τ)≦τ <q+ε ∴ α ≦ q
よって α = q

> あるτについて a[n] < τ ≦ a[n+1] かつ τ=f(τ) (∵中間値の定理) ...
(中間値の定理は 連続性をを使っているが、以下のように変更可能)
∀x ∈ (a[1],a[n] ) x < f(x)
a[n+1] not∈ (a[1], α ) より a[n]≦ α ≦ a[n+1]
f(α) = α ≦ a[n+1] = f(a[n]) ≦ f(α) =α
よって a[n+1] = α

つまり前提は「y=f(x) は単調増加」だけでよい。
連続性、微分可能性は不要
0068132人目の素数さん
垢版 |
2019/07/10(水) 13:04:45.79ID:yooy0TZe
移行って=超えればプラスマイナス逆になるんだよね?

b-a=3を変形したらa=b-3になったんだけどわかる人いる?
0069132人目の素数さん
垢版 |
2019/07/10(水) 13:12:23.52ID:6ZOvBEib
>>68
○=△の関係があるとき、
両辺に同じ数字や式を足す、引く、0以外の数をかける、などしても両辺イコールの関係はずっと成立する

例えば
○+3=△+3
○+a=△+aになる

なので
b-a=3
↓(両辺にaを足す)
b=3+a
↓(両辺から3を引く)
b-3=a
0070132人目の素数さん
垢版 |
2019/07/10(水) 13:22:39.79ID:9LWZSbhT
>>63
ここはおまえが出題者ぶって問題を出す場所じゃないから失せろ統合失調のガイジ
0071132人目の素数さん
垢版 |
2019/07/10(水) 13:23:31.11ID:msIkxgjl
>>63
おまえ頭の病気だって理解してる?
なんで自分だけ誰からも相手にされてないか理解できる?
007257
垢版 |
2019/07/10(水) 14:00:40.92ID:oDIzpewt
>>67 (訂正)
●1.2 にて、lim a[n] = β → lim f(a[n]) = f(β) つまり連続性を使ってしまっている.
さらによく考えてみれば, 連続でないなら α ;= f(q) で α ≠ q の場合もある.

なので、もう少し修正が必要だが、それでも連続性の前提は外せると思う。
0074132人目の素数さん
垢版 |
2019/07/10(水) 14:46:37.91ID:miBhNnEC
>>63

(1)
 (2,7) (4,11)

(2)
 3^2 = 9 ≡ 1 (mod 8)
3^n ≡ 1,3  (mod 8)
 kk ≡ 0,1,4 (mod 8)
∴ m = kk - 3^n ≠ 2,4  (mod 8)
0075132人目の素数さん
垢版 |
2019/07/10(水) 17:18:15.37ID:gOXX67fF
平面上にn個(n≥4)の点がある。
これらのうちどの4個を選んでもそれらが同一円周上にあるならば、n個の点全てはある1つの円周上にあることを示せ。
0077132人目の素数さん
垢版 |
2019/07/10(水) 17:21:27.60ID:x6WtExth
>>75
ここはあなたが問題を出す場所ではありません
迷惑なので書き込まないでください
0078132人目の素数さん
垢版 |
2019/07/10(水) 17:54:07.06ID:+QeaVgFW
高専3年
偏導関数の応用
以下urlの不等式条件付き最大値最小値問題が分かりません。
等式が成り立っている楕円上はラグランジュの未定乗数法で求まると思うのですが、楕円内の最大値最小値の求め方が分かりません。
よろしくお願いします。

https://i.imgur.com/hWbEcan.jpg
007957
垢版 |
2019/07/10(水) 17:58:14.49ID:oDIzpewt
>>72 (続き)
連続性の前提を外す件
↓こういう気持ち悪い逃げ道ができる事があるので、やめたほうがよさそう。
しょうもない追加前提が必要になってくる。
0081132人目の素数さん
垢版 |
2019/07/10(水) 19:08:13.27ID:gOXX67fF
x(x+y)-y^2=1
を満たす自然数の組(x,y)は無数に存在することを示し、そのうちxが10番目に小さい組を求めよ。
0082132人目の素数さん
垢版 |
2019/07/10(水) 19:11:35.35ID:ZKU6nTFx
>>81
なんでこの人そんなに嫌われてるの?
0084132人目の素数さん
垢版 |
2019/07/10(水) 19:43:11.77ID:mpBTQqr9
陰関数定理について質問させてください。
f(x, y)=0のとき、y’=-fx/fyという定理です。

レムニスケート曲線4(x^2+y^2)^2-(x^2-y^2)=0(https://i.imgur.com/pONXvGp.jpg)
にこれを使うと、(x, y)=(0, 0), (±√6/8, ±√2/8)にてy’=0となることがわかります。しかし、実際には(0,0)ではy'=0ではありません。なぜこのようなことが起きるのでしょうか?この点は、陰関数定理を適用できる条件を満たしていないのでしょうか?

よろしくお願いします。
0086132人目の素数さん
垢版 |
2019/07/10(水) 20:56:48.24ID:inSILswH
>>82
統合失調症はどこでも嫌われるだろw
スレの使い方明らかに間違えてるんだから
言外の意味を理解できない5chによくいる糖質の典型例
0087132人目の素数さん
垢版 |
2019/07/10(水) 22:47:15.34ID:QJF2qd7R
>>69
それは等式の性質だ
大学以降は同値関係で説明するべきである
0088132人目の素数さん
垢版 |
2019/07/10(水) 22:48:07.53ID:QJF2qd7R
>>86
統失と自閉症スペクトラムは全く別の病だが大丈夫か?
0089132人目の素数さん
垢版 |
2019/07/10(水) 22:53:13.69ID:oDIzpewt
>>78
・xyt空間曲面: f(x,y,t) = xx/4 + yy + tt = 1 (t≧0) ...(1)
・その法線ベクトル: (x/2, 2y, 2t )

・xyt空間平面: x + y + t = z ...(2)
・その法線ベクトル: (1,1,1)

■ t > 0
zが極値をとる条件は 2つの法線ベクトルが平行になっている事 (実質上はラグランジュ未定乗数法そのもの)
(x/2, 2y, 2t ) = k*(1,1,1) {k:比例係数}
(1)に代入して 4kk/4 + kk/4 + kk/4 = 1 ∴ k = ±√(2/3)
z = x + y + t = 2k + k/2 + k/2 = 3k = ±√6

■ t = 0
・xx/4 + yy = 1
・x + y = z
今度は 2次元版で同様に処理する
(x/2, 2y) = k'*(1,1)
4k'k'/4 + k'k'/4 = 1 ∴ k' = ±2/√(5)
z = 2k' + k'/2 = (5/2)*k' = ±√5

よって +√6 が最大値, -√5 が最小値である

てか... 右上に偏導関数だとか書いてあるんですけど...
え?大学受験レベルでそういうのアリなの?
0090132人目の素数さん
垢版 |
2019/07/10(水) 23:29:16.35ID:XHEBJH9i
精神分析は別スレで
0091132人目の素数さん
垢版 |
2019/07/10(水) 23:47:13.49ID:miBhNnEC
>>78

7.27
xx/4 + yy ≦ 1 の範囲を x,y が動くとき、関数
   z = x + y + √(1 -xx/4 -yy),
の最大値および最小値を求めよ。   (長岡技科大)
--------------------------------------------

x = 2a cos(t), y = a sin(t) とおく。   >>80
題意より 0≦a≦1,

(1) まづ θを一周すると
 (x+y)^2 = 5(xx/4 + yy) - (x/2 - 2y)^2 = {5 - [cos(t)-2sin(t)]^2}aa ≦ 5aa,
 (等号は y=x/4, tan(t)=1/2 のとき)
-(√5)a ≦ x+y ≦ (√5)a,
よって
 -(√5)a + √(1-aa) ≦ z ≦ (√5)a + √(1-aa),

(2) 次に 0≦a≦1 で動かす。
 -√5 ≦ z ≦ √6,
 (左等号は a=1 のとき。右等号は a=√(5/6) のとき)
0092132人目の素数さん
垢版 |
2019/07/11(木) 00:08:27.91ID:ZbtAiiQS
>>64
ありがとうございます
しかし、ちょっと用語が難しく、理解には至りませんでした
精進します。
0093132人目の素数さん
垢版 |
2019/07/11(木) 00:36:47.93ID:o5noo+Gj
>>81
フィボナッチ数を F_n とおく。
 G_n = F_n・{F_n + F_(n+1)} - {F_(n+1)}^2
 = F_n・{2F_n + F_(n-1)} - {F_n + F_(n-1)}^2
 = (F_n)^2 - F_(n-1)・{F_(n-1) + F_n}
 = - G_(n-1)
 = ・・・・・
 = (-1)^(n-1)・G_1
 = (-1)^(n-1) {F_1・(F_1+F_2) - (F_2)^2}
 = (-1)^(n-1),

∴ (x, y) = (F_{2m-1}, F_{2m}) は題意を満たす。(mは自然数)

m=10 のとき (F_19, F_20) = (4181, 6765)

>>84
 おいらの連鎖律
0094132人目の素数さん
垢版 |
2019/07/11(木) 01:02:35.25ID:o5noo+Gj
>>81

(x, y) = (1, 1) は題意を満たす。

(a+b){(a+b)+(a+2b)} - (a+2b)^2 = a(a+b) - b^2,
∴ (x, y) = (a, b) が題意を満たすならば (a+b, a+2b) も題意を満たす。(終)
0095132人目の素数さん
垢版 |
2019/07/11(木) 07:28:42.62ID:BupW6tST
>>89
なるほど!
この方法で解くとエレガントですね。
わかりやすい解説ありがとうございました。
また機会がありましたらよろしくお願いします。
0096132人目の素数さん
垢版 |
2019/07/11(木) 13:08:24.66ID:lIR+O4h8
下記のような式は微積分のどういう分野で出てくるのですか?

∫logyf(xy)dy=G(y,x)
0097イナ ◆/7jUdUKiSM
垢版 |
2019/07/11(木) 17:27:22.04ID:M0G3fYJ6
>>27
>>12正四面体PABCと単位円柱内部を足しあつめて引く場合、どっち方向に切った断面積をどっち方向に足しあつめますか?
rとθの2つの変数がある場合、どこを起点にし(どこにr=0,θ=0または0°をとり)ますか?
そこを明確に示せば式は人それぞれ勝手に立てて解くと思うんです。
こういう積分の答案には、方向と起点を書いてください。
0098132人目の素数さん
垢版 |
2019/07/11(木) 17:40:32.62ID:o5noo+Gj
r=0 は円柱の軸 (z軸、OP),
θ=0゚ は ∠BOCの中央の向き (OAと反対の方向)

なお、PABCは正四面体ではなく正三角柱。
0099Frank student
垢版 |
2019/07/11(木) 17:56:34.55ID:WsUJLiUV
>>81
ss = Solve[x^2 + x y - y^2 == 1 , y];
n = 1;
Do[
If[IntegerQ[py = (y /. ss[[2]])], Print[{n++, x, py}]], {x, 0,
10000}]

{1,1,1}

{2,2,3}

{3,5,8}

{4,13,21}

{5,34,55}

{6,89,144}

{7,233,377}

{8,610,987}

{9,1597,2584}

{10,4181,6765}

で片が付く

証明? フィボナッチが当然でてくるが、それをしらなくても なんとかできるでしょう

15以上は計算時間がかかりそうだね
0100132人目の素数さん
垢版 |
2019/07/11(木) 18:01:31.40ID:j4iePiJP
0<α<β<90とする。
PA=a、PB=bである△PABにおいて、∠A=α°、∠B=β°とする。
このとき、以下の極限を求めよ。
lim[β→α]{cosβ°-cosα°}/(b^2-a^2)
0102132人目の素数さん
垢版 |
2019/07/11(木) 19:48:26.15ID:oa3AzB1v
数学の新発見をしたと思うので証明など送ろうと日本数学会にホームページに行ったが
送り先などを受け付けるメアドなどが分からない
ちょうどそこにおいてあるログインメール先にちょっと聞きに行ったら
システムバグってる。たぶん管理者このバグで動かないのに気が付いてない。
誰か日本数学会へのアクセス知らない?

>アカウント作成(Activation)等のアクティベーションシステムに関するお問い合わせ受付は6月19日(水)17時までで一時停止しています。
>これ以降のお問い合わせについては、7月1日以降に順次対応いたします。
>どうぞよろしくお願いいたします。

>日本数学会事務局
0104132人目の素数さん
垢版 |
2019/07/11(木) 19:59:23.05ID:CAQH8pc2
Wolfram言語はプラットフォームに最適化された
最新のコードを使って,初等関数を非常に効率的に
機械精度で評価するだけでなく,多くの独自のアルゴリズムを
使って任意精度において世界最速で評価することもできる.
Wolfram言語は記号関数と変換の高度な繋がりにより,
過去には主要な数学的成果とみなされていた
結果を簡単に得て,初等関数について
厳密な数値・代数操作を行うことができる.
0105132人目の素数さん
垢版 |
2019/07/11(木) 20:15:39.35ID:q2vDyNEa
>>100
β = α + δ, b = a - δ' と置く
正弦定理より a sinα = b sinβ = (a - δ' )sin(α + δ) = (a - δ' ){ sinα + δ.cosα + o(δ^2) }
δ' sinα = a δ.cosα - δ'δ.cosα + o(δ^2) ∴ lim (δ/δ') = tanα / a
{cosβ-cosα}/(b^2-a^2) = {cos(α + δ)-cosα}/{(2a - δ')(-δ')} = sinα/(2a) * δ/δ' + o(δ^2)
= sinα/(2a) * tanα /a + o(δ^2)
→ sinα tanα /(2aa)
0106132人目の素数さん
垢版 |
2019/07/11(木) 20:33:08.51ID:oa3AzB1v
質問だけじゃまずいか
問題に答えるスレらしいので
>>81
(loop for x from 1 to 100000 do (loop for y from 1 to 100000 when (eq 1 (- (* x (+ x y)) (expt y 2))) do (format t "x:~a y:~a~%" x y)))
x:1 y:1
x:2 y:3
x:5 y:8
x:13 y:21
x:34 y:55
x:89 y:144
x:233 y:377
x:610 y:987
x:1597 y:2584
x:4181 y:6765
0107132人目の素数さん
垢版 |
2019/07/11(木) 21:54:10.02ID:oa3AzB1v
発見した内容は大まかにいえば
実数と直交する虚数とは異なるもう一つの虚数の世界がある、
3次方程式の実数解はその空間の円を3等分した写像である
さらに4次方程式も同様に扱える(Rを出し、jのデータを出し、回転させる)


こんな感じの定理は今までの数学にはある?
0108132人目の素数さん
垢版 |
2019/07/11(木) 21:57:16.57ID:oa3AzB1v
教科書で3時方程式ではカルダーノ、4次はデカルトとかでてきたけど
これ全然書いてないのが、もし知られてるならおかしい
幾何学的にも代数的にも綺麗で3時も4次も同じメソッドで解けるとか最高にクール
0109132人目の素数さん
垢版 |
2019/07/11(木) 21:58:20.05ID:eH5tM5cq
なるほど、今度の診察の時おくすり増やしてもらおうね
0111132人目の素数さん
垢版 |
2019/07/11(木) 22:05:06.24ID:eH5tM5cq
奇数の完全数くんみたいに論文()をハゲ散らかしてみたら?
0115132人目の素数さん
垢版 |
2019/07/11(木) 22:19:29.21ID:eH5tM5cq
歴史的大成果?を挙げた奇数の完全数くんですら、コツコツとwebで論文ハゲ散らかしだよ
200くらい版を重ねて
0116132人目の素数さん
垢版 |
2019/07/11(木) 22:21:08.79ID:oa3AzB1v
>>114
たいていは新発見なんて誤解で誰か見つけてるのがオチじゃん

>>107
これ知ってるよ Xxxが見つけた Xxx の定理
とかレスがつくはず

お前らのような滅茶苦茶数学出来るやつらなら
知ってる可能性が高い
0117イナ ◆/7jUdUKiSM
垢版 |
2019/07/11(木) 22:49:52.10ID:M0G3fYJ6
>>112
>>98どっち方向に切ってどっち方向に足しあつめたら、
あるいは直角三角錐台から3方向切頭単位円柱の1/6のどっち方向に切った断面積をどっち方向に引きあつめたら、
求める体積の1/6、
2√3/3-π/3
になるかが知りたいです。
0118132人目の素数さん
垢版 |
2019/07/11(木) 23:13:05.48ID:oa3AzB1v
名前のデフォルトにもなっているように、ここ素数好きみたいだから
お土産として素数ジェネレータのコード置いておくよ
https://pastebin.com/EEQkRR1W

速度の出せるC言語でキャッシュ層無しで3MBもメモリ使わない感じ
10000000000未満の素数全部書き出すコード
テキストファイルに全出力流し込むと約5GBぐらいになるはずだ
速度は1秒間に1万個ぐらい素数見つけるペースだから
たぶん日常生活で素数表見たくなったら1秒ぐらいで十分なんじゃないでしょうか
素数しばらく見ないならファイル消してコードだけ持っていればほぼファイルサイズ0で保存がきき
必要なら数ギガバイトとか出力しておけば色々調べるのに便利かも

ここの住民騒がせたちょっとしたお詫びも兼ねる
0119132人目の素数さん
垢版 |
2019/07/12(金) 01:20:30.56ID:wFPQ8Nqi
>>100
a,b が定数の場合
 sinα/sinβ = sin(∠A)/sin(∠B) = PB / PA = b/a,
 β→α とできるから  a=b?

a,b が変数の場合
 PABのサイズを自由に変更できるので不定?
0121132人目の素数さん
垢版 |
2019/07/12(金) 02:33:17.50ID:0zWfxKrH
tを実数の定数とする。
-1≤x≤1において、以下の2つの関数の値域が一致するという。
f(x)=|ax^2+tx+c|
g(x)=|cx^2+tx+a|
このとき、ac≠0である実数a,cが満たすべき関係式を求めよ。
0122132人目の素数さん
垢版 |
2019/07/12(金) 02:41:54.78ID:yVAwQ8sG
>>118

早速走らせてみました。 長くては困るので10000以下にしましたが、
言われた通りです。

Cを使わなくなってひさしいのですが、昔通りのCで懐かしいです。
ありがとうございました。
0124132人目の素数さん
垢版 |
2019/07/12(金) 06:36:50.27ID:0zWfxKrH
a,θ,αを正の定数とする。
PA=a、∠P=θ、∠A=αの△PABを考える。
∠B=βとしてβを変化させる(すなわち、それに伴いPB=xも変化する)。
このとき、以下の極限を求めよ。
lim[β→α]{cosβ-cosα}/(x^2-a^2)
0125132人目の素数さん
垢版 |
2019/07/12(金) 07:56:41.32ID:aApBTvF9
等号を同値関係ではなく
等式の意味で使っている以上
全部ゴミだと思うよ
0128132人目の素数さん
垢版 |
2019/07/12(金) 08:21:00.58ID:lAtPgwZC
p が偽の時、 p → q は真になります。


A を任意の集合とする。

p ∈ φ ⇒ p ∈ A

p ∈ φは偽だから、 p ∈ φ ⇒ p ∈ A は真である。

定義により、 φ は A の部分集合である。
0129132人目の素数さん
垢版 |
2019/07/12(金) 08:23:27.69ID:lAtPgwZC
p が偽の時、 p → q は真

という約束はただ単に便利だからそう約束するまでだ

と本には書いてあります。

>>128

空集合は任意の集合の部分集合であることを証明できたりして、確かに便利です。

>>128

の例以外で、この約束が便利な場面ってありますか?

p
0130132人目の素数さん
垢版 |
2019/07/12(金) 08:27:21.48ID:lAtPgwZC
p(x) ⇒ q(x)

普通は、 p(x) = True となるような x に対してしか p(x) ⇒ q(x) を考えません。

数学において、

p(x) = True となるような x に対してしか p(x) ⇒ q(x) を考えない

となぜ約束しないのでしょうか?

具体的にこのように約束すると不便な場面を挙げてください。
0131132人目の素数さん
垢版 |
2019/07/12(金) 08:29:10.44ID:lAtPgwZC


を二項演算と考えるとすると、確かに以下の4通りの引数に対して戻り値が定義されていないと困ります。、

True ⇒ True
True ⇒ False
False ⇒ True
False ⇒ False
0132132人目の素数さん
垢版 |
2019/07/12(金) 08:31:58.76ID:lAtPgwZC
そして、

False ⇒ True
False ⇒ False

の値を定義しなければならないというのならば、その値を真と定義するのが妥当である

ということに同意するのに吝かではありません。
0133132人目の素数さん
垢版 |
2019/07/12(金) 08:38:25.97ID:lAtPgwZC
実際に数学の本を読んでいても、

p が偽の時、 p → q は真

という約束が便利であると思うことは少ないと思います。
0134132人目の素数さん
垢版 |
2019/07/12(金) 08:41:46.64ID:lAtPgwZC
具体的に、ある数学的議論の中で、

p(x) = True となるような x に対してしか p(x) ⇒ q(x) を考えない

場合と、

True ⇒ True == True
True ⇒ False == False
False ⇒ True == True
False ⇒ False == True

と約束する場合に、

それぞれ、議論がどのようになるのか、例を挙げて示してください。
0135132人目の素数さん
垢版 |
2019/07/12(金) 08:43:22.71ID:lAtPgwZC
True ⇒ True == True
True ⇒ False == False
False ⇒ True == True
False ⇒ False == True

と約束することに関する説明として、判で押したように「便利である」とだけ書くのはいかがなものでしょうか?

うまく説明できないから逃げているとしか思えません。
0136132人目の素数さん
垢版 |
2019/07/12(金) 08:58:19.93ID:aApBTvF9
命題の真偽について疑問があるのなら
成田正雄『初等代数学』共立出版 1966
にすべて書いてある
公理化し得る定理(除法の定理)や
群やイデアルの定義など証明困難なものに対して
偽の命題を仮定して真の命題を導出している
0137132人目の素数さん
垢版 |
2019/07/12(金) 08:59:22.27ID:aApBTvF9
定義を証明するというのは異端のように思われがちだが
公理化する前の定義の証明は必要だという立場もある
0139132人目の素数さん
垢版 |
2019/07/12(金) 09:23:32.90ID:Qk2m9H+k
>>105 で o(δ^2) と 書いたとこは o(δ) に訂正する
lim{δ→ 0} o(δ) / δ → 0 となるような微小量 (ランダウ記法)
0140132人目の素数さん
垢版 |
2019/07/12(金) 11:42:59.50ID:y5NuZNNj
>96
再掲

下記のような式は微積分のどういう分野で出てくるのですか?

∫logyf(xy)dy=G(y,x)
0141132人目の素数さん
垢版 |
2019/07/12(金) 14:06:54.99ID:+l48Bv4t
質問です。210/ 5.5を計算したらx=38と10/55 になりました。これはどう計算するんでしょうか?僕がやったら38と2/11になってしまいます。
0142132人目の素数さん
垢版 |
2019/07/12(金) 14:18:25.12ID:+l48Bv4t
>>141
少し解決しました。2100/55で計算したらそうなります。なぜ、420/11で約分できるのにしないんでしょうか?
0143132人目の素数さん
垢版 |
2019/07/12(金) 14:23:54.54ID:2qynQG0w
ぼくがかんがえたさいきょおのときかたを書いてみれば?
0145132人目の素数さん
垢版 |
2019/07/12(金) 15:28:34.13ID:0zWfxKrH
m,nを自然数とする。
a[m,n] = log_2[6] - (√m + 1/√(n+1))
を最小にするm,nの組を求めよ。
0147132人目の素数さん
垢版 |
2019/07/12(金) 19:06:51.47ID:lAtPgwZC
>>136

ありがとうございます。

読んでみます。
0148132人目の素数さん
垢版 |
2019/07/12(金) 22:56:22.73ID:wFPQ8Nqi
>>145

log(6)/log(2) = 2.5849625
{log(6)/log(2)}^2 = 6.68203113
より
 m ≦ 6

a[4,2] = 0.0076122315315
a[5,7] = -0.0593537672425
a[5,8] = 0.0155611898880
a[6,53] = -0.0006100055500
a[6,54] = 0.00063278544533

(m,n) = (6,53) のとき最小。
0150132人目の素数さん
垢版 |
2019/07/13(土) 01:04:08.17ID:SQn5vctJ
>>145
a[m,n] = log_2[6] - (√m + 1/√(n+1))
をわかりやすくきじゅづしてくさい。

a[m,n] = log_2[6] - (m + 1/√(n+1))^(1/2)
a[m,n] = log_2[6] - (m^1/2 + 1/√(n+1))
or
....
0151132人目の素数さん
垢版 |
2019/07/13(土) 10:20:43.04ID:TO+CYkEf
 F(n,s) := Σ{k=0, n} C{n,k} * (-1)^k * (n-2*k)^s
とします。
 s=0, 1, ..., n-1 の時、 F(n,s) = 0
となる事を示してください。
s=0 だと (1-1)^n = 0 、n,s の偶奇が正反対の場合も簡単ですが、
n,s が偶偶 または奇奇 のパターンは、どうしたらいいのか分かりません。

例えば
F(2019, s) = 0 (s=0,1,2,..., 2019 )、F(2019, 2019) = {6407桁の数} 、F(2019, 2020)=0
となります。
0152132人目の素数さん
垢版 |
2019/07/13(土) 10:48:41.14ID:TO+CYkEf
(補足説明)
∫ {x=-∞, +∞} dx (sin(x) / x)^n = lim{ε→0} ∫ {z=-∞, +∞} dz sin(z)^n / (z^n + iε^n) = ...
の計算途中に、 F(n,s) / ε^{n-s} に比例する項が現れます。
この積分の ε→0 収束が F(n,s) = 0 (s=0, 1, ..., n-1) を保証しているはずですが
もう少し直接的な証明を知りたいです。
0153132人目の素数さん
垢版 |
2019/07/13(土) 12:31:16.92ID:TO+CYkEf
>>151 自己解決しました

F(n,s) := Σ{k=0, n} C{n,k} * (-1)^k * (n-2*k)^s (s=0,1,...,n-1)

・k についての多項式: (n-2k)^s の次数は n-1 以下である
・Σ{k=0, n} C{n,k} * (-1)^k * k^t (t = 0,1..., n-1)
 = Σ{k=0, n} C{n,k} * (-1)^k * (d/dα)^t e^{kα}  (α→0)
 =(d/dα)^t Σ{k=0, n} C{n,k} * (- e^{α} )^k  (α→0)
 =(d/dα)^t (1-e^{α})^n  (α→0)
 =(d/dα)^t (1-e^{α}).... (1-e^{α})  (α→0)
 = 0 (∵ t < n)
よって F(n,s) = 0 (s=0,1,...,n-1) である。
一晩くらい悩んだのに意外と簡単だった。

ついでに F(n,n) の式も求まった。
F(n,n) := Σ{k=0, n} C{n,k} * (-1)^k * (n-2*k)^n
= (-2)^n * Σ{k=0, n} C{n,k} * (-1)^k * k^n
= (-2)^n * (d/dα)^n (1-e^{α}).... (1-e^{α})  (α→0)
= (-2)^n * n! * (-e^{α})...(-e^{α})  (α→0)
= 2^n * n!
0154イナ ◆/7jUdUKiSM
垢版 |
2019/07/13(土) 16:01:10.38ID:JAfIi0Rb
>>149
△PBCを含む平面の式が、
z=2y+2=2(1+y)
√3/2≦x≦√3の範囲で、z=0平面と△PBCを含む平面とy=-x/√3平面とで囲まれた部分の体積は、
(1/6)(√3-√3/2){-1/2-(-1)}2[1+{-(√3/2)/√3}]
=(1/6)(√3/2)(1/2)
=√3/24――@
0≦x≦√3/2の範囲をどう出すか。
y=-2x/√3をx^2+y^2=1に代入し交点の座標を求めると、
x^2+4x^2/3=1
x^2=7/3
(√21/3,-2√7/3)
√3/2≦x≦√21/3の範囲で、z=0平面と△PBCを含む平面とy^2+y^2=1曲面とで囲まれた部分の体積は、
    ――A
0≦x≦√21/3の範囲で、z=0平面と△PBCを含む平面とy^2+y^2=1曲面とで囲まれた部分の体積は、
    ――B
@ABより、
求める体積は、
6(@+A+B)=4√3-2π

rとθで表すにしても、どの体積をどの式で表すかきちんと書くべきだと思う。
0155イナ ◆/7jUdUKiSM
垢版 |
2019/07/13(土) 18:23:14.39ID:JAfIi0Rb
どうだった? __人人__
/__人人__/__/(__^_)_
/_(__)_)_/__/(____)_
/_( ___)_/__/(^o^))_
/_(_(`)_/__/_(__っ┳
/_(υ__)┓__◎゙┻υ◎゙
◎゙υ┻-◎゙_/__/キコキコ……/__/__/_キコキコ……__/__/__/__/前>>154やっぱり>>12のように解くしかないか。
0157132人目の素数さん
垢版 |
2019/07/13(土) 19:45:35.91ID:TO+CYkEf
>>156
log[(x^2)+2] = log(x + i√2) + log(x - i√2) より
不定積分: ∫ dx log[(x^2)+2] = (x + i√2)log(x + i√2) - (x + i√2) + (x - i√2)log(x - i√2) - (x - i√2)
= x log(x^2 + 2) + i√2 log[(x + i√2)/(x - i√2)] - 2x
= x log(x^2 + 2) - 2√2 arctan[ (√2) / x ] - 2x (←導出過程が怪しく思うならコレを微分して確かめるとよい)
あとは数値を入れるだけ
0158107
垢版 |
2019/07/13(土) 23:48:47.25ID:ZQatj4m1
sample

正規化して x^3 -13 * x から円の半径R = 2 * ( (13/3)^2 ) を得る
a を実数解の一つとすると、その虚数i とは異なる軸j のデータは (52 - a^2)^0.5 である
複素数平面とj軸を合わせた立体空間を120°回転させて他の2つの解を得る
つまり他2つの解は (-a +- (52 - 3 * (a^2) )^0.5 )/2 である
これが整数解かつ実数の条件で解くと a= +-1 等を得る(普通は6個解が出る)
a=1 の時他の解は代入して -4, 3 であり
a=-1 の時他の解は代入して 4, -3 である
0159132人目の素数さん
垢版 |
2019/07/13(土) 23:49:30.75ID:ZQatj4m1
コピペみす

kは整数である。3次方程式
x^3 -13x +k = 0
は3つの異なる実数解をもつ。その3つの解とは?


正規化して......
0162132人目の素数さん
垢版 |
2019/07/14(日) 00:02:22.10ID:vRXSIFAS
>>160
似ているが違う
そもそも自分は i j を認識しているが kを認識できてない
そして
ij = k となっているが
自分の見つけた法則ではkに移らない

たとえば j 軸のデータのルートの中が負だと
虚数i で表わせるけど
真ドモアブルの定理を使うと複素数平面(皆が使う 実数と虚数iの座標)
に変換される。断じてkではない
0165132人目の素数さん
垢版 |
2019/07/14(日) 00:11:47.33ID:vRXSIFAS
三元数と命名するのやめた
2元数 と 4元数 なに書いてあるのかぜんぜん理解できない
もし命名してたら混乱まねきかねない
0166132人目の素数さん
垢版 |
2019/07/14(日) 00:19:03.85ID:vRXSIFAS
>>158
>>159
いままでの定番だと1つの実数解をaとしたら
他の2つの解がk まじるよね?
私のはaで表記できます
しかも
カルダーノの難しい式のような難解な3乗根もいらない
0167イナ ◆/7jUdUKiSM
垢版 |
2019/07/14(日) 00:29:09.21ID:zPm799N0
>>12四面体PABC-(1/2)^3四面体PABC-6(四面体内部から削り出す0≦z≦1,0°≦θ≦60°の部分)
=2√3(7/8)-6(π-√3)/3
=7√3/4-2π+2√3
=15√3/4-2π
最初は2√3-6(π-√3)/3
=4√3-2πかと思ったんですが、(π-√3)/3を6つ削り出す元の立体は、0≦z≦1だと四面体PABCの1/8は除外され体積(2√3)(7/8)=7√3/4の三角錘台になるんじゃないでしょうか?  前>>155
0168イナ ◆/7jUdUKiSM
垢版 |
2019/07/14(日) 02:17:25.57ID:zPm799N0
>>167それとも>>12の、
v/2=∫[0〜1]∫[0〜60゚]h(r,θ)dθrdr
=∫[0〜1]∫[0〜π/3]2(1-rcosθ)dθrdr
=∫[0〜1]2rdr∫[0〜π/3]dθ-∫[0〜1]r^2dr∫[0〜π/3]2cosθdθ
=(π/3)-(1/3)[2sinθ](0→π/3)
=(π-√3)/3
はOBを斜辺とする直角三角形(△OBCの半分)を底辺とする三角錘の頂点Pを含む体積ですか?
0169132人目の素数さん
垢版 |
2019/07/14(日) 02:50:36.36ID:inP0wo2T
tを正の実数とする。
曲線C:y=ln(x)
の区間[t,t+1]の部分の長さをL(t)とする。
2点A_t、B_tをA_t(t,ln(t))、B_t(t+1,ln(t+1))と定める。
点A_tでのCの接線の、区間[t,t+1]での長さをA(t)とする。また線分A_tB_tの長さをB(t)とする。

(1)L(t)とA(t)の大小を比較せよ。

(2)lim[t→∞] L(t) = 1 を示せ。

(3)実数p[t],q[t],r[t]を以下の規則で定める。
(a)p[t]はA(t),B(t),L(t)のうち最大でないもの2つの相加平均
(b)q[t]はA(t),B(t),L(t)の相加平均
(c)r[t]はA(t),B(t),L(t)のうち最小でないもの2つの相加平均
このとき、以下の極限を求めよ。
lim[t→∞] {p[t]+r[t]-2}/{q[t]-1}
0172132人目の素数さん
垢版 |
2019/07/14(日) 12:46:27.27ID:47siAlyQ
>>169
このスレはわからない問題を書くスレですその問いの特に何が分からないんですか?
0174132人目の素数さん
垢版 |
2019/07/14(日) 14:19:13.71ID:Xfj84fYJ
>>151
s次多項式は
 1, x, x(x-1), x(x-1)(x-2), ・・・・, x(x-1)・・・・(x-s+1)
の形の和で表わせる。

そこで >>153 を少し変えて
Σ{k=0,n} (-1)^k ・ C{n,k} ・ k(k-1)・・・・(k-s+1)
 = Σ{k=0,n} (-1)^k・C{n,k}・{(∂/∂β)^s β^k} (β→1)
 = (∂/∂β)^s {Σ{k=0,n} (-1)^k・C{n,k}・β^k} (β→1)
 = (∂/∂β)^s {(1-β)^n} (β→1)
 = (-1)^s {n!/(n-s)!} {(1-β)^(n-s)} (β→1)
 = 0        (s=0,1,・・・・,n-1)
 = (-1)^n ・ n!   (s=n)

あるいは直接

Σ{k=0,n} (-1)^k ・ C{n,k} ・ k(k-1)・・・・(k-s+1)
 = Σ{k=s,n} (-1)^k ・ {n!/(n-k)!・(k-s)!}
 = (-1)^s {n!/(n-s)!} Σ{L=0,n-s} (-1)^L C{n-s,L}
 = (-1)^s {n!/(n-s)!} (1-1)^(n-s)
 = 0       (s=0,1,・・・・n-1)
 = (-1)^n ・ n!  (s=n)
0175132人目の素数さん
垢版 |
2019/07/14(日) 14:22:26.54ID:Xfj84fYJ
>>156

部分積分で
∫ log(xx+2) dx = x・log(xx+2) - ∫ 2xx/(xx+2) dx
 = x・log(xx+2) + ∫{4/(xx+2) -2} dx
 = x・log(xx+2) + (2√2)∫1/(tt+1) dt -2x
 = x・log(xx+2) + (2√2)arctan(t) -2x
 = x・log(xx+2) + (2√2)arctan(x/√2) -2x,

0→√2 で積分すると
 (2√2){log(2) + π/4 -1} = 1.35353063127
0176132人目の素数さん
垢版 |
2019/07/14(日) 14:28:15.88ID:Xfj84fYJ
>>159 >>166
 「1つの実数解をaとするとき、他の2つの解をaで表記せよ。」
なら
 b+c = -a,
 bc = aa -13,
から補助方程式
 tt + at + aa -13 = 0,
これを解いて
 b, c = {-a ± √(52-3aa)}/2,
0177132人目の素数さん
垢版 |
2019/07/15(月) 01:17:59.85ID:LQZPlIQC
1から99までの整数Nで、|sinN|を最小にするものを求めよ。
必要があれば以下を用いて良い。
π=3.1415926535...
π=[3; 7, 15, 1, 292, 1, 1, 1, 2, ...]
0178132人目の素数さん
垢版 |
2019/07/15(月) 01:50:11.33ID:UJ+igGkh
むしろ

自然数 n にたいして 実数列a[n]を a[n]=Inf{sin(i)| 1≦i≦n} で定める。
lim{n →∞}a[n] の値を知りたい
0179132人目の素数さん
垢版 |
2019/07/15(月) 02:18:11.33ID:bOVU3cFf
limit[a[n]] = -1
0180132人目の素数さん
垢版 |
2019/07/15(月) 02:29:58.10ID:bOVU3cFf
sin[22]= 0.00885131
0181132人目の素数さん
垢版 |
2019/07/15(月) 02:36:09.04ID:bOVU3cFf
{Pi+2m Pi,{m,1,15}}
{9.42478, 15.708, 21.9911, 28.2743, 34.5575, 40.8407, 47.1239, \
53.4071, 59.6903, 65.9734, 72.2566, 78.5398, 84.823, 91.1062, 97.3894}

21.9911 --> 22
0182132人目の素数さん
垢版 |
2019/07/15(月) 02:39:37.01ID:3fq4SDZ7
>>178
1/(2π) が無理数なので 複素平面上に z=e^{ik}=e^{i2π* (k/(2π))} k=1,2,3,.... をプロットすると 単位円を稠密に覆い尽くす。(クロネッカーの稠密定理より明らか)
特に z= -i の任意のε近傍にいくらでも散らばっている。
つまり ∀ε ∃k |sin(k) - (-1)| = | Im(z - (-i)) | ≦ | z - (-i) | < ε
よって lim{n →∞}a[n] = -1
0183132人目の素数さん
垢版 |
2019/07/15(月) 02:45:25.90ID:bOVU3cFf
Integrate[Log[x^2] + 2, {x, 0, Sqrt[2]}]
==Sqrt[2] Log[2]

=0.980258
0188132人目の素数さん
垢版 |
2019/07/15(月) 04:49:44.99ID:hmERs5gh
>>177
 sin(11) = -0.999990206550703
 sin(55) = -0.999755173358620
 sin(99) = -0.999206834186354
のどれか。
0189132人目の素数さん
垢版 |
2019/07/15(月) 04:53:08.03ID:hmERs5gh
>>178
 f(i) = i - 2π[i/2π]
とおくと
 0 ≦ f(i) < 2π

 f(0), f(1), f(2), ・・・・, f(m) の(m+1)個の中の或る i≠j について
 0 < |f(i) - f(j)| < 2π/m,
 n を |i-j| ずつ増加したときの f(n) の変化は 2π/m より小さい。

 任意の ε>0 をとって m = [π/√(2ε)] + 1 とおく。
 |f(i) - f(j)| < 2√(2ε),
 n を |i-j| ずつ増加したときの f(n) の変化は 2√(2ε) より小さい。
 ある f(n) は区間 [ -(π/2)-√(2ε), -(π/2)+√(2ε)] の中に入る。
 sin(n) = sin(f(n)) < sin(-(π/2)±√(2ε)) = -1 + 2{sin√(ε/2)}^2 < -1 + ε,
0191132人目の素数さん
垢版 |
2019/07/15(月) 09:21:25.10ID:LQZPlIQC
tを実数の定数とする。
-1≤x≤1において、以下の2つの関数の値域が一致するという。
f(x)=|ax^2+tx+c|
g(x)=|cx^2+tx+a|
このとき、ac≠0である実数a,cが満たすべき関係式を求めよ。
0196132人目の素数さん
垢版 |
2019/07/15(月) 15:03:25.62ID:6ZGn4nmc
前層 F の層化とは層 [F] と準同型 f: F→[F] であって、次の普遍性を満たすものです:
任意の層 G と任意の準同型 g: F→G に対し、準同型 h: [F]→G が一意的に存在して g = hf となる

層化が存在するとすれば [F] は同型を除いて一意的です。では、f は一意的でしょうか?
0197132人目の素数さん
垢版 |
2019/07/15(月) 15:36:14.28ID:0qY5hNMz
>>196
G=[F]とするとgとfを変換してくれるhが存在するわけだから
同型を除いて一意じゃないの?
0199132人目の素数さん
垢版 |
2019/07/15(月) 16:53:12.24ID:Zl9fl5pZ
理三余裕な人でもなきゃ二次関数絡む大学受験の難問なんていっぱいあるだろ
理一合格者平均程度でも25分で完答厳しいやつな
0201132人目の素数さん
垢版 |
2019/07/15(月) 18:13:36.21ID:4xRy/BZH
コーシーリーマンの方程式で∂u/∂x=∂v/∂y
∂v/∂x=-∂u/∂yがある開集合Dで成り立ってるとしてΔu=Δv=0 in Dを示すときに例えば∂u/∂x=∂v/∂y-➀とするの両辺をxで偏微分したときも➀は成り立つのですか?
(u(x,y)とv(x,y)はDにおいてC2級、Δはラプラシアンです)
基本的なことですみません
0203132人目の素数さん
垢版 |
2019/07/15(月) 19:20:38.13ID:3fq4SDZ7
>>201
z = x + iy , f(z) := F(x,y) = u(x,y) + i v(x,y)
コーシーリーマン等式により f(z) は Dで正則である。
∂^2u/∂x^2 = ∂^2v/∂x∂y = ∂^2v/∂y∂x = -∂^2u/∂y^2 ∴ Δu = 0
∂^2v/∂x^2 = -∂^2u/∂x∂y = -∂^2u/∂y∂x = -∂^2v/∂y^2 ∴ Δv = 0

g(z) := U + iV = ∂u/∂x + i ∂v/∂x ( = ∂v/∂y - i ∂u/∂y ) とすると、
∂U/∂x = ∂^2u/∂x^2 = -∂^2u/∂y^2 = ∂V/∂y
∂U/∂y = ∂^2v/∂y^2 = -∂^2v/∂x^2 = -∂V/∂y つまりコーシーリーマン等式が成立する。
よって g(z) もまた正則であり、ΔU = ΔV = 0 である。 ( 求めていたのはこれで良い?)

さて、δz = δr.e^{iθ} = δr.cosθ + i δr.sinθ = δx + iδy と置くと
δf/δz := { (f(z+δz) - f(z))/ δr }. e^{-iθ}
= { (∂u/∂x+i∂v/∂x).cosθ + (∂u/∂y+i∂v/∂y).sinθ }. e^{-iθ} + o(δr)
= { (∂u/∂x+i∂v/∂x).cosθ + (-∂v/∂x+i∂u/∂x).sinθ }. e^{-iθ} + o(δr)
= { (∂u/∂x + i∂v/∂x).(cosθ + i.sinθ) }. e^{-iθ} + o(δr)
= ∂u/∂x + i∂v/∂x + o(δr) = g(z) + o(δr)
よって g(z) = f’(z) である。
0206132人目の素数さん
垢版 |
2019/07/15(月) 19:25:49.56ID:NUfAXLU0
高校生からやりなおし
0207イナ ◆/7jUdUKiSM
垢版 |
2019/07/15(月) 19:50:09.48ID:07zimsFQ
>>168復習。
四面体PABC=2√3
そのうち円柱内部
x^2+y^2≦1
の1/6は、
∫[0→1][0→π/3]2(1-rcosθ)rdrdθ
=∫[0→1][0→π/3]2rdrθ-∫[0→1][0→π/3]2r^2cosθdrdθ
=1・(π/3)-(2/3)sin(π/3)
=π/3-√3/3
=(π-√3)/3
求める体積=2√3-6{(π-√3)/3}
=4√3-2π
0208132人目の素数さん
垢版 |
2019/07/15(月) 20:16:58.77ID:N2pjtPVo
なに?x=yのときに写像fで写してf(x)=f(y)が成り立つのかってこと???
0209132人目の素数さん
垢版 |
2019/07/15(月) 20:45:01.81ID:lFI10ZKS
明示はしてないが基本的に中への写像が成り立つという前提で議論をしている
しかしこのポンコツ等式で証明した気になっても困るわ
何を前提にしているのかすべて書き出さなきゃ証明の価値がない
なぜか?
公理体系を無視した証明はゴミだからだよ
0210132人目の素数さん
垢版 |
2019/07/15(月) 21:05:30.08ID:4xRy/BZH
>>203
2,3段落が何をしているのかわからないのですが、∂u/∂x=∂v/∂yならば
∂^2u/∂x^2=∂^2v/∂y∂xになるということでいいですか?
0211132人目の素数さん
垢版 |
2019/07/15(月) 21:28:11.82ID:YPtGQj8g
俺がいいと言ったらいいのかよ
0212132人目の素数さん
垢版 |
2019/07/15(月) 21:51:47.40ID:mwSlliUK
>>204
方程式とあなたは言ってますが、なにに関するどのような方程式ですか?

それがはっきりすればわかるはずですね
0213132人目の素数さん
垢版 |
2019/07/15(月) 22:21:41.97ID:qbQnTpgy
わからない問題ってわけでもないんですがお聞きします。

Σ1/n^2 = π^2/6

を証明するのに

1) α(n,k)=sin^2(kπ/2n) (k:1〜n)を解とする整式を作る。
2) 解と係数の関係からs(n) = Σ[k]1/α(n,k)を求める。
3) lim s(n)/n^2 から求める値を求める。

みたいな証明があるんですが、これ大元はロシア人の論文で比較的新しいものが出典らしいとどこかのスレで書いてありました。
しかしどのスレで見かけたのか忘れてしまいました。
コレの元論文なにかわかりますか?
0215132人目の素数さん
垢版 |
2019/07/16(火) 01:32:51.64ID:pvjqiRFK
単純なミスが多いんだけどどうしたらいい?二桁の掛け算でもうっかりミスが多い。

例えば6かける1普通に1にしてしまったりする。集中ないのかね?

うっかりミスどころか記憶に定着してないのかと不安になる
0216132人目の素数さん
垢版 |
2019/07/16(火) 01:44:47.20ID:BEPPb1ZZ
互いに素な自然数a,b,cは等式
a+b=c
を満たす。
aの全ての素因数を要素とする集合をS_aとし、b,cについても同様にS_b,S_cを作る。
和集合S_a∪S_b∪S_cの全ての要素の積をf(a,b;c)とするとき、以下の等式を成り立たせる組(a,b,c)は有限個しか存在しないことを示せ。
c>f(a,b;c)^2
0217132人目の素数さん
垢版 |
2019/07/16(火) 03:38:33.11ID:BSOiXZcn
有限個なら通常のabc予想が正しければ正しい。
2乗にしているなら存在していないっていう強予想
0218132人目の素数さん
垢版 |
2019/07/16(火) 08:29:06.78ID:HD2mKkeG
lim[n=1→∞](-1)^n/n^2
lim[n=1→∞](-1)^n/n^3
lim[n=0→∞](-1)^n/(n!)^2
を高校数学の範囲で求めたいのですが、可能でしょうか?
(-1)^n/nの無限和はIn=∫[1,e](logx)^n/xdxを設定して、漸化式立ててとき進めたらlog2って求められたのですが、n^2,n^3の時がどうしても分かりません…
0219132人目の素数さん
垢版 |
2019/07/16(火) 08:40:25.18ID:HD2mKkeG
あと、(-k)^n/n! (k>0)の無限和は、
In=∫[0,k]x^n e^xdxと設定して漸化式作って、いろいろしたら(1-e^k)/e^kって求められたのですが、(n!)^2の時がどうしてもわからないです
0220132人目の素数さん
垢版 |
2019/07/16(火) 09:18:51.55ID:jGB9TIiM
>>215
マジレスしとこう
単純計算は、2回計算して計算した結果が合うことを確かめる。
何度も計算ミスしているうちに計算ミスをする場所が大体わかってくるから
そこに関してだけでも2回計算する

小学生用のドリルで二けた×二けたの問題でも毎日20問位やっておく
0221132人目の素数さん
垢版 |
2019/07/16(火) 09:19:14.52ID:yHQfAv3M
>>215
2回解く(倍かかるが包括的なエラー検出)
概算(大まかなエラー検出)
mod10(間違いは1の位に集中しているならこれはなかなか)
...(速さと精度のトレードオフの何か)
0222132人目の素数さん
垢版 |
2019/07/16(火) 10:29:53.63ID:yRQIihLg
>>218
Σ[n=1,∞](-1)^n/n^2 =-(1/2)ζ(2)= -π^2/12 : 高校数学で可能、検索キーワード「バーゼル問題」
Σ[n=1,∞](-1)^n/n^3 = -(3/4)ζ(3) : リーマンゼータ関数ζ(s)の定義を知っていれば高校数学で可能
Σ[n=0,∞](-1)^n/(n!)^2 = J0(2) : ベッセル関数Jn(x)の定義を知っていれば高校数学で可能
0226132人目の素数さん
垢版 |
2019/07/16(火) 12:30:10.17ID:HD2mKkeG
>>222
うーむ…
諦めますた
教えてくれてありがとう
0227132人目の素数さん
垢版 |
2019/07/16(火) 14:19:39.41ID:q4msH7U6
互いに素な自然数a,b,cは等式
a+b=c
を満たす。
aの全ての素因数を要素とする集合をS_aとし、b,cについても同様にS_b,S_cを作る。
和集合S_a∪S_b∪S_cの全ての要素の積をf(a,b;c)とする。
このとき、以下の等式を成り立たせる組(a,b,c)は存在しないか、または有限個しか存在しないことを示せ。
c>f(a,b;c)^2
0229132人目の素数さん
垢版 |
2019/07/16(火) 16:32:27.64ID:9KeKBZR4
>>227
反例
c=2^N, 3<=Nとおく
a, b のどちらかが合成数ならば
与式はつねに成り立つ
0231132人目の素数さん
垢版 |
2019/07/16(火) 17:12:22.17ID:W9kIC/E2
一辺の長さが√2の正方形ABCDの周及び内部の領域をSとする。
PA+PB+PC+PD=1+2√2となるようにS上を動く点P全体からなる閉曲線で囲まれる部分の面積を求めよ。
0233132人目の素数さん
垢版 |
2019/07/16(火) 17:38:29.27ID:Bnubg+Ut
>>232
スレタイも読めないのによく日本語かけたね
コピペ?
0234132人目の素数さん
垢版 |
2019/07/16(火) 18:21:16.79ID:X4bwU8E6
原点から縦軸に光を発射し、点(0,1)で反射して横軸と交わる点をXとする
反射角がランダム、つまり一様分布U(-π/2,π/2)に従うとき、Xの密度関数を求めよ

答えが1/(π(1+x^2))となるのですが途中式が分からないのでお願いします
0236132人目の素数さん
垢版 |
2019/07/16(火) 20:09:48.44ID:W9kIC/E2
任意の正の実数aに対し、
∫ [0 to a] 1/{(1+x^2)exp(x^2)} dx < 4/5
が成り立つことを示せ。
必要ならばx=tan(ln(t))と置換せよ。
0237132人目の素数さん
垢版 |
2019/07/16(火) 21:20:30.99ID:8CbDFjrq
f, g をθ,x 其々についての密度関数とすると
f(θ) dθ = g(x) dx が成り立っています。

配置条件より x = tanθ
dθ/dx = 1/{dx/dθ} = (cosθ)^2 = 1/( 1 + xx )

f(θ) は -π/2 ≦ θ ≦ +π/2 一定なので f(θ) = 1/π

よって g(x) = f(θ) dθ/dx = 1/ {π.( 1 + xx )}
0238132人目の素数さん
垢版 |
2019/07/16(火) 21:30:24.45ID:8CbDFjrq
>>234
補足説明
> f(θ) dθ = g(x) dx が成り立っています
つまり ∫[θ=a,b] f(θ) dθ = ∫[x=θ(a),θ(b)] g(x) dx という事で、
これが言えるのは x = tanθ [θ:-π/2, +π/2] のような x と θ が一対関係の場合のみである事に注意する必要があります。
一対一でない時は多少の書き換えが必要です。
0240132人目の素数さん
垢版 |
2019/07/16(火) 23:17:32.26ID:Z38S986h
x,y は整数
|a|はaの絶対値
このとき
|x|+2|y|==100 をみたす{x,y}は何個あるか?

また
|x|+2|y|== k のときは如何? (kは自然数)


おながいします。
0241132人目の素数さん
垢版 |
2019/07/16(火) 23:21:31.57ID:7gEN3YuF
なんで名無しにwwwつけてんのじゃねーんだよ
0242132人目の素数さん
垢版 |
2019/07/16(火) 23:26:02.81ID:Z38S986h
n>=6 ,x,y,z は自然数のとき

x^n+y^n == z^n
の解は\存在しないことを証明せよ
0243132人目の素数さん
垢版 |
2019/07/16(火) 23:35:32.05ID:Z38S986h
>>215

人は間違うものです。 程度問題です。
自分で欠点のところが具体的にわかればそこをなおせばよい。

漠然としていたら、 必ず間違えると覚悟して
丁寧に問題を溶ける環境をつくる。
無理なときは、2回計算するか、検算する。

別の解釈で問題をとき合致している腕化を見る。

独自の判定法を自分で在庫にしておく


などなど。。。

人間はまちがえるものです。。。。。
0244132人目の素数さん
垢版 |
2019/07/16(火) 23:40:01.42ID:W9kIC/E2
自然数nを10進法表記したときの先頭の数字をa[n]、末尾の数字をb[n]、c[n]=10a[n]+b[n]とする。
例えばn=25597のとき、a[n]=2、b[n]=7、c[n]=27 である。
なおnが1桁のときは、a[n]=0、b[n]=nとする。

いま、数列K[j]を以下で定義する。
K[1]=N
K[j+1]=P[j]K[j]+1
ここに、P[j]はj番目の素数を表す。


(1)どのような自然数Nに対しても、C[ K[i] ] = 87となる、ある自然数iが存在することを示せ。

(2)dを1以上99以下のある整数とする。(1)において、C[ K[i] ] = dの場合はどうか。
0246132人目の素数さん
垢版 |
2019/07/17(水) 00:58:00.64ID:d6A0g1cK
>>236

解1
exp(xx) > 1+xx より
(与式) < ∫[0,a] 1/(1+xx)^2 dx
 = (1/2)a/(1+aa) + (1/2)arctan(a)
 < π/4
 = 0.7853981634

解2
GM-AM より exp(-xx)/(1+xx) < {1/(1+xx)^2 + exp(-2xx)}/2,
(与式) < (1/4)a/(1+aa) + (1/4)arctan(a) + (1/8)√(2π)erf((√2)a)
 < {π + √(2π)}/8
 = 0.706027616

なお、真値は (eπ/2)・erfc(1) = 0.67164671
0247132人目の素数さん
垢版 |
2019/07/17(水) 01:53:29.87ID:a3oDTUjX
任意のn個の実数x1,x2,x3,…,xnに対して、
fn(x1)+fn(x2)+…+fn(xn)=nfn(x1+x2+x3+…+xn)
fn(0)=a(a≠0∧a∈R)
を満たす、関数fn(x)は一意に定まりますか?

全ての自然数nに対して一意に定まるか?
↓(定まらない場合)
一意に定まるようなnは求められるか?

一意に定まるため必要なfn(y1),fn(y2),fn(y3),…(y1,y2,…は実数の定数)の最小の個数は求められるのか?

が知りたいです
0248132人目の素数さん
垢版 |
2019/07/17(水) 01:55:24.44ID:a3oDTUjX
投下して気づいたのですがn=1で既に一意に定まらないっすね…すいません
0249132人目の素数さん
垢版 |
2019/07/17(水) 01:59:19.72ID:a3oDTUjX
連投すいません
逆にn≧2で一意に定まらない証明とかってのは可能でしょうか?
0251132人目の素数さん
垢版 |
2019/07/17(水) 16:04:18.83ID:QQCXZMLE
俺がスレのレベルを測ったるでー ってノリなんだよね。 何様だよって話。
まあ、それに答える人がいてもいいんだけど、
誰も答えないからってフッ...こんなもんかw みたいに思われても困るんだよね。
いつも同じ人が見てるワケじゃないし、興味持てる問題/持てない問題は人それぞれだし。
0252132人目の素数さん
垢版 |
2019/07/17(水) 16:08:32.14ID:asCld8kq
思われても困らんけど?
アホはスルー、反応するやつはアホを増長させるので同類
0253132人目の素数さん
垢版 |
2019/07/17(水) 16:58:59.26ID:ruiOzpbL
反応なくてもずっと書いてるきちがいニートなんだから無視しとけばいいだろ
粘着すんなボケキッズ
0254132人目の素数さん
垢版 |
2019/07/17(水) 17:20:37.59ID:xFNgvUqK
でも書いてる奴もさすがに自分がこのスレで賢い方だとは思ってない気はするけどね。
0255132人目の素数さん
垢版 |
2019/07/17(水) 17:27:26.12ID:QQCXZMLE
自分はよくこのスレに世話になってた側だからね、それなりに愛着あるわけよ。
無視すべきなんだろうけど「スレのレベルを測ったるでー」系にはイラっとするんだ。
0256132人目の素数さん
垢版 |
2019/07/17(水) 17:30:12.03ID:sbVq1zcp
何いってんだこのガキ
0257132人目の素数さん
垢版 |
2019/07/17(水) 17:34:53.54ID:CwSMJP8/
何にせよ、答がわかっている問題を書く行為はスレチじゃないのかい?
0258132人目の素数さん
垢版 |
2019/07/17(水) 17:40:29.76ID:lPW5WRc1
スレチであることに異論あるやつはいないだろ
キッズがIDコロッたのか?
0260132人目の素数さん
垢版 |
2019/07/17(水) 19:08:02.06ID:J3uEXj15
命題が真なのか分からないのですが、一番下の行の命題の証明を教えて下さい
解析力学のラグランジュ方程式の導出で必要になった式です
読みにくくてすみませんがよろしくお願いします
https://i.imgur.com/nwU5Ge8.jpg
0261132人目の素数さん
垢版 |
2019/07/17(水) 19:10:04.96ID:J+9Sqlae
失礼します。
低レベルな質問かもしれないですがご容赦ください。
二重積分で、xで積分するということはイメージが湧くのですが、yで積分するというのがイマイチイメージが湧きません。
どんなイメージか教えていただけませんか?
0262132人目の素数さん
垢版 |
2019/07/17(水) 19:32:39.79ID:yEsdrfM4
重積分ってことは2変数関数z=f(x,y)だと思うけど……xもyも同じでしょ
もしかして1変数のイメージのまま、反射的にyをxの関数(従属変数)だと思い込んでたりしない?
0263132人目の素数さん
垢版 |
2019/07/17(水) 19:37:30.76ID:J+9Sqlae
>>262
なんというか…
xで積分するということはxy平面状で帯?を集めて平面の面積が出るイメージなんですが
そのあとyで積分するとなるともう一度同じように面積を出している感じで考えています。もう既に間違えているような気がしますが…
うまく説明できなくてすみません。
0266132人目の素数さん
垢版 |
2019/07/17(水) 20:15:47.81ID:J+9Sqlae
>>265
ありがとうございます。
とんでもない誤解をしていたことがわかりました。
やっとイメージできました。
0268261
垢版 |
2019/07/17(水) 20:52:14.34ID:J+9Sqlae
>>267
わかりました。
やってみます。

答えていただいた方ありがとうございました!
0269132人目の素数さん
垢版 |
2019/07/17(水) 22:23:28.31ID:JxkPCs7L
Σ[k=0,∞]( (λz)^k / k! ) = e^(λz)
これってどうやって導出するのでしょうか
0271132人目の素数さん
垢版 |
2019/07/17(水) 23:02:23.73ID:EBvlCZ1f
正の整数nの各位の数の和をs(n)とする。
例えばs(1)=1,s(15)=1+5=6,s(510)=5+1+0=6である。
s(n^5)=nとなるような最大のnを求めよ。

これ解ける方いませんか?
0272132人目の素数さん
垢版 |
2019/07/17(水) 23:05:37.14ID:d6A0g1cK
f(z) = e^(λz)
より
f '(z) = λ・f(z)   ・・・・・ (*)

f(z) のマクローリン展開は |z|<∞ で収束する:
f(z) = Σ[k=0,∞] a_k z^k,
とおくと a_0 = f(0) = 1,
(*) から k・a_k = λa_{k-1}
∴ a_k = λ^k / k!
0274132人目の素数さん
垢版 |
2019/07/17(水) 23:27:59.34ID:vUfI/lS0
>>263
二次元をそもまま三次元に拡張してxyz空間を考えればいい
z=f(x,y)として、与えられた範囲のxy平面上にわたってf(x,y)を足し合わせて体積めるのが二重積分
0275132人目の素数さん
垢版 |
2019/07/17(水) 23:35:29.24ID:eA1vCJXq
ここの回答者って簡単な問題にはすでに解決済みの質問だとしても回答するんですね
0276132人目の素数さん
垢版 |
2019/07/17(水) 23:51:55.89ID:xMqK12SR
簡単な問題は誰でも答えられるからな
その分そりゃあとから答えつけるようなのも増えるだろう
何もおかしくなくね?
0278132人目の素数さん
垢版 |
2019/07/18(木) 00:03:08.41ID:Np/EecNs
>>251
>困るんだよね。

どこの誰がなんで困るの?
0279132人目の素数さん
垢版 |
2019/07/18(木) 00:19:31.46ID:7GiM+Qtb
n^5 は 10進表示で {log(n^5)/log(10) + 1} 桁。
1 ≦ n < 9{log(n^5)/log(10) + 1},    >>273
1 ≦ n < 98.7552
n = 1, 28, 35, 36, 46
0281132人目の素数さん
垢版 |
2019/07/18(木) 00:43:43.13ID:aQQIH6oS
>>264
計算をしたいというと語弊がありそうですが、デカルト座標系から座標系Qへの時間に依存した変換Txが分かっているとき、速度をQにおける座標qとその時間微分の値qドットと時間tで表す関数は右下の式で表されるものに限られるか、というようなことを証明したいです
次元が変というのは具体的にどのあたりでしょうか?
0282132人目の素数さん
垢版 |
2019/07/18(木) 01:21:53.83ID:pE9fuMtQ
>>281
最後のところですよ
内積とってるんだがなんだかわかりませんけど、何やってるんですか?
0283132人目の素数さん
垢版 |
2019/07/18(木) 01:49:09.00ID:aQQIH6oS
>>282
N×(N+1)型行列Tx'(q, t)とN+1次元列ベクトル(qドット, 1)の行列積です
見辛くてすみません
0285132人目の素数さん
垢版 |
2019/07/18(木) 02:24:45.80ID:aQQIH6oS
>>284
各x∈R^(N+1)について
Tx(x+h)=Tx(x)+Tx'(x)h+o(|h|) (h→0)
を満たすようなR^(N+1)→M(N, N+1: R)の写像です
0286132人目の素数さん
垢版 |
2019/07/18(木) 02:49:23.70ID:6BLGk1+I
>>279
2行目から3行目の変形がわからないんですが、どうして具体的な数字に持っていけるのですか?
0287132人目の素数さん
垢版 |
2019/07/18(木) 07:37:02.30ID:tZ0x7/gJ
Fを標数p>0の素体とする時
F(x,y)/F(x^p,y^p)の中間体が無数にあることはどのように示せますか?
0288132人目の素数さん
垢版 |
2019/07/18(木) 10:15:00.03ID:g/4TLPu6
>>287
例えばz[k]=x+y^kとおいて相異なるk lをとるときz[l]がF(x^p, y^p, z[k]) に含まれないことを示しておく。
具体的には、もしそうでないとすると整式P(U,V,W)でWについての次数がp未満で

z[l] = P(x^p, y^p, z[k])

となるものがとれるが係数比較で矛盾。
とくにF(x^p, y^p, z[k])の全体は全て相異なる。
とか。
係数比較のくだり精査してないからダメかも。
だったらゴメン。
0289132人目の素数さん
垢版 |
2019/07/18(木) 10:40:24.42ID:g/4TLPu6
>>288
ちょっと修正。
そうでないとすると0でない整式P(U, V, W)とQ(U,V)で

z[l] Q(x^p, y^p) = P(x^p, y^p, z[k])

が存在して以下同文。
0291とある私大の苦学生
垢版 |
2019/07/18(木) 18:39:31.28ID:3FJHVhmz
【至急 超難問】
数理経済、その中でもフランス現代思想の数理表現の問題です。
問「欲動を多様体、構造を構造群で表現した時、商空間が表現するものは、欲動を構造で分類した仮象である」
これを論証せよ。

どなたか分かりますか?
とある私大の経済系の授業で出題され、理解できた学生には大幅加点というシステムなのですが誰も分からず、yahoo知恵袋でも撃沈し、一縷の望みをかけ皆さんを頼らせて頂こうかと…
宜しくお願い致します。
0292132人目の素数さん
垢版 |
2019/07/18(木) 20:09:04.66ID:wo1NL/kl
割り込みすいません。
40人のクラス全員の誕生日が異なる場合の数は、
365C40
でだめなんですか?
A君の誕生日を確定させる必要はあるんですか?
第一学習社の教科書に疑問を持ちました。
0293132人目の素数さん
垢版 |
2019/07/18(木) 20:53:39.73ID:FZiHOK3l
>>292
迷ったら人数を2人とか3人に絞って考えるといいです。
A, Bの2人で考えると
Aが取りうる日付は 365 通りあり、その其々に対して Bは Aとは異なるつまり 364の選択肢がある。
よって、365*364 ( = 365P2 ) 通り。

これを 365C2 ( = 365P2 / 2! ) で考えるという事は、
例えば {A: 4/1, B: 12/31} と {A: 12/31, B: 4/1 } のパターンを区別しないという事です。
特に記載がなければ、普通は区別すると思います。
0294132人目の素数さん
垢版 |
2019/07/18(木) 21:54:24.39ID:SCjSnaz8
10個の玉と3個の箱がある、どの箱にも必ず1個以上の玉が入る、という設定の問題で質問です
玉に区別がなく箱に区別がある場合10個の〇の間に2個の|を入れる場合と同じで9C2=36通り
玉に区別がなく箱にも区別がない場合(118)(127)(136)(145)(226)(235)(244)(334)で8通り
が答えです
しかし、これでは区別がある場合から区別をなくして3!で割ったもの(=6通り)と一致しません
なぜでしょうか
0297132人目の素数さん
垢版 |
2019/07/18(木) 23:19:33.26ID:1BlmoUiy
円Cに内接する四角形PQRSは、PQ=a、QR=b、RS=c、SP=dであり、対角線PRの長さはe、QSの長さはf である。
Cの半径をa,b,c,d,e,fのうち必要なものを用いて表せ。
0298132人目の素数さん
垢版 |
2019/07/18(木) 23:23:03.71ID:1BlmoUiy
αを複素数の定数、sを実数の定数とする。複素数zについての方程式
z(z-2α)+α(αz-z')=sz
を解け。
0299132人目の素数さん
垢版 |
2019/07/18(木) 23:24:49.45ID:d3yhI1hr
>>294
÷6で答え出すなら
n(aの箱=bの箱)=4
n(aの箱=cの箱)=4
n(bの箱=cの箱)=4
なので
(36+4+4+4)÷6=8
0301とある私学の苦学生
垢版 |
2019/07/19(金) 01:47:53.43ID:iXiiR0Fo
再掲すみません。どなたか分かりませんか??

【至急 超難問】
数理経済、その中でもフランス現代思想の数理表現の問題です。
問「欲動を多様体、構造を構造群で表現した時、商空間が表現するものは、欲動を構造で分類した仮象である」
これを論証せよ。

どなたか分かりますか?
とある私大の経済系の授業で出題され、理解できた学生には大幅加点というシステムなのですが誰も分からず、yahoo知恵袋でも撃沈し、一縷の望みをかけ皆さんを頼らせて頂こうかと…
宜しくお願い致します。
0302132人目の素数さん
垢版 |
2019/07/19(金) 02:17:01.03ID:zSIlgHcB
物理学部2年の者です。複素関数論の問題です。

z∈ℂ\[0,1]に対し、f(z) = ∫₀¹ dt/(t-z)とする。
(1)f(z)の連続性を示せ。
(2)次の極限を求めよ。ただし、s∈[0,1]
(a)lim[ε↓0]f(s+iε) (b)lim[ε↓0]f(s-iε)

よろしくお願いします。
0303132人目の素数さん
垢版 |
2019/07/19(金) 03:43:58.12ID:INZ0NY63
>>301

これは困りましたね。
論証だから自由に考えを述べればいいんじゃないかな。

人間の脳の根本は情動(欲動+愛情)で動きます。
脳に解剖学的な座標機能領野に関数と座標を導入します。

商空間は、脳機能探索の期待的表現である。


これは、かって日本最高の能楽者である松本元が刺激を与えた構想です。
彼の死とともに忘れられようとしていますが、根源的な情報処理学者は
これを追求しています。

その道の専門家「いるとしたら、理研脳センターあたりかな」に聞いてみるといいです。
0305132人目の素数さん
垢版 |
2019/07/19(金) 09:02:35.26ID:9ZwOSAvv
>>301
商空間を構成する類の一つひとつは作用群の軌道である、ナンチャッテ数学の言葉遊びか?
0306132人目の素数さん
垢版 |
2019/07/19(金) 09:06:10.46ID:6PPcn4XY
さすがに釣りだろ、本気にしてこれだから文系はwとかやらかさないほうがいいよ。
0307132人目の素数さん
垢版 |
2019/07/19(金) 13:12:28.40ID:7CIxbz6/
普通のオーソドックスな新古典派近代経済学でも無差別曲線という名の同値類割り商空間でミクロ経済学の議論するんだけどね。
0308132人目の素数さん
垢版 |
2019/07/19(金) 13:20:08.27ID:7CIxbz6/
人間も含めた動植物、生き物の生存と繁殖を目指した欲動も
ミクロ経済学も
ゲーム理論で一般に議論できる。

ナッシュ均衡がESS進化的安定戦略と同じものだと発見されたのがそういうことだからね。
利己的な遺伝子とおカネ絡みのミームの違いはあれど。


この手の議論は第二次大戦後の行動科学という分野の隆盛に対応するのが
戦後京都学派ノイマン氏近似
じゃないや
戦後京都学派の今西錦司以降の棲み分け≒ニッチ生態学的地位的な動物行動学だろうね。

おフランス哲学じゃなくて。
0309132人目の素数さん
垢版 |
2019/07/19(金) 13:25:30.89ID:7CIxbz6/
「選好」として区別しない無差別だというのが一般には高次元でも議論できる無差別曲線だから。

あんまり専攻内容として食わず嫌いが少ない方だと自分自身自覚してるがおフランス現代思想の先公は感心せんな。自分が分かってない分野を学生のテストに出すなよ。
0310イナ ◆/7jUdUKiSM
垢版 |
2019/07/19(金) 13:54:33.08ID:xhNaDqxt
>>297Cの半径をrとすると、正弦定理より、
2rsin∠PQR=e
r=e/2sin∠PQR――@
余弦定理より、
cos∠PQR=(a^2+b^2-e^2)/2ab
sin∠PQR=√{1-(a^2+b^2-e^2)^2/4a^2・b^2}
@に代入し、
r=e/2√{1-(a^2+b^2-e^2)^2/4a^2・b^2}
=abe/2√{a^2・b^2-(a^2+b^2-e^2)^2/4}
=abe/√[4a^2・b^2-{(a^2+b^2)^2-2(a^2+b^2)e^2+e^4}]
=abe/√[4a^2・b^2-{a^4+2a^2・b^2+b^4-2(a^2+b^2)e^2+e^4}]
=abe/√(2a^2・b^2-a^4-b^4+2a^2・e^2+2b^2・e^2-e^4)=abe/√(2a^2・b^2+2a^2・e^2+2b^2・e^2-a^4-b^4-e^4)
 ̄]/\_____________
__/\/      /|
 ̄\/      / |
 ̄|\__________/| |__
]| ‖ ̄ ̄ ̄ ̄‖ | / /
_| ‖ □ □ ‖ |/ /
______________‖/_/
 ̄ ̄ ̄ ̄ ̄ ̄ ̄ ̄ ̄‖
]  □   □  ‖ /
_____∩∩_________‖/
 ̄⊂(-.-))⌒ つ~ ̄ ̄
.   `υ __                                  前>>207
0311イナ ◆/7jUdUKiSM
垢版 |
2019/07/19(金) 14:07:00.54ID:xhNaDqxt
>>310追加、訂正。>>300たしかにこうなる。
Cの半径をrとすると、正弦定理より、
2rsin∠PQR=e
r=e/2sin∠PQR――@
余弦定理より、
cos∠PQR=(a^2+b^2-e^2)/2ab
sin∠PQR=√{1-(a^2+b^2-e^2)^2/4a^2・b^2}
@に代入し、
r=e/2√{1-(a^2+b^2-e^2)^2/4a^2・b^2}
=abe/2√{a^2・b^2-(a^2+b^2-e^2)^2/4}
=abe/√[4a^2・b^2-{(a^2+b^2)^2-2(a^2+b^2)e^2+e^4}]
=abe/√[4a^2・b^2-{a^4+2a^2・b^2+b^4-2(a^2+b^2)e^2+e^4}]
=abe/√(2a^2・b^2-a^4-b^4+2a^2・e^2+2b^2・e^2-e^4)
=abe/√(2a^2・b^2+2a^2・e^2+2b^2・e^2-a^4-b^4-e^4)
=abe/√(a+b+e)(-a+b+e)(a-b+e)(a+b-e)
 ̄]/\_____________
__/\/      /|
 ̄\/      / |
 ̄|\__________/| |__
]| ‖ ̄ ̄ ̄ ̄‖ | / /
__| ‖ □ □ ‖ |/ /
____‖________‖/_/
 ̄ ̄ ̄ ̄ ̄ ̄ ̄ ̄ ̄‖
]  □   □  ‖ /
_____∩∩_________‖/
 ̄⊂(-.-))⌒ つ~ ̄ ̄
.   `υ __
0312132人目の素数さん
垢版 |
2019/07/19(金) 15:13:36.14ID:25THqKse
>>300
ヘロンの公式に似ていますが、導出過程で使うのでしょうか
私はこの問題が解けませんでした。三角比で計算していたので、原理的にはヘロンの公式と同等のことをやっているはずなのですが
0313132人目の素数さん
垢版 |
2019/07/19(金) 15:45:41.48ID:Le/JpqUM
エレガントな解答にこだわらないなら三辺わかってる三角形四つもあるんだからどれ使ってもできんじゃないの?
R=(1/2) 辺×sin 対角 で余弦定理で cos 対角も出せるし。
0316132人目の素数さん
垢版 |
2019/07/19(金) 22:59:04.11ID:GdkLODBg
すいません、

fを多項式とする

f(y)-f(x)は(y-x)で割り切れることを因数定理より示せ。

これが分かりません
バカですみませんがよろしくお願いします
因数定理は理解してますが使いこなせません
0317132人目の素数さん
垢版 |
2019/07/19(金) 23:00:22.93ID:GdkLODBg
全項書き出して同じ係数を持つ同じべきの各項を引いてくくれば、全てy-xでくくれて割り切れる、のはわかりますが、因数定理との結びつきがわかりません…
0318132人目の素数さん
垢版 |
2019/07/19(金) 23:08:49.83ID:CTsH6wd0
yを定数としてfy-fxをxについての式だと見る
fy-fxのxにyを代入して0になるのが明らか、だからx-yを因数に持つ
0319132人目の素数さん
垢版 |
2019/07/19(金) 23:09:40.77ID:UT6R2apf
xを定数と思ってg (y) = f (y) - f (x) とおけ
0320132人目の素数さん
垢版 |
2019/07/19(金) 23:19:33.81ID:fPHjTPm9
隣り合う(= ad-bc=1が成り立つ)2分数a/bとc/d(文字は全て自然数)の間にある分数の中で分母が最小であるのは(a+c)/(b+d)であることを証明せよ。

お願いします
0322132人目の素数さん
垢版 |
2019/07/20(土) 00:25:28.01ID:JIxksdVK
>>298
自己解決しませんでした。

α=0 のときは z(z-s)=0, z=0,s

α≠0 のとき、与式から
 (z -2α + αα-s)z = αz'
ここで
 z = r・e^(iθ)   (r,θは実数)
とおくと
 z = 2α -αα + s + α・exp(-2iθ)
さて、どうするか・・・・・
0323132人目の素数さん
垢版 |
2019/07/20(土) 02:15:53.15ID:spYMNvSr
1次と2次のキュムラントがE[X]とE[(X-μ)^2]に一致するのはなぜなのでしょうか
0324132人目の素数さん
垢版 |
2019/07/20(土) 05:33:26.78ID:JIxksdVK
xを定数だと思って f(y) - f(x) を y-x で割る。
 f(y) - f(x) = (y-x)Q(x,y) + R(x),
ここで yにxを代入すれば
 R(x) = 0,
0325132人目の素数さん
垢版 |
2019/07/20(土) 06:12:43.63ID:xFzgemcg
>>298
z′はzの共役複素数ですか?
それならば与式の共役な式
z′(z′-2α′)+α′(α′z′-z)=sz′
に与式をz′=の形に変形したものを代入すると力業でおそらく解けます
0327132人目の素数さん
垢版 |
2019/07/20(土) 10:57:05.91ID:bSAoQnjE
5715
ふうL@Fu_L12345654321
学コン1傑いただきました!
とても嬉しいです!

https://pbs.twimg.com/media/D-IuUuqVUAALnAB.jpg
https://twitter.com/Fu_L12345654321/status/1144528199654633477
https://twitter.com/5chan_nel (5ch newer account)
0328132人目の素数さん
垢版 |
2019/07/20(土) 11:04:33.45ID:Ki6s0Uy9
>>312
ヘロンの公式と似ているのは、余弦定理、正弦定理、
sin^2+cos^2=1を使って色々するからかな。
答えをヘロンの公式みたいにa+b+e=Sとかと置いて表すと、
少し簡単な形にはできるね。

>>297の問題のポイントは、
結局、どの長さが独立して決まって、
どこからが外接円の半径に影響しない長さになるのかに気づくことだろう。

円上にある3点が決まってしまえば、円は一意に決まる。
それは、例えば、△PQRが決まるということと等しい。
△PQRが決まれば、頂点Sの場所は自由に決められない。
つまり、△PQRの外接円の円周上のどこかにSを取らないといけなくなり、
どこに取ったとしても、外接円の半径は不変である。

だから、答えは、(例えば、)a、b、eだけで表せないとおかしいということに気づく。

この問題は要するに、「円に内接する△ABCの辺の長さをa、b、cとしたとき、
外接円の半径をa、b、cで表せ。」
という問題と同じことをすればよいと気づけば、だいぶ話は簡単になる。

正弦定理と余弦定理(とsin^2+cos^2=1)を組み合わせて、
外接円の半径を辺の長さで表せばよい。
0329132人目の素数さん
垢版 |
2019/07/20(土) 11:12:18.23ID:DjHCCM3E
>>323
log( E[exp(tX)] )
= log( 1 + tE[X] + tt/2! E[XX] + ... )
= (tE[X] + tt/2! E[XX]) - 1/2 (tE[X] + tt/2! E[XX])^2 + ...
= tE[X] + tt/2! E[XX] - 1/2 tt (E[X])^2 + ...
= tE[X] + tt/2! E[(X-μ)^2] + ...

∵ E[(X-μ)^2] = E[XX -2Xμ + μμ] = E[XX] - 2μμ + μμ = E[XX] - E[X]^2
0330132人目の素数さん
垢版 |
2019/07/20(土) 11:39:48.66ID:f3Pyo0YZ
受験用語だと思いますが「解けない漸化式」というのがあります。例えば、
a[1]=2
a[n+1]=na[n]/(n+a[1]+...+a[n])
のように一般項が初等的に表せないもののことだと思うのですが、このような数列で、極限が初等的な形で求まるかどうか判別する方法はありますか?
0331132人目の素数さん
垢版 |
2019/07/20(土) 11:58:45.27ID:DjHCCM3E
0 < a[n+1]=na[n]/(n+a[1]+...+a[n]) =a[n]/{ 1+(a[1]+...+a[n])/n } < a[n]
下に有界な減少数列なので収束する。 収束値 をα とすると
→ α = α /(1 + α) より ∴ α = 0 ( (a[1]+...+a[n])/n → α を使った )

一般的な判別方法なんて無い。
手持ち道具 (例では下に有界な減少数列、平均の収束) を使って出来るだけの事をするしかない。
他にも収束値αが存在すると仮定して矛盾を引き出せれば、発散or 振動が言える。
0332132人目の素数さん
垢版 |
2019/07/20(土) 12:10:03.49ID:xnRD7Bja
>>315
z=a+ibと置いて実部と虚部に分けてそれぞれ連続であることを示しました、あんまり綺麗なやり方ではないと思うのですが…
原始関数であるLogを使った解法も考えたのですが、僕は上手くいかなかったです…
0333132人目の素数さん
垢版 |
2019/07/20(土) 12:10:50.22ID:ixWiuyhi
分野としては差分ガロア理論で、定理としては漸化式の解を添加した差分体がリウヴィル拡大になるかどうかかな
(確か差分方程式の場合は解を添加すると体どころか整域ですらなくなる場合があるから差分体では不十分かも)

ただし具体的にガロア群(代数群)を計算するのは線形のときですらそれ自体難しい問題だし、非線形なんてまだまだ発展途上の段階
0334132人目の素数さん
垢版 |
2019/07/20(土) 13:20:49.91ID:DjHCCM3E
>>332
f(z+h) - f(z) = ∫₀¹ dt 1/(t-z-h) - 1/(t-z) = ∫₀¹ dt h/{(t-z-h)(t-z)}

L := inf { |z-t| | t ∈ [0,1] } ( = distance{ z, [0,1] } ) と置き
任意のε (正数) に対して, | h| < min( L/2, ε L.L/2 ) となるように h を選べば
・| t-z-h | = |(z-t) +h| ≧ |z-t| - |h| ≧ L - L/2 = L/2
・|f(z+h) - f(z)| ≦ ∫₀¹ dt |h|/|(t-z-h)(t-z)| ≦ ∫₀¹ dt |h|/|(L/2).L| = |h|/|(L/2).L| < ε
よって f(z) は連続である.

∫₀¹ dt 1/(t-z) = log(1 -z) - log(0-z) = log|1 -z| + i arg (1-z) - log|-z| - i arg(-z)
偏角arg(...) の選び方に注意すれば後半は簡単
0335132人目の素数さん
垢版 |
2019/07/20(土) 17:42:05.61ID:xnRD7Bja
>>334
すごい、これεδ論法で解けたんですね……ありがとうございます!

なんとなくmin{L/2, εL.L/2}を考えるところが連続関数の商の連続性の証明に似てる気がします……
0336132人目の素数さん
垢版 |
2019/07/20(土) 19:44:40.78ID:JIxksdVK
 αz'/z = z -2α + αα -s,
その複素共役は
 α'z/z' = z' -2α' + α'α' -s,
辺々掛けて平方根すると
 |α| = | z -2α + αα -s|
だから
 z -2α + αα -s = α e^(ib),
さて、どうするか・・・・
0337132人目の素数さん
垢版 |
2019/07/20(土) 21:57:59.67ID:4wdnwaiD
>>321
ファレイ数列で暫く色んなサイト見たんですけど行列式とか使っていてよく理解できないです
与えられた式から変形などで証明はできないのでしょうか?
0338132人目の素数さん
垢版 |
2019/07/20(土) 22:06:35.24ID:kDpRSoqm
>>337
大学への数学の増刊の新数学演習に載ってたはず
0339132人目の素数さん
垢版 |
2019/07/20(土) 22:12:37.81ID:cYiEK9Mx
>>337
字面だけ追ってけば証明は理解できるだろうけど、それで終わりではダメだろ?
キチンと(a,b), (c,d)と原点を3頂点に含む平行四辺形の周、内部に格子点が4個しかないこと、それがどのように主張につながっていくか理解しないと。
行列は数学続けるなら必須なんだから勉強しとけよ。
0340132人目の素数さん
垢版 |
2019/07/20(土) 23:17:33.39ID:4wdnwaiD
>>339
確かにそれも大切だと思います。
しかしまだ高校生なので今の知識と実力で理解できる解法では解けないのかな、と思って質問した次第です。

平行四辺形を利用する方法では自分では恐らく少し改題されただけで解けなくなりそうなので式変形などから解ける方法を探しています。
0341132人目の素数さん
垢版 |
2019/07/21(日) 00:21:41.58ID:S+Wb3s17
間にある分数を p/q とおく
分母を払って差をとってそれらを s,t とおく
その等式を p,q について解く
0343132人目の素数さん
垢版 |
2019/07/21(日) 04:01:41.53ID:4k/Gtesi
〔類題〕
円Cに内接する四角形PQRSは、PQ=a、QR=b、RS=c、SP=dであるが、内部は立入禁止である。
Cの半径を a, b, c, d を用いて表せ。
0344132人目の素数さん
垢版 |
2019/07/21(日) 04:08:17.45ID:4k/Gtesi
答えは
 半径 = √{(ab+cd)(ac+bd)(ad+bc)/(-a+b+c+d)(a-b+c+d)(a+b-c+d)(a+b+c-d)}
らしい。
なお、対角線の積は ef = ac+bd (トレミーの定理)
0345132人目の素数さん
垢版 |
2019/07/21(日) 10:42:13.72ID:eju/Jy4X
>>341
c/d<p/q<a/bとする(但しad-bc=1)
p/q-c/d=(dp-cq)/dq>0
a/b-p/q=(aq-bp)/bq>0

dp-cq=s,aq-bp=tとおく
2式を連立して解くと
q=bs+dt
代入して
p=as+ct
よって
p/q=(as+ct)/(bs+dt)
各数は自然数なので明らかにs=t=1で最小でありそのとき
p/q=(a+c)/(b+d)

合ってますか?
0347132人目の素数さん
垢版 |
2019/07/21(日) 11:59:56.84ID:eju/Jy4X
>>346
p/q=(a+c)/(b+d) の分子分母の最大公約数をgとおくと
互いに素な自然数mnを用いて
a+c=gm,b+d=gnとおける
このときp/q=m/nとなりp=m,q=n
b+dが最小となるからn=b+dであり、g=1
したがって既約
0350132人目の素数さん
垢版 |
2019/07/21(日) 17:23:48.93ID:o+kv6UFz
>>347
p=a+c, q=b+d のとき既約を証明してもダメなのでは?
p/q はそれ以外のc/dとa/bの間の元なんだから。
つまりp=as+ctとq=bs+dtのGCDが1を言わないと。
一般にはad-bc=1から行列[[a,b],[c,d]]の定める一次変換がZ係数で可逆なのを利用して
gcd(p,q)=gcd(1,1)=1
といくもんだけど行列使いたくないとか贅沢なこと言ってるからな。
もちろん行列使えば一発で済む事をグチャグチャ初等的な変形だけでも示せるだろうけど、そんな事してなんの役に立つものやら。
0351132人目の素数さん
垢版 |
2019/07/21(日) 18:42:20.48ID:4k/Gtesi
>>330

 a[1] = 2,
 a[2] = 2/3,
 a[3] = 2/7,
 a[4] = 18/125,
より
 a[1] + a[2] + a[3] + a[4] = 3 + 253/8128 > 3,
n≧4 のとき
 a[n+1] < a[n] * n/(n+3) < a[n] * {(n+2)/(n+3)}^3,      ・・・・ (*)
 a[n+1](n+3)^3 < a[n] (n+2)^3 < ・・・・ < a[4] (4+2)^3 = 31.104
 ∴ a[n] < 31.104/(n+2)^3,
 ∴ Σ a[k] は収束する。(α=0)
 s = a[1] + a[2] + ・・・・ = 3.35753325

n >>1 では
 a[n] ≒ 37.9/n^(s-1),
 a[1] + a[2] + ・・・・ + a[n] ≒ s - 16.1/n^s,

* {1,1,n/(n+3)} で GM-AM すると n/(n+3) < {(n+2)/(n+3)}^3,
0352118
垢版 |
2019/07/21(日) 20:01:51.47ID:ZGolyGNp
長くては困るということでより実用的な機能を追加しておいた
https://pastebin.com/hzdfNr6t
https://i.imgur.com/maFfQwQ.jpg

引数無しなら前回といっしょ
1つの引数で、それが素数か判定。素数ならそれを返す。倍数なら返さない。
2つの引数で、その2つの区間の素数を書きだす。1万以下だけ必要でもコードに手を入れる必要はない。

数の上限は 大体上のjpg画像あたりが限界だ
ちなみに引数無しでのtxt 出力は5GBで、7zipすると500MB程度に収まる
0353132人目の素数さん
垢版 |
2019/07/21(日) 20:52:27.04ID:/lozKTlV
>>352
俺は PARI/GP って数式ソフトを電卓代わりに使ってる。これオススメだよ。
かなり軽量だし数論関係のコマンドも充実してる。
ちょっとしたプログラムも書けるからそういう欲求にも答えてくれると思う。
0354132人目の素数さん
垢版 |
2019/07/21(日) 21:15:03.04ID:ZGolyGNp
>>353
thanks
GPL のマルチプラットフォーム言語か

個人的には電卓なら Common Lisp が気にいっている
---
しょせん素数ジェネレータはおみやげだ
昔の自分は大きな素数が書いてある素数表を貴重だと思って買ったことがある
しかしこれがあれば広辞苑のようなぶ厚い素数表が出版されたとしても、
買う必要がない
純粋なおみやげなんだ
0355132人目の素数さん
垢版 |
2019/07/21(日) 21:19:08.61ID:/lozKTlV
PARI/GP 、例えば...
? for(k=0,2000, n=10^100+k; if( isprime(n), printf("prime! 10^100 + %d\n",k)))
prime! 10^100 + 267
prime! 10^100 + 949
prime! 10^100 + 1243
prime! 10^100 + 1293
prime! 10^100 + 1983

こういうアホな思いつきでもすぐ実行できるのが楽しい
しかもメモリが許す限りの任意桁長の計算が可能
0356132人目の素数さん
垢版 |
2019/07/21(日) 21:40:51.32ID:ZGolyGNp
>>355
その大きな数で実用的だとすると
計算済みの素数記録してキャッシュにしてるかんじだな
(素数定理によると約230倍の速度で計算しているはず)
メモリ消費は心配だけど、非常に便利だね
0359132人目の素数さん
垢版 |
2019/07/22(月) 00:29:31.98ID:/v5meAwp
優収束定理使いたいんですけど|n*sin(x/n)*{x*(x+1)}^(-1)|を上から抑える関数の例って何かありますか
0361132人目の素数さん
垢版 |
2019/07/22(月) 01:59:18.17ID:vDQA99OD
>>357
上限なら・・・・

・nが偶数の場合
 k = n/2 のとき最大。
 C(n,n/2±j) = C(n,n/2) n(n-2)・・・・(n-2j+2)/{(n+2)(n+4)・・・・(n+2j)}
 < C(n,n/2) {n/(n+2)}^(j^2)      ・・・・ (*)
 = C(n,n/2) x^(j^2),
 (与式) < C(n,n/2)^m・θ_3(0,x^m)
 ここに x = n/(n+2),
 (*) (n-2j+2)/(n+2j) ≦ {n/(n+2)}^(2j-1) = x^(2j-1),

・nが奇数の場合
 k = (n-1)/2, k = (n+1)/2 のとき最大。
 C(n,(n+1)/2+j) = C(n,(n-1)/2-j)
 = C(n,(n+1)/2) (n-1)(n-3)・・・・(n+1-2j)/{(n+3)(n+5)・・・・(n+1+2j)}
 < C(n,(n+1)/2) {(n-1)/(n+3)}^{j(j+1)/2}  ・・・・ (**)
 = C(n,(n+1)/2) y^{j(j+1)},
 (与式) < C(n,(n+1)/2)^m・(y^m)^(-1/4)・θ_2(0,y^m)
 ここに y = √{(n-1)/(n+3)},
 (**) (n+1-2j)/(n+1+2j) ≦ {(n-1)/(n+3)}^j = y^(2j),

 θ_2(0,z) = Σ[j=-∞,∞] z^{(j+1/2)^2}
 θ_3(0,z) = Σ[j=-∞,∞] z^(j^2)
は楕円テータ函数
0363132人目の素数さん
垢版 |
2019/07/22(月) 03:08:23.74ID:Eipdk+6R
添字演算についてです。vを3次元ベクトル
w=rot(v)とするとき
(|w|)^2を添字演算によって求めてください。

(w_i)(w_i)=(ε_ijk)(ε_ijk)(∂v_k/∂x_j)(∂v_k/∂x_j)
から進めません。
0364132人目の素数さん
垢版 |
2019/07/22(月) 10:00:31.50ID:TmHe99r/
>>363
・(ε_ijk)(ε_ij’k’) = δ_jj’δ_kk’ - δ_jk’δ_kj’
↑この公式を覚えておくと
ベクトル解析の公式で面倒なやつの大半が一瞬で導出できるようになります。
(自力で「発見」した時はどうして教えてくれなかった!と思いました)

|w|^2 = (w_i)(w_i)=(ε_ijk)(ε_ij’k’)(∂v_k/∂x_j)(∂v_k’/∂x_j’)
= (δ_jj’δ_kk’ - δ_jk’δ_kj’)(∂v_k/∂x_j)(∂v_k’/∂x_j’)
= (∂v_k/∂x_j)(∂v_k/∂x_j) - (∂v_k/∂x_j)(∂v_j/∂x_k)
= tr( M(M^t - M ) ) (行列: M_ij = ∂v_j/∂x_i と置きました)
最後の変形は必要なのか分からないけど添え字を消したいならこんな感じという事です
0367132人目の素数さん
垢版 |
2019/07/22(月) 10:59:56.00ID:/QcrS9WQ
>>365
調べたらisprime関数は1000桁の判定に15分〜30分かかる、と書いてあったのを見つけた。

WolframAlpha先生に試しにisprime 10^1000+453を聞いたらすぐに素数と返してきたけど、これはどうやって判定してるのかねえ。
0369132人目の素数さん
垢版 |
2019/07/22(月) 12:30:18.88ID:TmHe99r/
上の方で PARI/GP を推しましたが、今は pythonで何でもできますね。 そういう時代なんですね。
bpython 等のREPL(対話式インターフェース) で、電卓代わりに使えますし、plotも綺麗ですし。
PARI/GPの利点は「ほん少しだけ起動が早い」「追加パッケージのinstall&import が不要」くらいでしょうか。

import time, gmpy2 # gmpy2 はググって見つけた数論パッケージ
t=time.time()
li= list( filter( lambda n:gmpy2.is_prime(n), [10**1000+k for k in range(2000) ]) )
t=time.time()-t
print(f" length: {len(li)}\n time: {t*(10*3):7.2f} msec\n" )
--->
length: 2 {10^1000+0...10^1000+1999 の範囲に素数は2つだけ}
time: 105.01 msec

PARI/GP で同じように 1000桁バージョンやらせたら数分待っても応答無しなので諦めました。
大半の数が直ぐに素数判定できるんですが、判定が苦手な数がポツポツ混じっていました。
isprime(10^1000 + 453) \\ 例えばコレとか... Wolfram先生や python のはアルゴリズムが異なるようですね。
0370132人目の素数さん
垢版 |
2019/07/22(月) 12:33:51.78ID:TmHe99r/
例えば PARI/GP の場合
? isprime(10^1000 + 453)
*** isprime: Warning: increasing stack size to 16000000.
*** isprime: Warning: increasing stack size to 32000000.
*** isprime: Warning: increasing stack size to 64000000.
{これ明らかに途中でキャッシュを積み増ししてるんですが、その後応答無し}

Wolfram先生は キャッシュ利用以外にも何かやってると思います。
0371132人目の素数さん
垢版 |
2019/07/22(月) 13:24:17.37ID:q9pttVSt
>>369
そのPythonはあまりに速すぎるけどやっぱりキャッシュなのかな?

|length: 2 {10^1000+0...10^1000+1999 の範囲に素数は2つだけ}
もうひとつは10^1000+1357
これもWolframAlphaは即座に判定する
0372132人目の素数さん
垢版 |
2019/07/22(月) 13:51:20.70ID:tQja156F
>>371
wolframengine インストールしてやってみたが
異常な早さだね
オフラインにしても変らないから鯖のデータベースに問いあわせてすらない

2からsqrt prime までの素数で試しに割っていく
ような速度じゃない
やはりハッシュ、素数をキーとしてそれが素数か否かを計算済みのハッシュを
調べているような速度だ
ファイル化のために大きなn進数を使っているのか、独自バイナリを使っているのか
目星はつかないがそういうことだと思う 早すぎる
0373132人目の素数さん
垢版 |
2019/07/22(月) 14:05:50.19ID:TmHe99r/
>>371
結局、その二つが素数なんですね。素数じゃない事の判定は一瞬で。
やはり根本的なアルゴリズムが違うのでしょう

> あまりに速すぎる
ミリ秒表示で t*(10**3) とするべき箇所が t*(10*3) (= t*30 て...) になってました。

import time
import gmpy2 as gm
t=time.time(); li= list( filter( lambda n: gm.is_prime(n), [10**1000+k for k in range(2000)])); t=time.time()-t
print(f" length: {len(li)}\n time: {t*(10**3):7.2f} msec\n" )
--->
length: 2
time: 3529.49 msec {それでも速い}

import time
import sympy.ntheory.primetest as sm # gmpy2 よりも sympyパッケージの方が有名 (たぶん)
t=time.time(); li= list( filter( lambda n: sm.isprime(n), [10**1000+k for k in range(2000)])); t=time.time()-t
print(f" length: {len(li)}\n time: {t*(10**3):7.2f} msec\n" )
--->
length: 2
time: 5270.99 msec {ちょっとだけ遅い。アルゴリズムが異なるのか微妙な実装テクニックの差なのかは不明}
0374132人目の素数さん
垢版 |
2019/07/22(月) 14:23:39.36ID:tQja156F
>>373
素数じゃないよ 倍数だよ
という判定なら
6k +-1 の形をしていない とか
ある程度大きい時、1桁目が1,3,7,9では無いとか
の判定混ぜれば早くできる

数が大きいと素数じゃないほうが大きいので、
そういう早期に倍数判定下せる方法盛るほうが
早さを出せるはず
0375132人目の素数さん
垢版 |
2019/07/22(月) 14:33:22.53ID:TmHe99r/
PARI/GP は数論関係のソフトウェアとしては老舗なので、そういうのやってないはずがないと思うんですけどねえ。
0376132人目の素数さん
垢版 |
2019/07/22(月) 15:03:40.16ID:AIiGTjIo
群論の問題ですがお願いします
Gを有限群、HをGの部分群、G≠H、(G:H)=k、Gはk!で割り切れないとする
このとき、HはGの単位元のみでない正規部分群を部分群として持つことを示せ
0378132人目の素数さん
垢版 |
2019/07/22(月) 16:11:53.58ID:tQja156F
wolfram engine は
In をキーとして Out をハッシュに入れているね
複雑な積分をIn に入れて
2回目からは一瞬でOut 出してきた
0379132人目の素数さん
垢版 |
2019/07/22(月) 16:43:23.78ID:tQja156F
https://www.wolfram.com/technologies/nb/index.ja.html?footer=lang
>ノートブックをテキストエディタ等で編集すると,無効なキャッシュデータが使われる可能性があります.これを防ぐためには,Wolfram言語ベースの製品以外でノートブックを編集する際に,ファイル上部付近にあるCacheIDを指定した行を必ず削除してください

なんかキャッシュの説明もあるね。調査終了
0381132人目の素数さん
垢版 |
2019/07/22(月) 18:56:30.89ID:KUQ/kL/E
そこかよw
0382132人目の素数さん
垢版 |
2019/07/22(月) 19:14:29.20ID:f0E0TLKw
10進法表記された自然数kの各桁の数を足し合わせた値をf(k)とおく。
例えば、k=3のときf(k)=3、k=25のときf(k)=2+5=7、k=1605のときf(k)=1+6+0+5=12、である。

(1)任意の自然数nに対して、f(n)≤nであることを示せ。また等号が成り立つnを全て求めよ。

(2)10^p≤n<10^(p+1)の範囲で、f(n)とlog(n)の大小を比較せよ。
ここでpは自然数の定数、log(n)は常用対数である。
0383132人目の素数さん
垢版 |
2019/07/22(月) 19:31:36.79ID:F7hJ5hzq
どこまでわかったかかけ
0384132人目の素数さん
垢版 |
2019/07/22(月) 20:20:18.18ID:uKurkYUn
>>383
(1)は1,2,3,4,5,6,7,8,9で、それ以上はn=a[i]*10^i+...と表記すれば容易にn>f(n)でした。
(2)はlog10が「桁数-1」なので111...110のような場合にイコールになることは分かりました。その先の場合分けがよくわかりません。
0385132人目の素数さん
垢版 |
2019/07/22(月) 21:08:31.33ID:rdHYUdOV
>>376
成立せんのじゃね?
Gが位数10の二面体群、Hをシロー2群とすればk=[G:H]=5で|G|=10は5!=120で割り切れないけどHの部分群でGの正規部分群になるのは自明群しかないけど?
0386132人目の素数さん
垢版 |
2019/07/22(月) 21:15:19.87ID:aG7ajwuC
めんどくさい反例
0390132人目の素数さん
垢版 |
2019/07/23(火) 00:23:29.31ID:AiNqRJKL
そこでサブロー群の登場です
0393132人目の素数さん
垢版 |
2019/07/23(火) 14:12:17.49ID:BjFGKdev
>>392
(1) f(θ) = cos4θ - {単調増加関数} だから最大値は直ぐ分かる。
微分して極値をとる点を探せば最小値も分かる。

(2) f(x) = x^3 -3x と y=x のグラフを描けば分かる。
0394132人目の素数さん
垢版 |
2019/07/23(火) 14:19:44.36ID:7unPkcJT
>>393
ちゃんと答えをかけよ偉そうにヒントみたいなの出して助けた気になってるおまえみたいなゴミが一番邪魔
0396132人目の素数さん
垢版 |
2019/07/23(火) 14:30:56.24ID:QY9LiU2l
1問目は微分する必要すらない。
cosの倍角公式を使ってsin^2の項をcos2θで書いて、cos4θの項もcosの倍角使ってcos2θで表示すれば、cos2θについての二次関数になる。
あとはcos2θの値域(-1以上1以下)調べて解けばいいだけ
0397132人目の素数さん
垢版 |
2019/07/23(火) 14:35:39.54ID:QY9LiU2l
2問目はもとは2004年東大理科の第4問だな。
平凡に解くとすらっとは解けない。それほど難しいわけではないが、地道な場合分けが必要。

具体的にfの増減調べてグラフを書いて、aの値ごとの解の個数と解の位置(-2,-1,1,2との大小)を調べて場合分けしていくと普通に解ける。
0398132人目の素数さん
垢版 |
2019/07/23(火) 14:37:54.17ID:QY9LiU2l
背景にあるチェビシェフの多項式を見抜けば一発だが、凡人が実戦的に思いついて得点できる解法ではない。
(x=2cosθとおくと、x^3-3x=2cos3θとなる)
0399132人目の素数さん
垢版 |
2019/07/23(火) 17:01:50.12ID:DZe/zZRp
これお願いします

10進法表記された自然数kの各桁の数を足し合わせた値をf(k)とおく。
例えば、k=3のときf(k)=3、k=25のときf(k)=2+5=7、k=1605のときf(k)=1+6+0+5=12、である。

(1)(解けた、(2)とも独立なので省略)

(2)10^p≤n<10^(p+1)の範囲で、f(n)とlog(n)の大小を比較せよ。
ここでpは自然数の定数、log(n)は常用対数である。
0401132人目の素数さん
垢版 |
2019/07/24(水) 00:09:39.91ID:xInihN6h
>>394
何この本当に役に立たないこと言ってるゴミ
ただただ邪魔だな
0402132人目の素数さん
垢版 |
2019/07/24(水) 00:17:08.18ID:LYIAox3Q
>>394
そしてこういうアホはid替わるとしれっと書き込むんだよな
自分がクズなの自覚してほしいが、クズだからわからない
匿名だからこの手のキチガイには対処できないのが辛いね
0403132人目の素数さん
垢版 |
2019/07/24(水) 03:25:04.70ID:br6alVJ1
>>392
(上)
 f(θ) = cos(4θ) - 4(sinθ)^2 とする。
 0゚≦θ≦90゚ における f(θ) の最大値および最小値を求めよ。

(略解)
 cos(2θ) で表わすと     >>396
f(θ) = {1+cos(4θ)} + 2{1-2(sinθ)^2} -3
  = 2{cos(2θ)}^2 + 2cos(2θ) -3
  = 2{cos(2θ) + 1/2}^2 -7/2

0゚≦2θ≦180゚ ゆえ -1 ≦ cos(2θ) ≦ 1,
 -7/2 ≦ f(θ) ≦ 1,
0404132人目の素数さん
垢版 |
2019/07/24(水) 03:56:59.81ID:br6alVJ1
>>392
(下)
関数 f(x) = x^3 -3x について以下の問いに答えよ。
(1) aを定数とする。f(x)=aを満たす実数xの個数を求めよ。
(2) f(f(x)) = 0 を満たす実数xの個数を求めよ。
(3) f(f(f(x))) = 0 を満たす実数xの個数を求めよ。

(略解)
 f(x) = 2・T_3(x/2),   ・・・・ 第一種チェビシェフ多項式
x = 2cosθ とおくと  >>398
 f(x) = 2cos(3θ)
 f(f(x)) = 2cos(9θ),
 f(f(f(x))) = 2cos(27θ),

(1) |a| < 2 のとき 3個
  |a| = 2 のとき 2個
  |a| > 2 のとき 1個

(2) 9個
(3) 27個
0405132人目の素数さん
垢版 |
2019/07/24(水) 04:43:36.99ID:AiHsJM0D
すみません行列の基本的なことで質問なんですけど2×1行列×2×2行列って計算できませんよね?
0407132人目の素数さん
垢版 |
2019/07/24(水) 08:43:44.42ID:tX09B9nR
この問題が分かりません。
解をα、βとおいてα^nやβ^nを作っても周期性が見えてこないです。

a,bを実数とする。方程式
x^2+ax+b=0
の重複も込めた2解をα、βとし、数列{a_n}を
a_n=α^n+β^n
で定める。このとき{a_n}が周期をもつための、a,bが満たすべき条件を求めよ。
ただし数列{a_n}が周期をもつとは、あるpが存在して、任意のnに対してa_n=a_(n+p)が成り立つことをいう。
0408132人目の素数さん
垢版 |
2019/07/24(水) 10:15:47.61ID:KXlu7sfk
>>407
実係数2次方程式の2解なので α = r*e^{+iθ}, β = r*e^{-iθ} と置く.
・r = |α| = |β| = 1 である.
そうでないと n→∞ で 発散または 0 に収束する.

2 cos(nθ) = e^{+inθ}+ e^{-inθ} = e^{+i(n+p)θ}+ e^{-i(n+p)θ} = 2 cos((n+p)θ) であり,
一方で cos(nθ)=cos(ξ) となるのは ξ = nθ + 2π N または ξ = -nθ + 2π N (Nは整数)
の場合に限られる. つまり pθ = 2π N または (2n+p)θ = 2π N
いずれにしろ
・θ = π Q (Q: 有理数) の形になっている.
逆にこのとき周期性を持つ事は明らか.

解と係数の関係より
a = -(α+β) = - e^{+iπQ} - e^{-iπQ} = 2 cos(πQ)
b = αβ = e^{+iπQ} e^{-iπQ} = 1

( 勝手に α=β=0 はナシとさせてもらったが
一応は x^2 = 0 の2重根である. 周期1として解に含めたければご自由に )
0409132人目の素数さん
垢版 |
2019/07/24(水) 10:19:51.72ID:zpQXtkNE
>>407
そんなもん問題にすらなってない。
強いて言えば

a=-2cos(2π/p)となる自然数pが存在し、b=1

とかだろうけど必要十分条件なんか一意に定まるものではない。
どこのカスが作った問題か知らんけどんな事も分からん奴の作った問題なんかほっとけ。
0410132人目の素数さん
垢版 |
2019/07/24(水) 11:14:31.78ID:lkvm9SmF
https://i.imgur.com/JhxfoVJ.jpg

(3)の解答を教えて頂きたいです。
また、(2)について、
Bが非正則ならBx=0を満たす非自明なxが存在するので条件に反し矛盾。よってBは正則。
という議論で解いたのですがこれだと〈Bx,x〉が実数という条件がいりません。なにか間違っているのでしょうか?
どなたかよろしくお願いします。
0411132人目の素数さん
垢版 |
2019/07/24(水) 11:15:49.20ID:uOzBiMoN
以下の場合分け証明法はどういう意味ですか?



推論規則
(場合分け証明法)
命題の列の中に、 A ∨ B, C および C がこの順序で現れ(間隔をおいてでもよい)、前の C には A が仮定として先行しており、
後の C には B が仮定として先行しているならば、 A と B は仮定から取り除いてもよい。
0412132人目の素数さん
垢版 |
2019/07/24(水) 11:50:42.03ID:zVdckgHv
>>408
α=0 のとき、βまたはβ^p-1のいずれかは0でなければならない。β=0の場合も同様。
よってこのときの(a,b)の実数解は、(0,0),(1,0),(-1,0),
0414132人目の素数さん
垢版 |
2019/07/24(水) 20:19:12.38ID:KXlu7sfk
>>410
(2)に関しては余計な条件かもしれませんが、
エルミート行列の話に限定したかっただけでしょう。間違ってはいません。

(3)
C,D 其々エルミート行列なので、固有値(n個, 実数)、固有ベクトル( n個) を次のようにとる事ができる.
s=rank(C), t=rank(D)
ci≠0 (i=1..s),  ci=0 (i=s+1..n)
di≠0 (i=1..t),  di≠0 (i=t+1..n)
C.Vci = ci.Vci,  D.Vdi = di.Vdi (i=1..n)
<Vci, Vcj> = <Vdi, Vdj> = δ[ij]
変換行列: P (正則) , Vci = P[ij] Vdj

条件より
・<D.Vdi, Vdi> = di ≧ 0 (i=1..n)
・i=s+1..n のとき、0 = ci = <C.Vci, Vci> ≧ <D.Vci, Vci>
 = Σ{jk} P[ij]P[ik]^* <D.Vdj, Vdk> = Σ{j} P[ij]P[ij]^* dj ≧ 0
よって P[ij] = 0 (i=s+1..n, j=1..t) ---(A)

行列 Pを列ベクトルの並び (p1,...,pn) とみなすと、
特に p1..pt は 一次独立であり、(A)により実質的には s次元数ベクトルである
よって s ≧ t (数ベクトルの一次独立性)
0415132人目の素数さん
垢版 |
2019/07/24(水) 22:27:05.85ID:lkvm9SmF
>>414
いくつかわからない点があるのですが
1
Vci = P[ij] Vdj
とかけるのはなぜでしょうか?Cの固有ベクトルはDの固有ベクトルの定数倍になるのですか?
2
(A)でjの条件がj=1,…,tとなっていますがj=1,…,nの間違いですか?

あと ^*は複素共役ですよね?
上の2つが分かれば後は理解できたと思います

よろしくお願いします
0416132人目の素数さん
垢版 |
2019/07/24(水) 22:49:02.68ID:2hzv1l24
定義域を{k∈ℤ|0≦k≦n},n∈ℕを定数として
f(k+1)-f(k)=nCk
を恒等的に満たすf(k)を探しています。
f(x)=(x(1+x)^n)/(1-x)
このようなf(x)の、xのk+1次の係数とxのk次の係数の差はnCkになりますよね
(しかしこの係数を扱いやすい式の形で出すことができない(テイラー展開を用いる)ので、これ以上のことは言えかねる)
他に何かありますか?
0417132人目の素数さん
垢版 |
2019/07/24(水) 23:27:29.39ID:KXlu7sfk
>>415
1. Vci = P[ij] Vdj = Σ{j=1..n} P[ij] Vdj です。
同じ添え字が2つ出てきたら和をとる「縮約記法」のつもりでした。
一方で、 ... = <C.Vci, Vci> = ... この添え字は i が2つなのに 和をとりません。
記法の統一ができない時はあまり使うべきではありませんね。

2. j=1,…,t は、そのままです。
dj ≧ 0 が分かっているので、0 ≠ dj > 0 ( j = 1.. t ), dj =0 ( j = t+1..n ) 
よって、
0 = ci = Σ{j=1..n} P[ij]P[ij]^* dj
= |P[i1]|^2 d1 +|P[i2]|^2 d2 + ... + |P[it]|^2 dt + 0 + ... + 0 = 0
≧ |P[ij]|^2 dj = 0 (i=s+1..n, j = 1.. t )
∴ P[ij] = 0 (i=s+1..n, j=1.. t)

3. 「^*は複素共役」です
0418132人目の素数さん
垢版 |
2019/07/24(水) 23:34:01.16ID:tX09B9nR
直感的にほぼ明らかだと思うのですが、記述をどうしたらいいか分かりません。
与えられた命題のうまい言い換えはないでしょうか。

三角形の内部に点Pをとる。また、各辺を直径とする円を描く。
どのような位置にPをとっても、Pはこれら3つの円のうち、少なくとも2つの円について、それぞれの周または内部に含まれることを示せ。
0421132人目の素数さん
垢版 |
2019/07/25(木) 06:30:40.21ID:nLNnK2bo
>>416
f(k) = Σ[j=0,k-1] nCj = 2^n - nCk 2F1(1, -(n-k); k+1)(-1),

ただし nCk = n!/{k!(n-k)!}、2F1(a,b;c)(z) は超幾何級数。
0423132人目の素数さん
垢版 |
2019/07/25(木) 16:26:34.64ID:hDvQRf6v
yをxで微分することをdy/dxと書き、これは分数式ではないが実際の計算では分数であるかのようにdyとdxを切り離して移項したりできる理由を知りたいです。
物理学科出身なので、物理家でわかる範囲で説明する方法があれば教えてください
0424132人目の素数さん
垢版 |
2019/07/25(木) 16:41:59.95ID:uSCR03cf
>>422
医学部の問題としてはかなり簡単

π/10=18°
2π/5=72°
足して90°なので√(1-tt)

cos5θ
=cos(3θ+2θ)
=cos3θcos2θ-sin3θsin2θ
=(4cos^3 θ-3cosθ)(2cos^2 θ-1)-(3sinθ-4sin^3 θ)(2sinθcosθ)
=(4cos^3 θ-3cos^2 θ)(2cos^2 θ-1)-(3-4sin^2 θ)(2cosθ)(1-cos^2 θ)

よってcosθ=zとして
cos5θ
=(4zzz-3z)(2zz-1)-(1-zz)(2z)(4zz-1)
=16zzzzz - 20zzz +5z
cos18°=√(1-tt)=kに上を適用するとcos90°=ゼロになるのでk≠0とあわせ

16kkkk-20kk+5=0
k^2=1-ttを代入して
16tttt-12tt+1=0

t=cos72°<1/2=cos60°
よって1/4>tt=T>0として
16TT-12T+1=0

解いてT=(3-√5)/8

この根号を取るとt=(√5-1)/4

かなり簡単だね。北大でも出そう
0426132人目の素数さん
垢版 |
2019/07/25(木) 16:48:00.89ID:uSCR03cf
一箇所式を書き損じてた。答えは変わらんがすまんな
cosθ=zとしての上の行だけ、なぜかcosの3倍角の公式を4zzz-3zzと書いてる
他の行は表記合ってる
0427132人目の素数さん
垢版 |
2019/07/25(木) 16:50:13.25ID:05QzeysJ
>>422って神戸大でも何年か前に類題が出てなかったっけ?
青チャートにも載ってそうな気がする(未確認)
0428132人目の素数さん
垢版 |
2019/07/25(木) 17:06:20.08ID:t8VXlehk
d/dx(f)のd/dxは、fをxで微分する作用という意味で、分数とは違うのではないんてすか
0429132人目の素数さん
垢版 |
2019/07/25(木) 17:13:19.28ID:uSCR03cf
何をどう流れに沿って当てはめて行けばいいのかにひねりが全くないし計算もハードじゃないので

チェビシェフの多項式の概念を知ってるかどうかだけで得点がかなり分かれてそうであんまいい問題だと思えないなあ

全く知らなかったらキツイだろう
0430132人目の素数さん
垢版 |
2019/07/25(木) 17:27:12.77ID:HUzn3ndY
証明 nが素数でないと仮定すると
         n=ab (1<a≦b<n)
  このとき   a^2=aa≦ab=n 
したがって  a≦√n
これより,nは√n以下のaの約数の素数で割り切れることにな
  り,仮定の条件に反する。よって,nは素数である。  終

師匠、教科書を超越する説明をしていただけませんでしょうか?
よろしくお願いします。頼みます。
0431132人目の素数さん
垢版 |
2019/07/25(木) 17:43:52.19ID:05QzeysJ
思い出した
いつごろからの方針かはわからないけど、横市の医学部の出題方針として、解くべき問題とパスすべき問題を
まぜて出題して、問題を選択する能力も調査するということだったはず
この問題を見たことがない人は勉強不足だという出題意図なんだろうなと思うよ
0432132人目の素数さん
垢版 |
2019/07/25(木) 17:52:27.35ID:05QzeysJ
>>422
θ=1/5πとおいて、3θ=π-2θを使うのが楽ちんだけど、cos(5θ)を求めろとか言われるし…
そういえば、理系の問題だから
(cosθ+i sinθ)^n = cosnθ + i sin nθ を使っていいんじゃないの?
0433132人目の素数さん
垢版 |
2019/07/25(木) 21:07:21.77ID:WiYE7hZn
それぞれ定数でない3つの実数係数の有理式A,B,Cに対し
A²=BCを満たすA,B,Cが存在する

A^n=B かつA^(2-n)=Cを満たすA,B,Cが存在する (n∈ℝ)
は必要十分である

これって正しいですか?
また違う場合どこを直したらいいですかね
0434132人目の素数さん
垢版 |
2019/07/25(木) 21:14:28.60ID:WiYE7hZn
具体的には次の問題を考えていての疑問です。
答案を(できれば具体的に反例をあげながら)指摘してもらえると幸いです

定数でない実数係数有理式f(x)、g(x)であって、
(g(x))^2 =(f(x))^3 -4
を満たすものは存在しないことを示せ

背理法で示す。
(g(x))^2 =(f(x))^3 -4 ...(✳)を満たす実数係数有理式f(x)、g(x)が存在すると仮定する。
(✳) ⇔ (g(x))^2={((f(x))^(3/2))-2}{((f(x))^(3/2))-2}
より、次の2つの場合が考えられる。

(1) g(x)= {((f(x))^(3/2))-2}, g(x)= {((f(x))^(3/2))-2}

(2)±(g(x))^n=((f(x))^(3/2))+2,
±(g(x))^(2-n)=((f(x))^(3/2))-2
(ただしnは2以上の自然数でn=1のとき(1))

(1)のとき、2式の辺辺を引くと0=4で矛盾

(2)のとき、2式の辺辺を引くと

4=±((g(x))^n)∓(1/((g(x))^(n-2)))
⇔ 4={±((g(x))^(2n-2))∓1}/{(g(x))^(n-2)}...(⭐)

ここで、g(x)は定数でない有理式なので(右辺)は定数関数となり得ない
(実際に、(⭐)の分母を払うと
4(g(x))^(n-2)= ±((g(x))^(2n-2))∓1
で右辺と左辺は明らかに一致しない)
よってこのときも不成立

(1),(2)より、背理法で示された。
0435132人目の素数さん
垢版 |
2019/07/25(木) 21:16:50.04ID:7OvjnKpy
論理式がどうなってるのかよくわからん
0437132人目の素数さん
垢版 |
2019/07/25(木) 21:25:13.64ID:7OvjnKpy
※のあたり計算ミスってないか
0439132人目の素数さん
垢版 |
2019/07/25(木) 21:30:29.22ID:gS5FEe1M
lim[n→∞] (1+1/n)^n (eの定義式)
が存在することをコーシーの判定法、つまりコーシー列であることを用いて証明するやり方を教えてください

an = (1+1/n)^n
と置いて
am - an
を計算することは分かってます
0440132人目の素数さん
垢版 |
2019/07/26(金) 00:43:09.24ID:PuqHbG6m
lim[n to inf] cos(2πen!)
が収束するらしいのですが(大学への数学より)、
cosの内部が無限大に発散するのにcos自体は収束するのは、直感的にどう理解すれば良いでしょうか?
無限大に発散するということは、nが十分大きいときでもcosは-1から1までの値を取りつつ、振動するように変化し続けるように思えるのですが…
0441132人目の素数さん
垢版 |
2019/07/26(金) 01:38:45.32ID:Wv4vrX/X
>>440
おぉ、面白い。
証明読んで納得するしかないなぁ。
大数には証明載ってないのかな?
exp(1)をマクローリン展開してn!eの端数部がコーシー列になるね。
面白い。
0442132人目の素数さん
垢版 |
2019/07/26(金) 01:52:56.74ID:v8EdcL7k
なんかまんまその話題今年に入ってからこのスレか面白い問題スレで見た気がするんだがw
n→∞でcos(π*n!*a)が収束するaを挙げよみたいな問題
0443132人目の素数さん
垢版 |
2019/07/26(金) 01:54:21.82ID:v8EdcL7k
n!でなくとも、指数関数より速く発散する関数に対しては一般にそういう数が構成できるという結論になってた気がする
0444132人目の素数さん
垢版 |
2019/07/26(金) 02:06:41.75ID:v8EdcL7k
eにするから分かりづらいんで
例えばαを定数としてn→∞で 
sin(2π*α*10^ n²)を考えるとして

α=Σ(1→∞) 10^-(n²+n)

とおけばα*(10^n²)の小数点以下初めて1が出現するケタはnの増加に連れてどんどん伸びてくから
2π倍すると周期性から上の極限はゼロになる

具体的に書くと0.010001000001……

10^1倍、10^4倍、10^9倍……してくと小数点以下の部分はどんどん小さくなっていく
0445132人目の素数さん
垢版 |
2019/07/26(金) 02:13:01.51ID:v8EdcL7k
en!もこれと全く原理同じ
無限級数和の途中までの項が整数になるような数列Aをかけていって
A倍された無限級数のそれ以降の和の部分(小数部分)はゼロに収束していって

2π倍してcosに入れると整数部分は周期性から全部無視できるから結局収束する
0448132人目の素数さん
垢版 |
2019/07/26(金) 03:55:04.49ID:KDg4xg9g
証明 nが素数でないと仮定すると
         n=ab (1<a≦b<n)
  このとき   a^2=aa≦ab=n 
したがって  a≦√n
これより,nは√n以下のaの約数の素数で割り切れることにな
  り,仮定の条件に反する。よって,nは素数である。  終

n=157のとき、n=√157×√157
nは素数でないのに、√157で割り切れることはおかしい。
よって、nは素数

よろしいでしょうか?
0449132人目の素数さん
垢版 |
2019/07/26(金) 04:29:01.24ID:KDg4xg9g
そもそもnが素数なら、n=abに具体的な数値を入れて考えてみることが不可能。
0450132人目の素数さん
垢版 |
2019/07/26(金) 06:00:14.71ID:79RhfTqe
マンホールがなぜ丸いか→円は定幅図形なので、ふたがずれても穴の中に落ちない。
という問題と回答があります。

これを数学的に解釈すると正しいのでしょうか。

実際のマンホールには、フタは厚みがあり、マンホールを上に置いたときに
落ちないように穴には出っ張りがあります。

出っ張り分を考えると、直径48cmの丸い穴に、直径50cmの丸いフタを通せということなので、
「通らない」というのが感覚的に分かります。

これを数学的に、直径100cmの円形の穴に、
直径100cmの円形のフタ(厚さが0)を通すことは出来るか?と考えた場合、通るのでしょうか。
0452132人目の素数さん
垢版 |
2019/07/26(金) 10:58:30.57ID:cuO54iw8
>>439
 k! ≧ 2^(k-1) より
 C(n,k) ≦ (n^k)/k! ≦ (n^k)/{2^(k-1)},
 a_n = 1 + C(n,1)/n + C(n,2) /(n^2) + ・・・・
  = 1 + (1 + 1/2 + 1/4 + ・・・・ )
  = 1 + 2
  = 3,
∴ a_n は上に有界

{1,1,・・・,1,1-1/n} のn個で 相加-相乗平均すると (1-1/nn)^n > 1-1/n,
 (n-1)個
∴ (1+1/n)^n > {1+1/(n-1)}^(n-1),
∴ a_n は単調増加

これから、a_n は収束し、よってコーシー列になります。(間接的ですが)

・別法
http://detail.chiebukuro.yahoo.co.jp/qa/question_detail/q128032107
0453132人目の素数さん
垢版 |
2019/07/26(金) 11:28:07.85ID:99q/BPu9
>>450
なぜ数学的な解釈をするときに出っ張りが無いことにするのかがわからない
マンホールの問題は数学的にも
> 直径48cmの丸い穴に、直径50cmの丸いフタを通せるか
という問題だろう
「円ではなく例えば四角形の場合は出っ張り部分をものすごく広くとらなければならなくなるから」が答えとして正しいのかも知れないけど
「人が通ることが可能であり、かつ、蓋が落ちることが無いようにし、かつ、材料費が最も安くなるのは円だから」がもっと正しいのかも知れない
0455132人目の素数さん
垢版 |
2019/07/26(金) 13:31:20.44ID:6lyO6ULv
ヤコビアンと面素ベクトルの符号の決め方に関して質問です
以下(vec)はベクトルとする
r(vec)=rcosθe_x(vec)+rsinθe_y(vec)+2r^2e_z(vec)
で表される
回転放物面z=2x^2+2y^2,0≦z≦8
において、dS(vec)を回転放物面から外側に出る向きとします

ここからが質問なのですが
∂r(vec)/∂θと∂r(vec)/∂rの外積をとると面素が求まります
しかし今、面素ベクトルは外側の向きであるので
符号の決め方をdS(vec)=n(vec)*dSと法線ベクトルn(vec)に担当させてdSだけを求めるとします
するとdSはヤコビアンの絶対値を取ります、
ここで
求めるヤコビアンの正負を判別する方法がわかりません
ヤコビアンは外積の順番を変えると符号が変わる上に、絶対値を取る際に中身が正であるのにも関わらず符号を変えてしまうと題意にそわない向きの面素ベクトルが出てしまいます
また仮にこのdSの符号の決め方が解決したとした時、符号を担当させたn(vec)は+にしたら外向き、-にしたら内側の向きと単純に考えても差支えはないのでしょうか?
0456挑発吉川晃司
垢版 |
2019/07/26(金) 14:29:21.91ID:KDg4xg9g
証明 nが素数でないと仮定すると
         n=ab (1<a≦b<n)
  このとき   a^2=aa≦ab=n 
したがって  a≦√n
これより,nは√n以下のaの約数の素数で割り切れることにな
  り,仮定の条件に反する。よって,nは素数である。  終

わからねえのか?w
0457132人目の素数さん
垢版 |
2019/07/26(金) 14:44:16.94ID:symLErG5
>>455
自分でどう定義するかだけ
r, θ は x, y を極座標にして mapping しただけだから
(∂/∂θ)×(∂/∂r) は -z 方向で、上に開いた放物面なら外向きというだけ
0458挑発吉川晃司
垢版 |
2019/07/26(金) 15:36:03.36ID:KDg4xg9g
>>457
証明 nが素数でないと仮定すると
         n=ab (1<a≦b<n)
  このとき   a^2=aa≦ab=n 
したがって  a≦√n
これより,nは√n以下のaの約数の素数で割り切れることにな
  り,仮定の条件に反する。よって,nは素数である。  終

わからねえのか?w
0459132人目の素数さん
垢版 |
2019/07/26(金) 15:49:45.01ID:6lyO6ULv
>>457
普通の座標変換レベルならば3次元だし微小な変化の方向は見て取れるからそこから外積の方向を見る感じが一般的ですかね?
0461132人目の素数さん
垢版 |
2019/07/26(金) 16:33:19.12ID:7qSCwZZH
>>445
分かりやすい
直感的な理解でいうなら整数部分が無視できるところがミソってことかな
0462132人目の素数さん
垢版 |
2019/07/26(金) 18:33:33.64ID:PuqHbG6m
>>445
とても分かりやすく説明していただいてありがとうございます。
整数部分が無視できるというのは、感覚的にも納得できました。
10^mだと一目瞭然ですね。
0463132人目の素数さん
垢版 |
2019/07/26(金) 19:33:34.24ID:v8EdcL7k
>>462
わざわざ書くほどのことではないけど
一応e*n!の少数部分がゼロに収束する証明も書いておくと

e=Σ(k=0→∞) 1/n!だから
n!倍するとk=0-nの項は全部整数になる
非整数の項を順に書くと1/(n+1),1/(n+1)(n+2),1/(n+1)(n+2)(n+3)…と

α=1/(n+1)として、それぞれα、α²、α³……と同じか小さいかなので、
αの累乗の無限和で上から抑えて、非整数部の和は1/nより小さい

だからn→∞でゼロに収束する
0464132人目の素数さん
垢版 |
2019/07/26(金) 20:01:15.33ID:KDg4xg9g
証明 nが素数でないと仮定すると
         n=ab (1<a≦b<n)
  このとき   a^2=aa≦ab=n 
したがって  a≦√n
これより,nは√n以下のaの約数の素数で割り切れることにな
  り,仮定の条件に反する。よって,nは素数である。  終

わからないのならわからないとちゃんと言えよ。無視するなよ。
0465132人目の素数さん
垢版 |
2019/07/26(金) 20:02:18.43ID:KDg4xg9g
証明 nが素数でないと仮定すると
         n=ab (1<a≦b<n)
  このとき   a^2=aa≦ab=n 
したがって  a≦√n
これより,nは√n以下のaの約数の素数で割り切れることにな
  り,仮定の条件に反する。よって,nは素数である。  終

わからないのならわからないとちゃんと言えよ。無視するなよ。
0466132人目の素数さん
垢版 |
2019/07/26(金) 20:21:26.71ID:qwHm+CXI
まず、何を証明したいのか書きなさい。
> 仮定の条件に反する
ってのは、 「nが素数でないと仮定」この仮定ではなくて、もっと前段の本文にあるのでしょう?
0467132人目の素数さん
垢版 |
2019/07/26(金) 20:57:47.84ID:E95HAM8V
>>448
何を証明しようとしているのかが不明。意味をなしていない。全てが間違っている。

仮定が「nが素数でない」
↓         
結論が「nは√n以下のaの約数の素数で割り切れる」

これを「矛盾」としているが矛盾は起きていない。
1<nを仮定しており、仮定よりnは素数でないのだから、素数の約数を必ず持つ。

背理法を理解できていないのでは?

「n=√157×√157
nは素数でないのに、√157で割り切れることはおかしい。」

√157は整数でないので割り切れるとか素因数とかの概念は適用できない。
abが整数という仮定が必要なのを見落としている。
0469132人目の素数さん
垢版 |
2019/07/26(金) 22:20:49.73ID:9FF4U//m
https://dotup.org/uploda/dotup.org1906354.jpg

右の画像のように決まった直径の円を隙間がないように重ねていった場合なんですが
重なった部分の累計面積は
円の直径が例えば 5cmと10cmではどちらが大きくなっていくんでしょうか。

重ねる率にもよると思いますが傾向だけでもわかると助かります。
0470132人目の素数さん
垢版 |
2019/07/26(金) 22:48:18.78ID:6wxjrxiB
>>469
5cmの重なりの図全体を二倍したらそのまま10cmの図になる
このとき重なりの面積は4倍になり重なりの個数は1/4になる
したがって全く同じ
0471132人目の素数さん
垢版 |
2019/07/26(金) 22:50:28.18ID:9FF4U//m
>>470
ありがとうございます。
とてもわかりやすい説明で理解できました。
そのまま2倍変換したと発想すればよかったんですね。
助かりました!
0472132人目の素数さん
垢版 |
2019/07/26(金) 22:56:46.32ID:UHf6zCi5
>>450
数学的に解釈すると
その理由なら、定幅図形であればいいハズで
丸である必要は無いって結論にしかならん
0473132人目の素数さん
垢版 |
2019/07/27(土) 00:10:34.86ID:owlHsi4n
セックスしたいのですが相手がいません
この問題はどう解決したらいいでしょうか
0476132人目の素数さん
垢版 |
2019/07/27(土) 10:50:23.46ID:xyZ5RUAW
分かスレってここじゃないの?
0477132人目の素数さん
垢版 |
2019/07/27(土) 11:09:50.06ID:HReJMvgd
>>466>>467
すいません。気づきました。仮定の条件に値するものがすぐ上にありました。

素数の判定
nを2以上の自然数とする。
nが√n以下のどの素数でも割り切れなければ,nは素数である。

証明 nが素数でないと仮定すると
         n=ab (1<a≦b<n)
  このとき   a^2=aa≦ab=n 
したがって  a≦√n
これより,nは√n以下のaの約数の素数で割り切れることにな
  り,仮定の条件に反する。よって,nは素数である。  終

仮定の条件に反する=仮定の条件と反対だ=仮定の条件と逆だ
でよろしいでしょうか?ここ重要だと思うんで教えてください。
非常に態度が悪くてすいませんでした。
0478132人目の素数さん
垢版 |
2019/07/27(土) 11:13:26.26ID:AnIO+yRP
>>476
すいません。気づきました。

スターリングの公式で
 a_n = (n!)^(1/n)
  = (n/e) (2πn)^(1/2n) (1 + 1/12n + ・・・・)^(1/n)
  = (n/e) exp{(1/2n)log(2πn)} {1 + 1/(12nn) + O(1/n^3)}
  = (n/e) {1 + (1/2n)log(2πn) + O(log(n)^2/n^2)} {1 + 1/(12nn) + O(1/n^3)}
  = (n/e) + (1/2e)log(2πn) + O(log(n)^2/n)
差分して
 a_(n+1) - a_n = (1/e) + (1/2en) + O(log(n)^2/n^2) → 1/e. (n→∞)
0480132人目の素数さん
垢版 |
2019/07/27(土) 11:40:25.86ID:HReJMvgd
すいません。気づきました。
『反する』は『対偶』であって『逆』ではありませんね。
0481イナ ◆/7jUdUKiSM
垢版 |
2019/07/27(土) 11:49:40.64ID:W9txtZbT
___∩∩___素直に微分し
⊂(-_-))`⌒ つ/|たらゎ
_____`υ_____//|zz...
 ̄∩∩ノ ̄ ̄‖
(`.`))⌒ヾ,‖クンクン……
__υυ`υυ...  >>469敷き詰めた円の半径をrとすると、円1枚あたりの重なり部分の面積の半分は、
f(r)=πr^2-6r^2√3/4
=πr^2-3r^2√3/2
f'(r)=(2π-3√3)r>0
∴rの値による極値はない。
すなわち直径2rが大きくなればなるほど重なり部分の面積は大きくなり、その大きさは半径の2乗に比例すなわち直径の2乗に比例する。
>>311但し、単位面積あたりの重なり部分の面積は、
(1-3√3/2π)×100%で一定ある。
0482イナ ◆/7jUdUKiSM
垢版 |
2019/07/27(土) 12:02:55.46ID:W9txtZbT
>>481
単位面積あたりの重なり部分の面積は、
(1-3√3/2π)×100≒17.3(%)
0483132人目の素数さん
垢版 |
2019/07/27(土) 12:05:01.22ID:kZbB6/+U
それを「単位面積あたりの重なり部分の面積」と呼んで良いものか、はてさて
0484132人目の素数さん
垢版 |
2019/07/27(土) 12:10:15.36ID:gpwhXhuz
>>477
必要な事を書かず
余計な事ばかり書くから
意味不明な証明になってしまう

まず、文字変数の定義は全て書け
a,bは何だ?

nが合成数であると仮定すると
ある自然数 a,b が存在し
n = ab (1 < a ≦ b < n)
を満たす。

1ステップごとに文章書け
> nは√n以下のaの約数の素数で割り切れる
ではなく、

a > 1 であるから、aを素因数分解すると 2以上 a以下の素因数が1つはある。
それを p とすれば、 p ≦ a ≦ √n である。
pは n = ab を割り切る。

「仮定の条件」なんて横着せずに、明示すべき。
問題文や証明内には、A⇒Bという形が複数あり、
どの仮定 A を指しているのか不明瞭になりやすい。

したがって、
自然数 n が合成数 ⇒ n を割り切る素数 p で√n 以下となるものが少なくとも 1つは存在する。
が示せた。

対偶を取れば
自然数 n が √n 以下の、どの素数でも割り切れなければ
n は素数である。
0485132人目の素数さん
垢版 |
2019/07/27(土) 12:14:51.64ID:HWuO6KRp
それ、偏差値65(偏差値70は夢のまた夢)くんだよ
0486イナ ◆/7jUdUKiSM
垢版 |
2019/07/27(土) 12:39:25.50ID:W9txtZbT
定義も偏差値も関係ない。証明なんかしなくていい。前>>482わかんなかったら微分。とにかく嗅覚を働かせて答えを出す。それが数学っちゅーもんや。
___∩∩________
⊂(-_-))`⌒ つ/|
______`υ____//|zz...
∩∩ ̄ ̄ ̄ ̄‖
`.`))⌒ヾ,クン‖……
υυ`υυ.....
0487132人目の素数さん
垢版 |
2019/07/27(土) 13:00:50.63ID:t80iRulR
>>478
問題を噛み砕くと
 命題P : 「 nが√n以下のどの素数でも割り切れない」(p) ならば 「 nは素数である」(q) (論理式: p→q)
を証明しろという事です。

・証明A
P の対偶は 「 nは素数でない」ならば 「 nが√n以下のある素数で割り切れる」 (¬q→¬p)
(... 中略 ...) 対偶が証明できたので、P は真である。

・証明B (同じ事をやや厳密に)
「 nが√n以下のどの素数でも割り切れない」(p) と「 nは素数でない」(¬q) が共に真となる事があり得ると仮定
つまり ∃n ( p∧¬q ) が真であると仮定
(... 中略 ...)
「 nが√n以下のある素数で割り切れる」(¬p) が導かれたが仮定である p に『反している』ため、これは矛盾である。(∵ 排中律)
つまり ¬∃n ( p∧¬q ) が真であるとの結論
よって p ならば q である。(∵ ¬∃n ( p∧¬q ) ) = ∀n ((¬p)∨q ) = ∀n ( p → q ) )

一応、用語を整理しておくと、論理式 : p→q = (¬p)∨q に対して
 否定: ¬(p→q) = p∧(¬q)
 逆: p←q = p∨(¬q)
 裏: (¬p) → (¬q) = p∨(¬q)
 対偶: (¬p) ← (¬q) = (¬p)∨(¬¬q) = (¬p)∨q (つまり p → q と等値)
となっています。
「否定(≒...に反する, ...の反対)」と「逆」は異なる概念なので注意が必要です。
>>480 その辺りがまだゴッチャになってるような気がするけど大丈夫?
0490132人目の素数さん
垢版 |
2019/07/27(土) 15:32:33.06ID:fFKy3PnN
aで割ると、

    α+√α   √α(√α +1)
P/a = -------- = ------------ = √α
    1+√α    1+√α


    α+2a√α  √α(√α +2a)
Q/a = -------- = ------------ = √α
    2a+√α     2a+√α
0492132人目の素数さん
垢版 |
2019/07/27(土) 23:30:16.75ID:9BSdEjiU
Excelのセルに[=NORMINV(RAND(),10,1)]と入力すると平均10標準偏差1の正規乱数が表示されるのですが、
この正規乱数を5セル作って、その5セルの標本標準偏差をSTDEV関数で求めることをマクロで10万回繰り返した平均値は1.00ではなく0.94となります(誤差ではなく0.06ずれる)。
また、この正規乱数のセルの数が2の場合、同様に10万回繰り返した標本標準偏差の平均値は0.80になります。
この正規乱数のセルが増えれば増えるほど10万回繰り返した標本標準偏差の平均値は1.00に近づきます。

なぜ正規乱数のセルの数が少なければ少ないほど正規乱数の標準偏差は1.00よりも少なくなるか分かる人がいれば教えてください。
標本標準偏差は不偏分散から計算されるはずですが、不偏といわれつつn数が少ない時は偏ってたりします?
それともExcelの関数のバグ?
0493132人目の素数さん
垢版 |
2019/07/28(日) 04:36:34.43ID:YMtuI2+B
>>487
問題を噛み砕くと
 a_n = (n!)^(1/n)
  = n/e + 1/(2e)・log(2πn) + 1/(8en)・{log(2πn)^2 + 2/3} + O(log(n)^3 /n^2),
差分して
 a_(n+1) - a_n
  = 1/e + 1/(2en) - 1/(8en^2)・{log(2πn)^2 -2log(2πn) + 8/3} + O(log(n)^3/n^3)
を証明しろという事です?
0495132人目の素数さん
垢版 |
2019/07/28(日) 06:06:13.96ID:YMtuI2+B
>>492
正規乱数からなるの大きさnの標本について
標本の標準偏差sは確率変数で、その期待値は
 E(s) = {Γ(n/2)/Γ((n-1)/2)}・(√2n)/(n-1)・√<s^2>
    = {Γ(n/2)/Γ((n-1)/2)}・√{2/(n-1)} σ,
ここにΓはガンマ函数、σは母分布の標準偏差です。

お尋ねの場合は σ=1 で
 n=5 のとき E(s) = 0.939985602986625σ
 n=2 のとき E(s) = 0.797884560802865σ
です。
各セルのRAND() 同士の相関はかなり低い(独立)と予想されます。
Excelのバグではないでしょう。

序ながら、標本の分散s^2は χ^2 分布に従い、その期待値は
 E(s^2) = {(n-1)/n}σ^2,
です。
0496132人目の素数さん
垢版 |
2019/07/28(日) 08:35:07.07ID:YMtuI2+B
不偏分散 (n/(n-1))s^2 の期待値は σ^2 です。
しかし 平方根の期待値は σ よりやや大きく
 √(n/(n-1))・E(s) = {Γ(n/2)√(n/2)/Γ((n+1)/2)}・σ
となります。
 { }内の係数は1よりやや大きいですが、nとともに単調減少して1に収束します。
0497132人目の素数さん
垢版 |
2019/07/28(日) 09:53:35.31ID:VxjhTh5s
>>496
回答ありがとうございます。
統計学的に説明できそうな現象らしいということが分かりました。

しかしながら、私は昔、統計検定準一級は合格しましたが、こういうのは予想できませんでした。
こういうことを数学的に理解できるようになるには、やっぱり私の勉強が足りませんね。
一級合格を目指すやる気が出てきました。
0498132人目の素数さん
垢版 |
2019/07/28(日) 18:00:08.00ID:mAKftFou
https://imgur.com/SFxFvYQ

この無限等比級数?の問題の解き方分かる方いらっしゃいますか?
解き方のアプローチだけでも教えてほしいです。。。
0499132人目の素数さん
垢版 |
2019/07/28(日) 20:54:39.76ID:pKqzo8Dk
>>498
ラグランジュの未定乗数法で解けると思いますよ
0501132人目の素数さん
垢版 |
2019/07/28(日) 21:05:11.33ID:L58vQkF+
n乗根を^(1/n)で代用することにします

n,mを正の整数とするとき、
α=2^(1/n)+3^(1/m)
とおく。整数係数の多項式f(x)で、f(α)=0を満たすものが存在することを示せ。

という問題なのですが、αをn乗したりmn乗してもうまくいきません。
代数的数なので必ず存在するはずなのですが、その予備知識を使わずに、どうアプローチしたらいいでしょうか?
0507132人目の素数さん
垢版 |
2019/07/28(日) 22:51:22.69ID:1Yy4Y6Pu
>>504
誰でもできそうなのは、泥臭く何とか乗と移項を繰り返して帰納法
最低次数だとめんどくさそう
0509132人目の素数さん
垢版 |
2019/07/29(月) 00:56:07.14ID:ai7lTVNF
一般的に細かいことは無視してz=f(x,y)で表される2変数関数があったときz=cの平面を表す方程式はf(x,y)-c=0ですよね?
0511132人目の素数さん
垢版 |
2019/07/29(月) 07:21:32.36ID:3Db6msx5
>>498
ラグランジュでもいいし
答えも大体nが2の場合で試してみれば予想されるようなシンプルなものだから予想してそれを示すだけでもよい
そしたら高1レベルの数学で解けるっちゃ解ける
ある点で最大になるとして、例えばx1とx2が異なっていたら
ずらしてどうなるかとかね

要は全部均等じゃなきゃちょっとずらすだけでもっと大きいSが作れるって簡単に示せる
0512132人目の素数さん
垢版 |
2019/07/29(月) 07:49:26.17ID:6L7yKrQP
>>498

(4) nを2以上の自然数、 x_i を
  Σ[i=1,n] (x_i)^n = 1,   x_i > 0
を満たすような実数とする。この時、
 S = Σ[i=1,n] x_i
を最大にするような {x_i} を求めよ。さらに、その時のSを求めよ。

(略解)
 S_k = Σ[i=1,n] (x_i)^k
とおく。
チェビシェフ不等式(*)で
 S_{k+1} ≧ (S/n) S_k,
    等号成立は x_1 =・・・・ = x_n = S/n のとき。
∴ S_k ≧ n(S/n)^k,
∴ 1 = S_n ≧ n(S/n)^n,
∴ S ≦ n(1/n)^(1/n),

* チェビシェフ不等式
 n・S_{k+1} - S・S_k = Σ[i<j] (x_i - x_j){(x_i)^k - (x_j)^k} ≧ 0,
0514132人目の素数さん
垢版 |
2019/07/29(月) 10:32:31.38ID:6L7yKrQP
>>498

(4) nを2以上の自然数、 x_i を
  Σ[i=1,n] (x_i)^n = 1,   x_i > 0
を満たすような実数とする。この時、
 S = Σ[i=1,n] x_i
を最大にするような {x_i} を求めよ。さらに、その時のSを求めよ。

(略解)
 y=x^n は下に凸だから Jensen で
  1 = Σ[i=1,n] (x_i)^n ≧ n (S/n)^n,
∴ S ≦ n (1/n)^(1/n),
0515132人目の素数さん
垢版 |
2019/07/29(月) 10:48:20.98ID:6L7yKrQP
>>505
いいえ。
局所的にはユークリッド空間と見なせるような、図形や空間(位相空間)のことです。
多様体上の好きなところに局所的な座標を描き込むことができます。
0517132人目の素数さん
垢版 |
2019/07/30(火) 00:00:42.68ID:ODJMMWQd
α=2^(1/3)+3^(1/5)
とおく。整数係数の多項式f(x)で、f(α)=0を満たす具体的な多項式を与えよ。

具体的次数で考えてみてるのですが全然解けません
わかる方いらしたらお願いします
0518132人目の素数さん
垢版 |
2019/07/30(火) 00:51:57.07ID:dUtFV1tV
既約じゃなくてもいいんなら移行して15乗してみればいいんでないの
0520132人目の素数さん
垢版 |
2019/07/30(火) 02:20:17.32ID:X/fYIYit
f(x) = {(x-a)^5 -3} {(xx+ax+aa)^5 -3(2x+a)(x^4 +2ax^3-6aax^2 -7a^3 x+a^4) +9},
ただし、a = 2^(1/3) = 1.25992105 (無理数だが)
0521132人目の素数さん
垢版 |
2019/07/30(火) 02:51:42.76ID:zQOHe42V
一点でのみ微分不可能な複素関数は「任意の点で正則」ではないですか?
0522132人目の素数さん
垢版 |
2019/07/30(火) 02:52:46.37ID:zQOHe42V
あと「その点でのみ正則でない」のですか?
0524132人目の素数さん
垢版 |
2019/07/30(火) 02:58:43.83ID:zQOHe42V
2番目の質問が分からないのは、微分不可能な点のすぐ近くの点は正則ではないと思うのでそこも含めないといけないのかよくわからないからです。近くの点というのがよくわからないです。
0525132人目の素数さん
垢版 |
2019/07/30(火) 03:08:51.34ID:V9mOKOqg
そもそも質問がまずよくわからないんですけど
正しい日本語でお願いしますね
0526132人目の素数さん
垢版 |
2019/07/30(火) 03:08:58.61ID:zQOHe42V
最初の質問は却下します。
0527132人目の素数さん
垢版 |
2019/07/30(火) 03:11:05.11ID:zQOHe42V
質問:
ある一点でのみ微分不可能な複素関数は「その点でのみ正則でない」と表現しますか?
「その点の近傍で正則ではない」と表現しますか?
または他の表現ですか?
0528132人目の素数さん
垢版 |
2019/07/30(火) 03:12:36.19ID:V9mOKOqg
一点で正則でない
一点の近傍で正則でない

明らかに違うことですよね
0529132人目の素数さん
垢版 |
2019/07/30(火) 03:14:50.41ID:zQOHe42V
>>528
どちらが正しいですか?
0530132人目の素数さん
垢版 |
2019/07/30(火) 03:15:20.71ID:zQOHe42V
または別の表現が必要ですか?
0532132人目の素数さん
垢版 |
2019/07/30(火) 03:18:45.20ID:zQOHe42V
>>531
近傍という表現が正しいのか分かりませんが、微分不可能な点のすぐ近くの点も、近傍が全て微分可能ではないので正則ではないと思ったので、単純に正則でない場所は「点」という表現でいいのか疑問に思いまし。
0534132人目の素数さん
垢版 |
2019/07/30(火) 03:20:53.38ID:LBMRjIX1
微分不可能な点のすぐ近くの点は普通に正則たりえます
fがある点xで正則、という事の定義は、(複素解析なら)fはxを含むある領域で満遍なく微分可能な事、つまりxの十分近くの領域を取ればその任意の点で微分可能になる事ですよね
例えばf(z):=1/zとか考えてみればわかると思いますが
これは原点0で微分不可能で、それ以外で微分可能です


さて、原点に近い点xを取ってきた時、原点を含まないようにxを中心とした円を書けるでしょうか
もし書けないならfはその点では正則ではありません
書けるならfはその点では正則という事です

原点以外の点では正則になりますよね?
0535132人目の素数さん
垢版 |
2019/07/30(火) 03:22:55.15ID:zQOHe42V
>>534
何となく分かりました。
0536132人目の素数さん
垢版 |
2019/07/30(火) 03:23:29.27ID:zQOHe42V
>>533
そんなこと言ってません。
0537132人目の素数さん
垢版 |
2019/07/30(火) 03:24:35.95ID:LBMRjIX1
>>532
ある点で正則可能な事の定義は「全ての近傍で微分可能」な事ではなくて
「ある近傍の全ての点で微分可能」な事です
満遍なく微分可能であるような領域を一つ見つけられれば正則なのです
微分不可能な点のすぐ近くに点を取ったとして、微分不可能な点に触れないようにその点の周りに円が書ければそれは正則という事です
0538132人目の素数さん
垢版 |
2019/07/30(火) 03:26:42.07ID:zQOHe42V
「全て微分可能ではない」

全て微分不可能という意味ではなく、ある点で微分不可能という意味です。
0539132人目の素数さん
垢版 |
2019/07/30(火) 03:29:16.28ID:zQOHe42V
>>537
>ある点で正則可能な事の定義は「全ての近傍で微分可能」な事ではなくて
「ある近傍の全ての点で微分可能」な事です

すみません、全ての近傍というのは近傍全体という意味で書きましたが正確でないかもしれないですね。
0540132人目の素数さん
垢版 |
2019/07/30(火) 03:29:58.99ID:zQOHe42V
ある近傍の全体です。
0541132人目の素数さん
垢版 |
2019/07/30(火) 03:56:42.99ID:24Akdc1R
a,b,c,dは正の整数で、
abc=d
a^3+b^3+c^3=d^2
を同時に満たす。

(1)このような(a,b,c,d)を1組求めよ。
(2)このような(a,b,c,d)をすべて決定せよ。
0543132人目の素数さん
垢版 |
2019/07/30(火) 06:42:01.81ID:X/fYIYit
1^3 + 2^3 + ・・・・・ + n^3 = (1+2+・・・・+n)^2,    (長岡京の公式)
0544132人目の素数さん
垢版 |
2019/07/30(火) 10:31:53.21ID:fKCY+YxO
最初の問題だけでもいいので答えを教えてください。
https://i.imgur.com/p0Ohy6bh.jpg
0546132人目の素数さん
垢版 |
2019/07/30(火) 10:43:35.69ID:fKCY+YxO
それは分かりますが私の回答は自信が無いです。
x=±yノットイコール0であるような点で微分可能、いかなる点でも正則で無いというは正しいのでしょうか。
0548132人目の素数さん
垢版 |
2019/07/30(火) 10:50:49.45ID:fKCY+YxO
そのですが。
0549132人目の素数さん
垢版 |
2019/07/30(火) 10:51:04.32ID:fKCY+YxO
そのままです。
0550132人目の素数さん
垢版 |
2019/07/30(火) 10:53:32.96ID:kZTjob9P
「∀z∈C f(z)は正則」でない
∀z∈C 「f(z)は正則でない」

どちらですか?
0551132人目の素数さん
垢版 |
2019/07/30(火) 11:01:10.34ID:fKCY+YxO
すみません、違いがわからないです。。
0552132人目の素数さん
垢版 |
2019/07/30(火) 11:02:10.55ID:kZTjob9P
私はあなたが下を言ってるように見えるんですけど、それは違いますよ
0553132人目の素数さん
垢版 |
2019/07/30(火) 11:02:33.33ID:fKCY+YxO
>>551
すみません、分かりました。
下だと思います。
正則な点が無いという意味です。
0554132人目の素数さん
垢版 |
2019/07/30(火) 11:03:02.77ID:fKCY+YxO
>>552
そうなのですか?
理由はなんでしょうか。
0556132人目の素数さん
垢版 |
2019/07/30(火) 11:04:55.83ID:fKCY+YxO
近傍領域の内部全部の点で微分可能ということですか?
0558132人目の素数さん
垢版 |
2019/07/30(火) 11:07:04.54ID:fKCY+YxO
>>557
ある近傍だと思います。
めちゃくちゃ小さい近傍を取ってそこで微分可能なら正則だと思います。
0560132人目の素数さん
垢版 |
2019/07/30(火) 11:08:24.49ID:fKCY+YxO
もしかして線に沿って近傍を取ったら全部で微分可能だから正則ということですか?
0561132人目の素数さん
垢版 |
2019/07/30(火) 11:08:59.66ID:fKCY+YxO
>>559
なぜでしょうか。
0563132人目の素数さん
垢版 |
2019/07/30(火) 11:10:13.79ID:fKCY+YxO
>>562
さっきの問題はy=±xの線の上に微分可能な点が並んでいると思うのですがその線です。
そもそもそれ自体間違ってるかもしれないですが。
0565132人目の素数さん
垢版 |
2019/07/30(火) 11:12:56.55ID:fKCY+YxO
>>564
だとしたらもはや終わってますね。
0566132人目の素数さん
垢版 |
2019/07/30(火) 11:14:25.65ID:fKCY+YxO
落第です。
0569132人目の素数さん
垢版 |
2019/07/30(火) 16:35:39.77ID:k/D2krS0
半円の弓形ってなぜ半径なの?
0571132人目の素数さん
垢版 |
2019/07/30(火) 23:08:04.94ID:6Bhodj3W
🤯
0573132人目の素数さん
垢版 |
2019/07/31(水) 12:39:58.37ID:sbhMOD3s
P(x,y)=(θ+r*sinθ,1+r*cosθ) (0≦θ≦2π)、r>1とすると、
Pの軌跡はトコロイドであるが、
r= t/sint を満たす 0<t<πが存在し、θ=π-t、θ=π+tの二点のみで、
異なるθに対してPが一致し、結節点を持つ。
この時、θ=π-t〜π+tまでの軌跡で囲まれる面積Sを求めよ、

という問題なのですが、
他に結節点無いなら線積分で一発だ、と思い、
S=∫(π-t〜π+t) y*dx/dθ dθ
=∫(π-t〜π+t) (1+r*cosθ)^2 dθ  [θ-π→φと置換]
=∫(-t→t) (1-r*cosφ)^2 dφ
=2∫(0→t) 1 - 2r*cosφ + rr *cos^2 φ dφ

=2[φ- 2r*sinφ+ rr*φ/2 +rr * sin2φ /4 ](0→t)

=2t - 4tt/sint + ttt/sin^2 t + tt/tant と思ったのですが

模範解答は ttt/sin^2 t + tt/tant - 2t となっていて間違いでした

これはどこでミスをしてしまったのか教えていただけないでしょうか
0574132人目の素数さん
垢版 |
2019/07/31(水) 12:51:34.05ID:/QKZxHJh
>>572
理解力がたりず申し訳ありません
なぜこれは整数係数になると予知できたのでしょうか?
0576132人目の素数さん
垢版 |
2019/07/31(水) 14:28:10.49ID:sbhMOD3s
>>575
おーまいがー
ありがとうございます!
見直しても気づかないので終わってますね^^;
0577132人目の素数さん
垢版 |
2019/07/31(水) 16:33:39.87ID:WA6HlvHH
>>572

X = x-aω, X~ = x-aω~, ω≠1 は1の3乗根
とおくと
XX~ = xx+ax+aa, (X-X~)^2 = -3aa,

∴(X^5 - 3)(X~^5 - 3)
= (XX~)^5 - 3[(X)^5 + (X~)^5] + 9
= (XX~)^5 - 3(X+X~)[(X^4) -(X^3)(X~) +(XX~)^2 -X(X~)^3 +(X~)^4] + 9
= (XX~)^5 - 3(X+X~)[(XX~)^2 + 3(XX~)(X-X~)^2 + (X-X~)^4] + 9
= (xx+ax+aa)^5 - 3(2x+a)[(xx+ax+aa)^2 - 9aa(xx+ax+aa) + 9a^4] + 9,

なぜ整数係数になるんでしょう???
0578132人目の素数さん
垢版 |
2019/07/31(水) 22:15:21.59ID:Tfohd+Nr
>>577

α = a + b, a=2^(1/3), b=3^(1/5)
の最小多項式は、

 2  4
 Π Π(x - a e^(2πi n/3) - b e^(2πi m/3)) = ((x-a)^5 -3)(X^5 - 3)(X~^5 - 3)
n=0 m=0
0579132人目の素数さん
垢版 |
2019/07/31(水) 22:47:58.52ID:sbhMOD3s
>>577
力量がたりず申し訳ありません

f(x) = {(x-a)^5 -3} {(xx+ax+aa)^5 - 3(2x+a)[(xx+ax+aa)^2 - 9aa(xx+ax+aa) + 9a^4] + 9}

この状態だとxの式の表示に無理数のaが含まれているので
これを整理してxについての多項式にすると全て整数係数になるのが自明だというのが分かりませんでした

最小多項式というのがあるのですね、勉強してみますありがとうございます
0580132人目の素数さん
垢版 |
2019/07/31(水) 22:49:30.25ID:j27kxDIj
>>577
こんなんでどうですかね。
a1 := a, a2 := a.ω, a3 := a.ω~
(x-a1)(x-a2)(x-a3) = x^3 - 2

基本対称式:
u1 = a1 + a2 + a3 = 0
u2 = a1.a2 + a2.a3 + a3.a1 = 0
u3 = a1.a2.a3 = 2

f(x) = ((x-a1)^5 - 3)((x-a2)^5 - 3)((x-a3)^5 - 3) {係数はa1,a2,a3 の対称式}
= F(x, u1,u2,u3) {(整数係数)多変数多項式}
= F(x, 0,0,2)
対称式は必ず基本対称式で表せるの? それが整数係数になるの?って辺りは
ヤング図形を使えば感覚的に分かる。

f(a+b) = ((a+b-a)^5 - 3)(...)(...) = (b^5 - 3)(...)(...) = 0

>>578
最小多項式である事はどうやって示せるんですか?
0581132人目の素数さん
垢版 |
2019/07/31(水) 23:40:18.52ID:Tfohd+Nr
>>579,580

最小多項式については、ガロア理論の知識が必要になる。
多項式が整係数であるためには、ガロア群による作用で不変であることが必要。
そのためには、多項式は、根αの全ての置換を根として持たなければならない。

後は、本人に任せる。
0582132人目の素数さん
垢版 |
2019/08/01(木) 06:22:38.28ID:2P5/gzI6
αの最小多項式は

f(x) = Π[n=0,2] {(x-aω^n)^5 -B}     >>578

 = {(x-a)^5 -B} {(x-aω)^5 -B} {(x-aω~)^5 -B}

 = (x^3 -A)^5 + (x^5 -B)^3 - x^15 - 30AB {3x^7 + (9/2)Ax^4 + Bx^2 + (1/2)AAx},

ここに A=a^3, B=b^5.

A=2, B=3 のとき >>519
0583132人目の素数さん
垢版 |
2019/08/01(木) 06:32:21.49ID:gZoImdn4
以下の問題で、正弦定理を使って辺長a,bと角α、βが結びつくのは分かるのですが、真ん中の(1-cosβ)/(1-cosα)をどう結びつけるのかわかりません。
f(x)=(1-cosx)も考えてみたのですがさっぱりです。
どなたかよろしくお願いします。

△ABCにおいて、BC=a,CA=b、また∠A=α、∠B=β(0<α<β<π)である。
このとき、以下の3数の大小を比較せよ。
(b^2)/(a^2)、(1-cosβ)/(1-cosα)、(β^2)/(α^2)
0585132人目の素数さん
垢版 |
2019/08/01(木) 11:45:59.06ID:2P5/gzI6
b/a : √(1-cosβ) / √(1-cosα) : β/α

 = (sinβ) / (sinα) : sin(β/2) / sin(α/2) :  β / α,

 cos(x/2)、 sin(x/2)/x は 0<x<π では単調減少。
0586132人目の素数さん
垢版 |
2019/08/01(木) 14:36:56.81ID:2P5/gzI6
>>580

〔補題〕 対称多項式は基本対称式の多項式で表わせ、係数は整数である。

{a1,a2,a3} の基本対称式は
 s = a1 + a2 + a3,
 t = a1a2 + a2a3 + a3a1,
 u = a1・a2・a3,
である。

対称多項式 g(a1,a2,a3) は (a1)^c1・(a2)^c2・(a3)^c3 の形の項の和

k次の項 ( c1+c2+c3 = k) のみを含む対称式について示せばよい。

{c1,c2,c3} が同じ項をまとめて類とし、c1≧c2≧c3 で代表する。
これらのうち、c1が最大の類を選ぶ。
2つ以上あるときは、c2が大きい類を選ぶ。
c3 = k-c1-c2 で決まる。(これを最も強い類という)

対称式gはk次式
 s^(c1-c2)・t^(c2-c3)・u^c3
を含む。これを引き去った
 g(a1,a2,a3) - s^(c1-c2)・t^(c2-c3)・u^c3
も対称式であるが、gより弱い。
2番目に強い類についても同様とする。
これを繰り返すと強い類から消えてゆき、最後は0になる。(終)

http://mathtrain.jp/symfundamental
0588132人目の素数さん
垢版 |
2019/08/01(木) 16:32:00.11ID:2P5/gzI6
>>580

〔対称式の基本定理〕 
対称多項式は、基本対称式の多項式で表わせる。
整数係数の対称多項式は、整数係数の多項式で表わせる。(コーシー,1829)

(略証)
文字数nについての帰納法による。

{a1,a2,…,an} のk次の基本対称式をσ(k) とおく。
対称式F(a1,a2,…,an) は (a1)^c1・(a2)^c2・・・・(an)^cn の形の項の和。
a1,a2,…,an はn次方程式 Σ{k=0,n] (-1)^k・σ(k)・X^(n-k) = 0 の実根だから、
次数下げをすれば 0 ≦ c1,…,cn ≦ n-1 の項で表わせる。
ただし、係数にσ(1),…,σ(n)の整多項式が現れる。

F(a1,a2,…,an) をa1について整理すれば
 F(a1,a2,…,an) = Σ[k=0,n-1] g_k(a2,…,an;σ1,・・・,σn)a1^k

ところで、帰納法の仮定により a2,…,an の対称多項式は
その基本対称式 τ(1), ・・・,τ(n-1) により表わされるが、更に
 τ(1) = σ(1) - a1,
 τ(2) = σ(2) - a1・τ(1),
  ・・・・・・
 τ(n-1) = σ(n-1) - a1・τ(n-2)
とも表わせる。
a1 の次数下げをして
 F(a1,a2,…,an) = Σ[k=0,n-1] h_k(σ1,・・・,σn)・a1^k

左辺は対称式だから、n-1次方程式
 F(a1,a2,…,an) = Σ[k=0,n-1] h_k(σ1,・・・,σn)・X^k
はn個の実根{a1,a2,…an} をもつ。
∴ 右辺は定数であり
 F(a1,a2,…,an) = h_0(σ1,・・・,σn),
0589132人目の素数さん
垢版 |
2019/08/01(木) 23:36:10.01ID:gZoImdn4
x,y,zは自然数で、
x^3+y^3+z^3=3xyz…(*)
を満たす。

(1)このような自然数の組(x,y,z)は無数に存在することを示せ。

以下、x,y,zがx<y<zを満たす場合を考える。

(2)(x,y,z)=(a,b,c)が(*)を満たすとき、(x,y,z)=(ka,kb,kc)も(*)を満たすことを示せ。ただしkは自然数の定数である。

(3)(*)を満たす(x,y,z)は存在しないことを示せ。
0593132人目の素数さん
垢版 |
2019/08/02(金) 01:49:21.98ID:XnzEQzpb
絶対値が大きくなりそうな数字でくくるといわれているのですが
どのような数字でくくればよいかわかりません
あとこの問題に限らず式自体でくくると言うのはダメなんですか?
https://i.imgur.com/GM98Av3.jpg
0596132人目の素数さん
垢版 |
2019/08/02(金) 02:02:05.42ID:79abfNhp
>>593
式@ひく式Aの形の極限で、
√が絡んで、
どちらも無限大に大きくなっていき、
@/Aが極限で1に近づいていく場合、
ほぼ100%有理化で解きます。

この場合くくるだけでは解けないです。
@/Aが1に収束していくということは、
@ーAの式は、何でくくっても無限大×ゼロの形になるからです。

この問題の場合は、
(√nn+4n+5 )+ nを分母分子にかけると解けます。
0598132人目の素数さん
垢版 |
2019/08/02(金) 07:54:25.72ID:xQklGNZN
>>591 >>593
 [6] 極限 lim[n→∞] (√{n^2 +4n+5} - n) を求めよ。

(略解)
n^2 +4n +5 = (n+2)^2 + 1,
だから 
√{n^2 +4n +5} - (n+2) = 1/(√{n^2 +4n +5} + n+2) → 0 
0600132人目の素数さん
垢版 |
2019/08/02(金) 16:50:52.13ID:NHSqT3eG
お願いします

8÷2(2+2)

某スレでも話題となりましたが
息子に1だといっても納得しません。
16だったらどうしよう。
0601132人目の素数さん
垢版 |
2019/08/02(金) 16:56:54.82ID:rMxXAc/O
>>600
結合強度が割り算より高いというのはスラングでしょ
数学の規約では 2x == 2 * x
やりたいことはわかるけど
8/(2(2+2)) だよ
0602132人目の素数さん
垢版 |
2019/08/02(金) 17:01:50.15ID:ZG+E9AJj
16だったらどうしようも何も16

8÷2(2+2)=8/2x4=4x4=16
0603132人目の素数さん
垢版 |
2019/08/02(金) 17:05:13.38ID:qceWcMmq
>>600
a(式b)という形で書く場合、これはa*bとして不可分な一体の数と見なすので先に計算する。

8÷2×(2+2) これなら16という解釈もできるかも
0604132人目の素数さん
垢版 |
2019/08/02(金) 17:08:33.19ID:ZG+E9AJj
解釈もできるも何も
括弧で括っていないなら左から順番に計算する
0607132人目の素数さん
垢版 |
2019/08/02(金) 17:36:25.03ID:qceWcMmq
8/2*4

これだと解釈の余地があるでしょ
約束によって変わる、統一規則はない
0608132人目の素数さん
垢版 |
2019/08/02(金) 17:41:49.02ID:fov6qHUL
w(y,t)に関する偏微分方程式
∂w/∂t = ∂^2w/∂y^2
初期条件
w(y,0)=siny + cos2y
の解き方を教えて下さい。変数を分離してw=f(y)g(t)とおく方法でやったのですが上手く計算が進みませんでした。
宜しくお願いいたします
0609132人目の素数さん
垢版 |
2019/08/02(金) 17:42:41.44ID:rMxXAc/O
>>607
それリアル世界だと平行に割り算引くよね
プログラム表記が伝統的な数学表記ほど明示ではないというのは同意
0611132人目の素数さん
垢版 |
2019/08/02(金) 17:48:51.78ID:TVBuX0Ob
>>607
> 8/2*4
>
> これだと解釈の余地があるでしょ
ない。

> 約束によって変わる、統一規則はない
括弧がない以上左から順に演算をしょりする。
0615132人目の素数さん
垢版 |
2019/08/02(金) 18:21:09.68ID:rMxXAc/O
>約束によって変わる、統一規則はない
これで信用とれるなんて凄いな
0618132人目の素数さん
垢版 |
2019/08/02(金) 18:30:38.61ID:ZG+E9AJj
こんな基本的な規則も知らんのか
0619132人目の素数さん
垢版 |
2019/08/02(金) 18:34:47.28ID:rMxXAc/O
>>617
/ == ÷
もともと右側の文字は
100ちょっとしか文字がプログラムで使えなかったころでも
割り算として使うには不適だったから
数学の複数行の平行な割り算表記に似た / が割りあてられた。
簡潔だからリアルの方でも 日常で 1/2 とかつかうようになった

四則計算意外の新しい算術が定義されたのではなく
そのエイリアス
0620132人目の素数さん
垢版 |
2019/08/02(金) 18:41:29.38ID:ymfeDFxI
そういう紛らわしい書き方するひとはレベル低いですし、紛らわしい数式に白黒つけようとする人たちもまたレベルが低いのですよ
0621132人目の素数さん
垢版 |
2019/08/02(金) 18:45:12.42ID:ZG+E9AJj
8/2(2+2)でも8÷2(2+2)でも結果は16

wolfram
0622132人目の素数さん
垢版 |
2019/08/02(金) 18:46:05.94ID:NHSqT3eG
なんか荒れさせてすいません。
もう一度お願いします。

8÷2(2+2)

正解は1なのか16なのか教えてください。
0623132人目の素数さん
垢版 |
2019/08/02(金) 18:53:29.17ID:ymfeDFxI
>>622
式を書けば答えがあると考えるのがおかしいんですよ

人間誰でも書き損じしますよね?
その式は書き損じです
紛らわしい式を解釈しようと努力することになんの意味もありません
0624132人目の素数さん
垢版 |
2019/08/02(金) 18:58:47.64ID:j9108KwD
環の公理がなければ分配法則は成立しない
群の公理がなければ結合法則は成立しない

数式

8÷2(2+2)

に公理から独立して意味はない
0625132人目の素数さん
垢版 |
2019/08/02(金) 19:04:15.54ID:hQMwcXOh
(2)までは分かりました。(1)と同じことをすればいいだけでした。
(3)は(2)を使ってどうやって解くのでしょうか?

x,y,zは自然数で、
x^3+y^3+z^3=3xyz…(*)
を満たす。

(1)このような自然数の組(x,y,z)は無数に存在することを示せ。

以下、x,y,zがx<y<zを満たす場合を考える。

(2)(x,y,z)=(a,b,c)が(*)を満たすとき、(x,y,z)=(ka,kb,kc)も(*)を満たすことを示せ。ただしkは自然数の定数である。

(3)(*)を満たす(x,y,z)は存在しないことを示せ。
0626132人目の素数さん
垢版 |
2019/08/02(金) 19:04:23.09ID:ymfeDFxI
公理云々は関係ないですよ

記法の問題ですから

数学ではなく国語の問題です
0627132人目の素数さん
垢版 |
2019/08/02(金) 19:05:57.32ID:ymfeDFxI
>>622
結局ですね、こうやってネットで何年もネタにされてるってことがこの式が不親切な書き方だってことの一番の証拠なんですよ

ちゃんとした理由付けができるならとっくに解決してるはずですからね

してないということは、誰も正しい計算方法決めてないし、だからこういう書き方は紛らわしいだけで書くべきではないということなんです
0628132人目の素数さん
垢版 |
2019/08/02(金) 19:13:45.58ID:j9108KwD
環の定義を公理化せずに分配法則を使ったらイカサマ
群も同様
0629132人目の素数さん
垢版 |
2019/08/02(金) 19:19:41.97ID:ymfeDFxI
環論や群論の教科書に2(2+2)とかいう記法の定義なんてありますか?
ないですよね

むしろ、ちゃんとしたそういう本には演算子の記号ちゃんと書くはずですけど
0635132人目の素数さん
垢版 |
2019/08/02(金) 20:34:22.71ID:rMxXAc/O
>>632
これが初めてだな
2x^2

だが特定の分野では保守性の低い書きかたはすべきではない
と答える
0637132人目の素数さん
垢版 |
2019/08/02(金) 20:45:28.41ID:rMxXAc/O
>>636
中学のときには nxn 行の逆行列も導いてたし
n個の点を通る n-1次の式も導いてた
そして >158 を見つけていたのは中学のときだ

たぶん日本全1だったんじゃないか?
0638132人目の素数さん
垢版 |
2019/08/02(金) 20:56:07.28ID:rMxXAc/O
省略したスマートな書きかたしたかったら
最初に書いておけばOK

S=Sin(x)
S(f(x))=Sin(f(x))
C=Cos(x)
C(f(x))=Cos(f(x)) のように
0641132人目の素数さん
垢版 |
2019/08/02(金) 21:22:53.26ID:ymfeDFxI
>>638
ではそのように書くべきですね
8÷2(2+2)

こういう紛らわしい書き方はいただけませんね
0642132人目の素数さん
垢版 |
2019/08/02(金) 21:35:47.17ID:kgxBUnqk
いつまでも論争が続くのは教科書にそのような表記が存在しないからだと思う
8÷2×(2+2)の意味ならそのように書かれるか、あるいは
 8
──(2+2)
 2
のように書かれ、
8÷{2×(2+2)}の意味ならそのように書かれるか、あるいは
  8
───
2(2+2)
のように書かれているんじゃないだろうか
÷と2(2+2)が混在する表記は用いられていないのでは?
混在する表記は用いられることがなく、定義されていないと解するべきと思う
0643イナ ◆/7jUdUKiSM
垢版 |
2019/08/02(金) 21:53:34.68ID:/ORP8+ab
/_/_/人人_/_/_/_
/_/_(_)_)/_/_/_
/_/_(_(_)/_前>>610
/_/_(^_^)) __/__/__
/_/__(っ┳υ__/_/_
/_/◎゙υ┫_/__/__/__
/_/_/_◎゙/_/_/_/_/_/キコキコ……♪_/_/_/_/_/_/_/_/_/_/_/_/_/_/_/_/_/_/_/_>>634三角錘の体積マジレスしたのにハンネがヒドラになったよ。/_/_/_/_/_/_/_/_
0644132人目の素数さん
垢版 |
2019/08/02(金) 22:18:25.39ID:rMxXAc/O
数学だとあまり保守性に言及されることは少ないけど
個人的には見た目そっくりな変数が使われることがあるのはきつい
(2,z)(a,ギリシャのアルファ)(v,ギリシャ?のそれ)

z を自分が書くときは 左上から右下に向かってななめに線をいれてる
0645132人目の素数さん
垢版 |
2019/08/02(金) 22:27:09.26ID:j2W8IxaZ
>>644
それだとsと区別つかなくないかい?
0647132人目の素数さん
垢版 |
2019/08/02(金) 22:46:09.72ID:TVBuX0Ob
>>646

>>644
> z を自分が書くときは 左上から右下に向かってななめに線をいれてる

「線をいれてる」とは線を追加している、という意味ね。
0649132人目の素数さん
垢版 |
2019/08/03(土) 00:12:03.03ID:h0Vp//K+
自然数とは、順位を表す数であると言えるよね。

例えば、順位に−2位や0位、1.5位などはないから

じゃあ、整数ってどんな数かと言われた時に、何て言えばいいかな?

・テストの点数に使える数のことである

と、最初は思ったのね。0点というのもあるし、−7点という表現は前回より7点下がったみたいな意味で使えるから。

でも、テストの結果を表す時に、全ての問題を間違えたとしても最低は0点だから、−7点とかになることはないから、よくわからなくなってきて

整数って、どういう時に使える数ですか?
0650132人目の素数さん
垢版 |
2019/08/03(土) 00:18:23.14ID:JSq4ZtF+
>>649
気温に使える数かな?

0度もマイナス7度もあるし

ただ、気温は10.7度とか小数が使えるからな
0651132人目の素数さん
垢版 |
2019/08/03(土) 00:42:40.39ID:/LnLCRO5
>>649
1 の+- で表現できる数である
数えられる物質のカウントに使える
-200万の赤字とか
50万の黒字など
0652132人目の素数さん
垢版 |
2019/08/03(土) 01:34:42.59ID:xCAdnJNv
>>625 , >>648
{ (x+y+z)/3 }^2 ≦ (xx + yy + zz)/3 (∵ Jensen不等式 (等号成立条件は x=y=z ) )
 = { (x+y+z)^2 - 2 (xy+yz+zx) } / 3
0 ≦ (x+y+z)^2 - 3(xy+yz+zx)
0 ≦ {(x+y+z)^3 - 3(xy+yz+zx)(x+y+z) + 3xyz } - 3xyz (∵ 0 < x+y+z )
0 ≦ x^3 + y^3 + z^3 - 3xyz
∴ x^3 + y^3 + z^3 ≧ 3xyz
x<y<z の時に等号が成立することはない。

(2)を使い道は分からん
0653132人目の素数さん
垢版 |
2019/08/03(土) 02:02:50.39ID:pz8hzzuL
面白い問題の面白い解法おもいついたぜ!的なやつじゃないの?
ただの相加相乗平均の等号成立条件の話にしかみえんのだが。
0654132人目の素数さん
垢版 |
2019/08/03(土) 02:08:42.10ID:rk4XK8I7
頭悪いやつが問題出して喜んでるのをみんな避けてるの、わからないかなぁ………
アスペはこれだから
0656132人目の素数さん
垢版 |
2019/08/03(土) 02:48:56.34ID:6+XGsIBj
等号成立条件は x=y=z と仮定する
中略
ゆえに
x<y<z の時に等号が成立することはない

等号成立条件を仮定しているのに
その等号が成立しない条件を示す

これに意味はない
イコールで書けるものがあると仮定しているのにもかかわらず
イコールでない場合を明示してどうする
既にイコールで書けることは仮定している
その条件が外れた場合はイコールを除いた不等号に決まっている
0657132人目の素数さん
垢版 |
2019/08/03(土) 07:29:17.57ID:QekS+DF3
すいません
8÷2(2+2)
のものです。皆さんレスありがとうございます。

「式が曖昧なので答えが確定しない」まで理解できました。

ちょうど息子がその辺を習っているのでネットで見かけた話題を一緒に考えてみました。
結果1と16に答えが割れました。そして
「どちらが正解なんだろう?」との疑問をこちらのスレの皆さんに解いていただこうと書き込みした次第です。

学習塾の先生がおっしゃるには
「÷の記号が本来数学にない」
とのことですが私には難しすぎて理解できませんでした。

ともあれ荒れさせてしまいすみませんでした。
この問題はこの辺で終了たさせてください。
レスを下さった皆さん重ねてありがとうございました。
良い夏休みをおすごしください。
0659132人目の素数さん
垢版 |
2019/08/03(土) 09:14:41.17ID:OkprqPHK
>>658
608です
よろしければどのように解くのか教えて頂けませんか?
変数分離の形の波動方程式なら解けるのですが、答えを見るに初めて見るパターンです。
0661132人目の素数さん
垢版 |
2019/08/03(土) 10:28:18.77ID:iBloBzZq
n次実正方行列PがP^2=Pを満たしR^nがIm PとIm Pの補空間の直和で表されるときPは対称行列であることを示せ。
という問題の解き方を教えてください。
0664132人目の素数さん
垢版 |
2019/08/03(土) 11:08:17.79ID:iBloBzZq
>>661
何度もすみません。問題を読み間違えてました。
正しくは

n次実正方行列PがP^2=Pを満たしIm PとKer Pが直交し、Im P∩Ker P={0}のときPは対称行列であることを示せ。

という問題です。
スレ汚して申しわけないです。
0665132人目の素数さん
垢版 |
2019/08/03(土) 11:56:42.28ID:MhbiabIn
>>659 >>660
w(y,t) = sin(ay+b)g(t)
とおく。 与式から
 ∂w/∂t = ∂^2 w /(∂y)^2 = -aa・w,
sin(ax+b) で割ると
 g '(t) = -aa・g(t),
これを解いて
 g(t) = exp(-aat),
 w(y,t) = sin(ay+b) exp(-aat),
これを重ね合わせる。
0666132人目の素数さん
垢版 |
2019/08/03(土) 11:58:42.84ID:xCAdnJNv
>>664
ImPの任意要素 v には 適当な R^nの要素 x が存在して v = P.x と書ける.
条件より P.v = PP.x = P.x = 1.v
つまり ImP は 固有値 1 の固有空間、
そして KerP は 固有値 0 の固有空間である.

条件よりImPとKerPは直交していて、かつ n=dim(ImP)+ dim(KerP) なので
R^n = ImP + KerP
つまり適当な直交基底に対して Pは diag(1,1,...,1, 0,...,0) と書ける. (diagonal matrix: 対角行列)
P = R.diag(1,1,...,1, 0,...,0).R^t  (Rは回転行列)
よって P^t = P
Pは対称行列である.
0667132人目の素数さん
垢版 |
2019/08/03(土) 12:29:24.51ID:xCAdnJNv
(追加)
> つまり適当な直交基底に対して...
この"正規"直交基底を f1, f2, ..., fn として, R := ( f1, f2, ..., fn ) とする.
R^t. R = ( f1, f2, ..., fn )^t . ( f1, f2, ..., fn ) = E {単位行列}
より Rは直交行列である. ( "回転行列" と言うのは det R = 1 の時だけらしい)
R^t. P.R = R^t . ( 1.e_1,1.e_2, ..., 0.e_n ) = diag( 1,1,..., 0 ) ( =: Λ とする )
よって P = R Λ R^t
0668132人目の素数さん
垢版 |
2019/08/03(土) 13:29:33.54ID:OkprqPHK
>>665
> w(y,t) = sin(ay+b)g(t)
> とおく。
これは熱伝導方程式を解くときはこうするのがお決まりみたいなものなのですか?それとも初期条件を見て今回はこのように置いたということなのでしょうか?
0669132人目の素数さん
垢版 |
2019/08/03(土) 16:12:02.01ID:MhbiabIn
お決まりと言ってもよいでしょう。
初期条件をフーリエ展開して
 w(y,0) = Σ[a] sin(ay+b),
となったら
 w(y,t) = Σ[a] sin(ay+b) exp(-aat),
0670132人目の素数さん
垢版 |
2019/08/03(土) 16:15:25.62ID:MhbiabIn
お決まりと言ってもよいでしょう。
初期条件をフーリエ展開して
 w(y,0) = Σ[a] C_a sin(ay+b),
となったら
 w(y,t) = Σ[a] C_a sin(ay+b) exp(-aat),
0671132人目の素数さん
垢版 |
2019/08/03(土) 21:15:25.82ID:JCVg1kWI
eを分母が1桁の有理数で近似するとき、最も精度が良いものを求めなさい。
という問題で、解説が一切なく答えのみ書かれていました。このような、無理数を有理数で精度良く近似する時は、何を使うのでしょうか。
道具として、連分数とテイラー展開は何となく思いつきますが、「分母n桁」と「最も精度が良い」をどうクリアしたら良いか分かりません。
ご指導お願いいたします。
0673132人目の素数さん
垢版 |
2019/08/03(土) 21:24:46.00ID:JCVg1kWI
>>672
数学検定1級の過去問題集にあった問題で、単に整数と書かれていました。
数値としてはe=2.718281828...のみが与えられています。
1次試験のため計算過程や論述は不要ですが、どうして最小の分数が19/7に決まるのか全く書かれておらず理論が分かりません。
0674132人目の素数さん
垢版 |
2019/08/03(土) 21:46:32.22ID:/LnLCRO5
>>673
たった9個なんてローラーでいいよ
en=m | n= 1..9, e=2.718281828
9個大隊計算していくと
n=3のときにm=7.15...と整数から0.15ずれてて有力だけど
n=7のときに19.0...とかなり整数に近いから
0680132人目の素数さん
垢版 |
2019/08/03(土) 23:48:05.99ID:/LnLCRO5
じゃあ絞ればいい
2.7 に整数 3を書けると
大体 8は安産だから
3, 6,7 9,程度の4か所をチェックすればいい

半減
0681132人目の素数さん
垢版 |
2019/08/04(日) 00:06:33.11ID:1of8rU+C
皆様レスありがとうございます。
高々9通り試せば良いことがよく分かりました。
実は、システマティックな方法があると思って問の前半のみを書いたのですが、後半として「分母が2桁の場合」もありました。3桁以降は問われていません。
これも10から99までの総当りで何とかなるのでしょうか。
0684132人目の素数さん
垢版 |
2019/08/04(日) 00:36:34.55ID:1of8rU+C
>>682
ありがとうございます。
あたりをつけていくことになるんですね。
eの近似値を求めるとあったので、連分数やテイラー展開を使うのかと思いましたが、eに全く関係ない問題だとは思いませんでした。
0686132人目の素数さん
垢版 |
2019/08/04(日) 01:01:20.83ID:ohb0XEhv
いや、ダメだ。
いま電卓片手にfarey数列の理論使ってやってみたけど電卓使っても恐ろしい手間かかる。
こんなのパソコンでも持ち込まなきゃ分母二桁の最良近似なんてとても無理だ。
0687132人目の素数さん
垢版 |
2019/08/04(日) 03:46:56.84ID:mMVmWw0w
近似表示したい値=e=2.718281828...
この値は、(2/1,3/1) にある
分母同士、分子同士を加えてできる 5/2 で この範囲を二つに分け、範囲(2/1,5/2) と 範囲(5/2,3/1) のどちらにeがあるか? 答え (5/2,3/1)
この範囲を、8/3で二分し、(5/2,8/3)と(8/3,3/1)のどちらにeが有るか? 答え (8/3,3/1)
11/4を加え、(8/3,11/4)と(11/4,3/1)のどちら? 答え(8/3,11/4)
19/7を加え、(8/3,19/7),(19/7,11/4)のどちら? 答え(19/7,11/4)
次加えるべき分数は、30/11となるが、(19/7,11/4)の範囲に、11より分母の小さい分数は無いから、
分母一桁という条件では、19/7 か 11/4 が候補
0688132人目の素数さん
垢版 |
2019/08/04(日) 07:18:16.17ID:nU8H17FN
2<e<3 なので
(e-2)/(3-e) = 2.54965 ≒ 5/2
と近似すれば出る。

分母2桁でもおk
0689132人目の素数さん
垢版 |
2019/08/04(日) 08:03:05.15ID:nU8H17FN
>>684
eの連分数展開は循環しないものの一定の規則性を持つ。

e = 2 + 1/(1 + 1/(2 + 1/(1 + 1/(1 + 1/(4 +・・・・)))))

 = 2 + [1, 2, 1, 1, 4, 1, 1, 6, 1, 1, 8, 1, 1, 10, ...]

初めの方は 5/2, 11/4, 19/7, 68/25, 106/39, 193/71, 299/110, 1457/536, 2721/1001, 4178/1537, 25946/9545, 49171/18089, ・・・・

http://oeis.org/A003417

分母1桁 19/7 = 2.7143
分母2桁 193/71 = 2.71831
分母3桁 1457/536 = 2.7182836
0690132人目の素数さん
垢版 |
2019/08/04(日) 08:30:44.05ID:ohb0XEhv
>>689
それはeに収束するだけで、分母が××以下の有理数近似で最良のものをを与えてくれるわけじゃないのでは?
正規連分数だからいけるの?
0692132人目の素数さん
垢版 |
2019/08/04(日) 10:32:36.37ID:vzU8Q2kc
https://i.imgur.com/MhKf38u.jpg

この(3)で
5つの連続した整数の和は5の倍数になるとあるのですが、

これって−2 −1 0 1 2

だった場合0になってしまうから、5の倍数にならなくないですか?
0694132人目の素数さん
垢版 |
2019/08/04(日) 11:15:00.28ID:91fIbE6U
>>689
連分数計算で得られる x/y は分母 y 以下の有理数の中では e の最良近似だけど、
例えば 193/71 が 分母 99 以下の有理数の中で最良近似かどうかは要チェックではないの?

効率考えずにプログラム走らせると
最良近似の数列は
... , [193/71], 685/252, 878/323, 1071/394, [1264/465], [1457/536], [2721/1001], ...
のように続く。(カギ括弧のは連分数展開で現れる有理数)
確かに ?/72,..., ?/99 の間で記録更新は生じないのだけど、それ手計算ですぐ確認できますかね...?
0695132人目の素数さん
垢版 |
2019/08/04(日) 11:45:15.27ID:mMVmWw0w
連分数展開を利用した分数は、真の値に振動しながら近づきます。
単に、前回の近似分数より精度がよいものが順に現れるのでは無く、
誤差の正負が逆転するという条件も伴っています。

従って、「分母がこれこれ以下」のような条件を満たすものを、連分数展開で現れる物だけを
チェックしていては、見逃すものが現れ得ますね。
0696132人目の素数さん
垢版 |
2019/08/04(日) 11:50:02.71ID:ueI8kLfK
>>694
wikiからの受け売りで未確認だけどfarey数列でできる有理数近似はどうも正則連分数展開で得られる有理数列と同じになるくさい。
ただし問題としてはネイピア数の正則連分数展開というさほどメジャーでない話を知ってないといかんというのが。
レポート提出の課題とかならまだしも数学検定とかの問題としてはちょっとダメな感じが。
0697132人目の素数さん
垢版 |
2019/08/04(日) 12:58:10.06ID:E1Hw19rZ
実数αに対して ||α|| = min_[nは整数] |α-n| を、実数αの整数との近さを表すものとして、
つまり ||en|| がわりと小さくなる(厳密には ||en||/n が最も小さくなる)正整数 n≦100 を探せってことだよね?

例えば n≦7 までの値が
||e||~0.282、||2e||~0.437、||3e||~0.155、||4e||~0.127、||5e||~0.409、||6e||~0.310、||7e||~0.028
等と求められれば、n≦6 の時の ||en||≧0.126 がわかるから、
三角不等式を使えば整数s,t について
||(7s+t)e|| ≧ abs(||7se|| - ||te||) ~ abs(||0.028t|| - ||te||)
とか使って探索範囲減らせるし、
次の(局所的に最良な)更新値である ||39e||~0.013 や、
n≦31 の時の ||en||≧0.028 等がわかれば同様に
||(39s+t)e|| ≧ abs(||0.013s|| + ||te||)
から次の最良値の目処も立つだろうし
電卓あれば比較的簡単に出せそうな気が(というか電卓アリだったのか数検…)
0699132人目の素数さん
垢版 |
2019/08/04(日) 14:10:40.93ID:nU8H17FN
〔ディオファントス近似に関するディリクレの定理〕

無理数αに対して
 | x/y - α | < 1/(y^2),
を満たす有理数 x/y が存在する。

x/y を求める方法の一つが連分数計算です^^
0701132人目の素数さん
垢版 |
2019/08/04(日) 22:05:14.67ID:mMVmWw0w
e = 2 + [1, 2, 1, 1, 4, 1, 1, 6, 1, 1, 8, 1, 1, 10, ...] に対し、
2 + [1]
2 + [1, 1] = 5/2
2 + [1, 2] = 8/3
2 + [1, 2, 1] = 11/4
2 + [1, 2, 1, 1] = 19/7
2 + [1, 2, 1, 1, 1] = 30/11
2 + [1, 2, 1, 1, 2] = 49/18
2 + [1, 2, 1, 1, 3] = 68/25
2 + [1, 2, 1, 1, 4] = 87/32
2 + [1, 2, 1, 1, 4, 1] = 106/39
2 + [1, 2, 1, 1, 4, 1, 1] = 193/71
2 + [1, 2, 1, 1, 4, 1, 1, 1] = 299/110
2 + [1, 2, 1, 1, 4, 1, 1, 2] = 492/181
...
2 + [1, 2, 1, 1, 4, 1, 1, 6] = 1264/465

のように、連分数表現において、最後の数字を一つ大きくするか、“,1”を加えることによって、
分母が大きくなる数列ができます。(前者の場合は、必ず、真の値に近づきます。)
これを使えば、「分母三桁以下」などの条件に当てはまるものを、確実に捕らえることができますね。
0702132人目の素数さん
垢版 |
2019/08/04(日) 22:20:25.72ID:ohb0XEhv
>>701

> のように、連分数表現において、最後の数字を一つ大きくするか、“,1”を加えることによって、
> 分母が大きくなる数列ができます。(前者の場合は、必ず、真の値に近づきます。)
そんなの正則でない連分数でも成立するじゃん?
分母が××以下ってのを調べるなら正則なやつじゃないとダメなんじゃないの?
0703132人目の素数さん
垢版 |
2019/08/04(日) 22:41:38.45ID:mMVmWw0w
701 の方法は、>>687のファレイ分数の方法を、連分数的な表現に言い直しただけです。
従って、ファレイ分数で知られている事実、
「分数 a/b と c/d が隣り合っているとき(|ad-bc|=1の時)、区間(a/b,c/d)の中の分数の中で、分母が最小なのは、(a+c)/(b+d)」
が使えます。確実に分母を大きくしつつ、真値に近い分数の探索に使えます。
0704132人目の素数さん
垢版 |
2019/08/04(日) 22:51:44.68ID:ohb0XEhv
>>703
そう、だから連分数を打ち切って得られる近似分数が隣接するfarey列の2数で挟んだもので得られるものと同じになれば十分だけどそれは正則な連分数でないと言えない。
しかし>>701で言ってる事は別に正則な連分数でなくても成立してしまってる。
もしかしたら正則でない連分数から作る方法があるかもしれないけど少なくとも>>701の説明で連分数の打ち切り近似で分母がより小さい有理数で近似できないかの説明にはなってないと思う。
0705132人目の素数さん
垢版 |
2019/08/04(日) 23:15:05.58ID:mMVmWw0w
>>704
・求めたい値が、区間 (a/b,c/d) 内にある。
・a/b と c/d は隣り合っている。つまり、|ad-bc|=1。

区間(a/b,c/d)内のすべての有理数の分母は(b+d)以上

閉区間[a/b,c/d] の中で、分母が(b+d)以下の分数で、求めたい値に近いものは、
a/b、(a+c)/(b+d)、c/d のいずれか

という論理ですが、問題ありますか?


前提条件になっているというか、順次範囲を狭め、成立させ続けている条件とも言えますが、
隣り合う分数で挟まれた区間 (a/b,c/d) 内に真値があるという時点で、
分母が b や d より小さい分数全て、この範囲の外にあるので、真値により近い分数としては、
a/b や c/d に敗北しているのです。
真値に近い候補としては、a/b 、c/d および、これらから「ファレイ操作(?)」によって作られる分数
に限ってよいのです。
0706132人目の素数さん
垢版 |
2019/08/04(日) 23:27:14.64ID:mMVmWw0w
こういった方がいいのかな?

適当に選んだ、二つの分数で定まる区間 (a/b,c/d) 内になら、
分母が、b+d 、あるいは、b や d より小さい分数は存在し得るが、
a/bとc/dが隣り合っているなら、この区間内の分数の分母は、一つの例外を除いて、全て(b+d)より大きい。
0707132人目の素数さん
垢版 |
2019/08/04(日) 23:41:56.86ID:ohb0XEhv
いや、隣接するfarey数列で挟めばその間に分母が小さい有理数が存在しないのはいいですよ。
問題は正則ではないかもしれない連分数の打ち切り近似で得られる上下からの挟み討ちは必ずしも隣接farey列の2数とは限らないですよね?
それは正則連分数なら確かに正しいし正則連分数が持ってる打ち切り近似の持っている性質を利用すれば証明できますが、
>>701で抑えている性質は正則ではない連分数でも持っている性質なので、そこから打ち切り近似が隣接farey列の2数とは言えないのではないかという疑問です。
0708132人目の素数さん
垢版 |
2019/08/04(日) 23:49:23.27ID:91fIbE6U
>>705, >>706
自分あまり理解できていなくてなんなんですけど、

> 最良近似の数列は
> ... , [193/71], 685/252, 878/323, 1071/394, [1264/465], [1457/536], [2721/1001], ...
> のように続く。(カギ括弧のは連分数展開で現れる有理数)

そのファレイ操作で 193/71 の次に来る 685/252 は捕捉できるのですか?
そうでないなら、193/71 の次に来る分母が3桁なのは偶然ラッキーなだけのように見えるのです。
2桁だ3桁だのは単に10進数に依拠した表示ですから。
0709132人目の素数さん
垢版 |
2019/08/05(月) 00:02:03.64ID:mWIRrJL5
>>707
出発時点で使った、2/1 と 3/1は 隣り合っている分数です。
分母同士、分子同士の和で作られる 5/2 は、2/1 とも、3/1とも隣り合っています。
つまり、隣り合った分数 a/b と c/d から作られる (a+c)/(b+d)は、a/bともc/dとも隣り合っています。

隣り合った分数a/bとc/dで挟まれた閉区間を、(a+c)/(b+d) で二分し、
[a/b,(a+c)/(b+d)] あるいは、[(a+c)/(b+d),c/d]に分け、注目する区間をどちらかに限定し、さらに、
同じような操作を繰り返した場合でも、常に注目する区間は、隣り合った分数で挟まれた区間となります。
何も問題はありません。
0712132人目の素数さん
垢版 |
2019/08/05(月) 00:35:00.14ID:Vj1uEW7V
>>708
横レスです。
一般に正の項からなる連分数の打ち切り近似からなる有理数列はその無限連分数の収束値を上下から交互に挟んで収束していく。
そしてその連分数が正則であるならその2数は"隣接する"二数になる。
例えば2721/1001<e<1457/536
は分母が1001以下の有理数の最良の上下からの近似を与える。
よって例えば分母が5桁の最良の近似が欲しければ正則連分数があれば打ち切り近似の分母が大きいが6桁を超えるところまで計算して、その中で分母が5桁の一番近いものが答え。
正則連分数が綺麗に並ぶもの(ランバート連分数とか)ならうまくいく。
でも超幾何関数の隣接漸化式から作ったような正則ではない連分数だと多分うまくいかない。
うまくいかない具体的な例は知らないけど。
0714132人目の素数さん
垢版 |
2019/08/05(月) 01:32:08.29ID:mWIRrJL5
>>712
その方法は、>>694 さんが計算してくれたところの、カギ括弧付きのもののみを候補とする方法だと思います。
恣意的な設定ですが、「分母が400以下」だと、本当の答えは、1071/394 ですが、
>>712の方法だと、193/71 の次が、1264/465 となるので、193/71と 答えてしまうのではないでしょうか?

e = [2; 1, 2, 1, 1, 4, 1, 1, 6, 1, 1, 8, 1, 1, 10, ...] に対し、
[2; 1, 2, 1, 1, 4, 1, 1]=193/71
[2; 1, 2, 1, 1, 4, 1, 1, 6]=1264/465
だから、193/71とするのでは無く、
[2; 1, 2, 1, 1, 4, 1, 1, k]として、k=1,2,3,4,5 のようなものも候補に加えればよいというのが、私の主張で、
実際、k=5で、1071/394 が得られます。
0715132人目の素数さん
垢版 |
2019/08/05(月) 02:36:06.22ID:Vj1uEW7V
あれ?おかしいな?
193/71と1264/465は隣接してるから
193×465-1264×71の間には分母が465より小さい有理数ないはずなんだけど?
0716132人目の素数さん
垢版 |
2019/08/05(月) 02:39:41.07ID:Vj1uEW7V
あ、文章ミスった
> 193×465-1264×71=1だからその2数の間には分母が465より小さい有理数ないはずなんだけど?
0717132人目の素数さん
垢版 |
2019/08/05(月) 02:46:58.24ID:Vj1uEW7V
あ、いや、今わかった。
確かにその通り。
正則連分数ですらダメなのか。
じゃ正則連分数表示知ってても結局役に立たず、地道にfarey列で挟んでいくしかないのか。
これはダメだ。
電卓でもなけりゃ絶対無理だwwww
0718132人目の素数さん
垢版 |
2019/08/05(月) 12:18:52.72ID:mWIRrJL5
与えられた有理数 n/m に対し、 |ad-bc|=1,n=a+c,m=b+d ,a/b < c/dを満たす唯一の整数解 (a,b,c,d) を
[a/b,c/d] のように表記し、>>701の内容の最右辺に添えました。

[2;1,1] = 5/2 : [2/1,3/1]
[2;1,2] = 8/3 : [5/2,3/1]
[2;1,2,1] = 11/4 : [8/3,3/1]
[2;1,2,1,1] = 19/7 : [8/3,11/4]
[2;1,2,1,1,1] = 30/11 : [19/7,11/4]
[2;1,2,1,1,2] = 49/18 : [19/7,30/11]
[2;1,2,1,1,3] = 68/25 : [19/7,49/18]
[2;1,2,1,1,4] = 87/32 : [19/7,68/25]
[2;1,2,1,1,4,1] = 106/39 : [19/7,87/32]
[2;1,2,1,1,4,1,1] = 193/71 : [106/39,87/32]
[2;1,2,1,1,4,1,1,1] = 299/110 : [106/39,193/71]
[2;1,2,1,1,4,1,1,2] = 492/181 : [299/110,193/71]
[2;1,2,1,1,4,1,1,3] = 685/252 : [492/181,193/71]
[2;1,2,1,1,4,1,1,4] = 878/323 : [685/252,193/71]
[2;1,2,1,1,4,1,1,5] = 1071/394 : [878/323,193/71]
[2;1,2,1,1,4,1,1,6] = 1264/465 : [1071/394,193/71]
[2;1,2,1,1,4,1,1,6,1] = 1457/536 : [1264/465,193/71]
0719132人目の素数さん
垢版 |
2019/08/05(月) 12:19:26.77ID:mWIRrJL5
[2;1,2,1,1,4,1,1,6,1,1] = 2721/1001 : [1264/465,1457/536]
[2;1,2,1,1,4,1,1,6,1,1,1] = 4178/1537 : [2721/1001,1457/536]
[2;1,2,1,1,4,1,1,6,1,1,2] = 6899/2538 : [2721/1001,4178/1537]
[2;1,2,1,1,4,1,1,6,1,1,3] = 9620/3539 : [2721/1001,6899/2538]
[2;1,2,1,1,4,1,1,6,1,1,4] = 12341/4540 : [2721/1001,9620/3539]
[2;1,2,1,1,4,1,1,6,1,1,5] = 15062/5541 : [2721/1001,12341/4540]
[2;1,2,1,1,4,1,1,6,1,1,6] = 17783/6542 : [2721/1001,15062/5541]
[2;1,2,1,1,4,1,1,6,1,1,7] = 20504/7543 : [2721/1001,17783/6542]
[2;1,2,1,1,4,1,1,6,1,1,8] = 23225/8544 : [2721/1001,20504/7543]
[2;1,2,1,1,4,1,1,6,1,1,8,1] = 25946/9545 : [2721/1001,23225/8544]
[2;1,2,1,1,4,1,1,6,1,1,8,1,1] = 49171/18089 : [25946/9545,23225/8544]

お気づきだと思いますが、これは、[a/b,c/d] を閉区間を表示したものと見なし、「ファレイ操作」によって、
区間を狭める一連の操作が行われたものとも見なせます。

>>じゃ正則連分数表示知ってても結局役に立たず、地道にfarey列で挟んでいくしかないのか。
そんな事はありません。連分数表示を途中で打ち切ってしまうのでは無く、「最後の数字を一つ大きくするか、“,1”を加える」操作の
道しるべと見なせば、両操作は、同じ事を行っている事が確認できます。
連分数表示を知っていれば、ファレイ操作において、いちいちどちらの区間に入っているかチェックする必要が無くなるとも言えます。
0720132人目の素数さん
垢版 |
2019/08/05(月) 13:47:39.29ID:tw19LQ24
確率論勉強しててスチルチェス積分ってのがでて来て特に詳しい説明もないまま流されたんだけど
これは知っておく必要がある知識なの?
0723132人目の素数さん
垢版 |
2019/08/05(月) 14:36:12.57ID:Vj1uEW7V
>>719
だって>>714さんの指摘にはどう答えるの?
打ち切り近似の中に解があるとは限らないから、打ち切り近似の中のたとえば分母400以下の最良表示がわかったとして、実際の分母が400以下の最良近似1071/394を求めるにはどうするの?
0724132人目の素数さん
垢版 |
2019/08/05(月) 15:32:50.97ID:mWIRrJL5
>>723
何を言っているのか、困惑しているのですが、>>714>>719も私の発言です。
最良近似1071/394は、
>> [2;1,2,1,1,4,1,1,5] = 1071/394 : [878/323,193/71]
として、候補に挙がってます。
このような候補を見逃さないように、最後の数字を一つづつ変化させているのです。
(この操作が、「ファレイ操作」に対応しています。)

普通、e = [2;1,2,1,1,4,1,1,6,1,1,8,1,1,10, ...] に対し、
[2;1],[2;1,2],[2;1,2,1],[2;1,2,1,1],[2;1,2,1,1,4],[2;1,2,1,1,4,1],[2;1,2,1,1,4,1,1],...
のような形で、打ち切り部分を、伸ばしていきますが、>>718のような形で、伸ばすことにより、
分母をじわじわ大きくしながら、真値に近い候補を探ることになります。

一度遠回しな表現はしたのですが、再度補足しておくと、>>718らに書かれている分数は、
分母は常に大きくはなっていますが、常に真値に近づいているわけではありません。
誤差の正負が反転する辺りで、誤差が大きくなります。
その後、誤差が減っていくのですが、正負が反転する当たりで、再度誤差がふくれます。
この繰り返しで、真値に近づいていきます。
0726132人目の素数さん
垢版 |
2019/08/05(月) 16:30:01.15ID:Vj1uEW7V
>>724
すいません。やはりわかりません。
具体的に>>723のような設問では連分数展開の打ち切り近似を利用した解法の記述ではどうなるんですか?

k=1,2,3,4,5 のようなものも候補に加えればよいというのが、私の主張で、
> 実際、k=5で、1071/394 が得られます。

これはkを上げていったものの中に必ず解があるのは何故ですか?
0727132人目の素数さん
垢版 |
2019/08/05(月) 16:32:03.58ID:Vj1uEW7V
あ、間違った。
この中に解があり、しかもk=5で探索が終了するのは何故ですか?
0732132人目の素数さん
垢版 |
2019/08/05(月) 19:55:15.71ID:p5oV6jpW
a,bを自然数とする。ax+by=1を満たす整数x,yが存在するとき,
a,bは互いに素であることを証明せよ。

背理法を用いる。
[a,bは互いに素でないとすると
  a=a'g, b=b'g (a', b'は互いに素, gはa,bの最大公約数で, g>1)
とおける。
ax+by=1に代入して整理すると g(a'x+b'y)=1
g, a'x+b'yは整数であるから…@ (g, a'x+b'y)=(1, 1),(-1, -1)
これは g>1に矛盾する。]

@で、あえてg>1を伏せて、無理やり証明するものなのですか?
0733イナ ◆/7jUdUKiSM
垢版 |
2019/08/05(月) 19:57:07.80ID:hcaEFDbJ
>>730
与式=(x^2-7x+11)^(x^2-13x+42)=1
x^2-7x+11=1のとき、
x=2,5――@
x^2-13x+42=20,12
与式=1^20=1^12=1
x^2-13x+42=0のとき、
x=(13±√169-168)/2
=7,6――A
与式=(7^2-7・7+11)^0=11^0=1
@Aより、x=2,5,6,7
0734132人目の素数さん
垢版 |
2019/08/05(月) 19:57:33.35ID:mWIRrJL5
>>726
 >>687で示した方法、つまり、ターゲットの範囲を、二つに分け、そのどちらに、目的の値が
存在するかをチェックし、存在する方の範囲を、新たなターゲット範囲として二つに分け、...
を繰り返していけば、目的の値を含む範囲をどんどん小さくしていけることは、自明だと思います。

そして、この時、範囲の両端として、「隣り合う有理数」を用い、さらに、範囲を二つに分ける分岐点として、
「中間数」(=(分子同士の和)/(分母同士の和))を用いれば、範囲を規定する分数の分母は常に大きくなる
という感じの便利な性質があり、「分母は○○以下」のような条件下での探索には、丁度よいことは、よろしいでしょうか?
そして、この方法で、確実に目的を達せられることもよろしいと思います。

このような方法を「ファレイ操作」を使った方法と勝手に呼んでいました。
これに対し、連分数の途中打ち切りを用いる方法は、「ファレイ操作」の方法の無駄な部分を切り落とした方法と言えます。

ファレイ操作では、ターゲット区間を二つに分け、上位の範囲か下位の範囲か検討しながら、進んでいきますが、
上位、下位、上位、下位、上位、下位、と反転が、立て続けに起こる場合もあれば、
上位、上位、上位、...と同じ側が、何度も続くような場合があります。
前者として、代表的なのが、黄金比であり、後者の好例がπです。

πの近似値を求めるべく、ファレイの方法をとると、ある段階で、[333/106,22/7] となり、次は、[333/106,355/113]
なのですが、この上限 355/113 は、πに近いため、この後、何度も何度も、上位ばかりが選ばれることになります。
0735132人目の素数さん
垢版 |
2019/08/05(月) 19:58:15.61ID:mWIRrJL5
πの連分数表記は、[3;7,15,1,292,1,1,1,2,1,3,1,14,...]ですが、
[3;7,15,1]=355/113 で 、[3;7,15,1,292]=103993/33102 と続きます。
下限 333/106 、上限 355/113 、次の近似値 103993/33102 と、連分数表記に現れる 292 には、
103993/33102 = (333+355*292)/(106+113*292)
という関係があります。数直線上に二点、333/106 と 355/113 を 取れば、この二点を 292:1 に分けた点が、103993/33102と言えます。

つまり、ファレイ操作では[333/106,355/113]の後、292回位連続で、上位範囲を選択して、103993/33102 に到達する事になります。
一方、連分数では、その回数分の数値292は、「値」として使うだけで、計算回数は一度増えるだけです。
これを上部で「無駄な部分を切り落とした方法」と評したのです。

連分数表記が既知なら、値を求める方法としては優秀です。ファレイの方法は、分母の値の評価が秀逸です。
だから、例えば、eの値を「分母5桁以下で、e最良近似を求めよ」という問題に対し、おそらく、最も効率のよい方法は、
それぞれの優秀なところを組み合わる次のような方法と思われます。

[2;1,2,1,1,4,1,1,6,1,1,8,1,1] = 49171/18089 = e + 2.766*10^(-10)
[2;1,2,1,1,4,1,1,6,1,1,8,1,1,10] = 517656/190435 = e -1.364*10^(-11)
を見つけて、この辺に目星をつけ、最後の10を一つづつ減らしながら、条件に当てはまるもの、
[2;1,2,1,1,4,1,1,6,1,1,8,1,1,5] = 271801/99990 = e - 2.7622*10^(-10) を候補として見つける。
あくまで、候補で、今回はより精度の高いものが見つかりましたが、条件によっては、連分数で計算したものがそのまま、
最良近似値となることもあります。
0738132人目の素数さん
垢版 |
2019/08/05(月) 20:40:57.33ID:MJnrEz8V
>>730みたいな問題、初出は98年の学習院大の入試だと思うけど、どういう形式で出たんだろう?
解の数が分かってる穴埋めならともかく、完答制だとしたら入試としては悪問な気がする
0739132人目の素数さん
垢版 |
2019/08/05(月) 20:55:08.99ID:Vj1uEW7V
>>734
そのあたりのfarey数列の話は大丈夫です。
問題は例えば、では

> [2; 1, 2, 1, 1, 4, 1, 1, k]として、k=1,2,3,4,5 のようなものも候補に加えればよいというのが、私の主張で、
> 実際、k=5で、1071/394 が得られます。

の部分です。
なぜその形の連分数表示を持つものの中に解があると断じることができるのでしょう?
そしてその探索をk=5で終了できるのは何故ですか?
そこの理屈がわからないのです。
0740132人目の素数さん
垢版 |
2019/08/05(月) 21:06:02.32ID:zGiOQCLN
>>730
 (xx-7x+11)^(xx-13x+42) = 1,
あるいは
 (xx-7x+11)^{(x-6)(x-7)} = 1,

>>731 によれば
Case 1:
 xx-7x+11 = 1
 (x-2)(x-5) = 0,

Case 2:
 (x-6)(x-7) = 0 (xx-7x+11≠0)

Case 3:
 xx-7x+11 = -1 ((x-6)(x-7) = 偶数)
 (x-3)(x-4) = 0,

∴ x = 2,3,4,5,6,7
0741132人目の素数さん
垢版 |
2019/08/05(月) 21:26:16.87ID:mWIRrJL5
>>739
具体例を示して説明を与えました。よく読んでいただきたいのですが、
「この辺に目星をつけ、最後の10を一つづつ減らしながら、」と書いています。
[2;1,2,1,1,4,1,1,6,1,1,8,1,1,10]、[2;1,2,1,1,4,1,1,6,1,1,8,1,1,9]、[2;1,2,1,1,4,1,1,6,1,1,8,1,1,8]、
[2;1,2,1,1,4,1,1,6,1,1,8,1,1,7]、[2;1,2,1,1,4,1,1,6,1,1,8,1,1,6]、[2;1,2,1,1,4,1,1,6,1,1,8,1,1,5]
と最後の数字を減じながら、分母が5桁に収まる[2;1,2,1,1,4,1,1,6,1,1,8,1,1,5]を見つけたのです。

そしてこれが、「解」だ等とは断じていません。あくまで候補だと。そして、その候補も、精度が、
[2;1,2,1,1,4,1,1,6,1,1,8,1,1]より悪かったら、捨てることになります。
なお、[2;1,2,1,1,4,1,1,6,1,1,8,1,1,4]から、[2;1,2,1,1,4,1,1,6,1,1,8,1,1,1]までは、
[2;1,2,1,1,4,1,1,6,1,1,8,1,1,5]より、精度が低いことが確実なので、チェックする必要はありません。

>> そしてその探索をk=5で終了できるのは何故ですか?
k=5で終了したのではありません。k=6に相当するものより、分母の小さいものを探すので、
真っ先にk=5をチェックしたのです。
条件によっては、k=4,3,...とkの値を減じてチェックしますが、その必要はありませんでした。。

>> なぜその形の連分数表示を持つものの中に解があると断じることができるのでしょう?
繰り返しになりますが、断じて等いません。候補として扱います。理由は、着実に分母を大きくしながら真値を含む
範囲を狭めるファレイの方法を、逆向きに評価していることを指摘すれば、納得して頂けるでしょうか。
0742イナ ◆/7jUdUKiSM
垢版 |
2019/08/05(月) 21:38:18.87ID:hcaEFDbJ
>>733追加。
与式=(x^2-7x+11)^(x^2-13x+42)=1
x^2-7x+11=1のとき、
x=2,5――@
x^2-13x+42=20,12
与式=1^20=1^12=1
x^2-13x+42=0のとき、
x=(13±√169-168)/2
=7,6――A
与式=(7^2-7・7+11)^0=11^0=1
x^2-7x+11=-1のとき、
(x-3)(x-4)=0
x=3,4――B
x=3のとき、x^2-13x+42=9-39+42=90(偶数)
(-1)^90=1
x=4のとき、x^2-13x+42=16-52+42=6(偶数)

(-1)^6=1
@ABより、
x=2,3,4,5,6,7
0744132人目の素数さん
垢版 |
2019/08/05(月) 23:42:37.81ID:OaQOexTD
難しいです
0746 【吉】
垢版 |
2019/08/06(火) 00:44:05.58ID:rNWPzR7y
>>742俺の嗅覚を甘く見るなよ。
>>743(-.-)クンクン...



√(1250^2+125^3)=1875
n=125
0747132人目の素数さん
垢版 |
2019/08/06(火) 01:37:38.34ID:KO1VR19N
O中心で弧の長さR扇型において、周上を動点PがAからBまで動く時に、弧APの長さがxのときのAQの長さを、xによる関数としてf(x)とします。(AP>AQ)
この時、AQが通る面積(斜線部分)は、f(x)をどのように定積分すれば求められますか?
Integral[0,R] f(x) dx
で良いのでしょうか?
簡単に言えば極座標において、極からの距離が角度で早くこの長さに依存するバージョンです

https://i.imgur.com/4JNGDJS.jpg
0750イナ ◆/7jUdUKiSM
垢版 |
2019/08/06(火) 10:04:08.34ID:rNWPzR7y
>>746
>>747違うかもやけど、
扇形を三角形と見て、
長さxの弧を細かく刻んだΔxを0からRまで足し集めたものを底辺、
弧の長さxに対する斜線部分の径f(x)を高さとすると、
三角形=(1/2)×底辺×高さ
f(x)の積分関数をF(x)+Cとすると、
斜線部分の面積
=(1/2)∫[x=0→R]f(x)dx
={F(R)-F(0)}/2
0751132人目の素数さん
垢版 |
2019/08/06(火) 15:14:37.74ID:E3F8Wrnc
0<x<∞で定義された関数f(x)で、
(C1)恒等的にf(x)>0
(C2)lim[x to 0]f(x)=+∞
(C3)lim[x to ∞]f(x)=0
(C4)いたる所微分可能
を満たすものを考える。
y=f(x)のグラフをxy平面に描き、各xにおいてf(x)の接線を引く。そのx軸との交点をA、y軸との交点をBとし、線分ABの長さをL(x)とする。

質問ですが、L(x)の最小値はf(x)が上記(C1)〜(C4)を満たすならば必ず存在しますか?
最大値が存在しない場合もあることは確認しました。よろしくお願いします。
0753132人目の素数さん
垢版 |
2019/08/06(火) 15:51:55.22ID:OX0mJe6w
>>725
725です
P(k1T_n <θ<k2T_n)=P(θ/k2<T_n<θ/k1)=P(T_n<θ/k1)-P(T_n<θ/k2)=1/(k1)^n - 1/(k2)^nと変形し
1/(k1)^n - 1/(k2)^n=1-αの条件のもとk2-k1が最小になるようにk1,k2を決定すればよい
ここまではできたのですがここで詰まってしまいました。それとも方針が間違っているのでしょうか?どなたかご教授願いたいです。
0754132人目の素数さん
垢版 |
2019/08/06(火) 16:04:38.31ID:SonbT+8g
そりゃl(x)は0にはならないけど
下限が0になるようなfは作れるだろ
0755132人目の素数さん
垢版 |
2019/08/06(火) 16:17:31.25ID:Q/NX+4C9
「恒等的にf(x)>0」って、ポエムのニューウェーブか何か?
0758132人目の素数さん
垢版 |
2019/08/06(火) 19:22:29.26ID:E3F8Wrnc
>>755
正しい言い方を教えて下さい。
あと下限でなくて最小値が存在するようにできませんか
0764132人目の素数さん
垢版 |
2019/08/06(火) 23:48:08.50ID:GwDuEPZI
恒等的にf(x)>0って言い方、違和感覚える人は確かにいるだろうね
(x+1)^2-x^2-2x>0は「恒等的に成立」って感じするけど、x^2+1>0は「全てのxに対して」って感じがするわ
前者は恒等式っぽさが、後者は方程式っぽさが否めない
0766 【大吉】
垢版 |
2019/08/07(水) 00:41:16.79ID:J+Sbr8E0
>>750
>>760
0≦x≦y≦2をxy平面に図示すると、2辺が2の直角二等辺三角形が描ける。
頂点は(0,0),(0,2),(2,2)
z=3x^2-xy-2x+y+1
=(3x^2-2x+1)-(x-1)y
=(3x-1)(x-1)-(x-1)y
=(x-1)(3x-1-y)
xy平面にx=1とy=3x-1のグラフを描き、この2本の直線上の点(x,y)はz=0だ。
直角二等辺三角形内のいくつかの点をにらみながら、zの値を調べると、
(x,y)→z=(x-1)(3x-1-y)
(0,0)→z=1
(0,1)→z=2
(0,2)→z=3
(2,2)→z=3
(1,2)→z=0
(1,1)→z=0
(1/2,1/2)→z=0
(2/3,2/3)→z=-1/9
(3/4,3/4)→z=-1/8
(4/5,4/5)→z=-3/25
現在の暫定最小値z=-1/8
0767132人目の素数さん
垢版 |
2019/08/07(水) 02:52:35.43ID:GSn/o782
>>751
g(x) = -(1-cosx)/(x^3) - 1/(x^100)
G(x) = -∫_(x,∞)g(t)dt
f(x) = xG(x)
とおけば、fは4つの性質を全て満たすし、Lの最小値も存在しない。
(ただし、接線とx軸の交点が存在しない場合はL=∞と定める)

(C1)(C4)は明らか。オーダーを調べることで(C2)(C3)もわかる。
点(a, f(a)) における接線の方程式は
y = (ag(a)+G(a))(x-a) + f(a)
= (ag(a)+G(a))x - (a^2)g(a)
であるから、点Aのy座標は常に正であり、更に a→+∞ の時 0 に収束することがわかる。
また、a=2πn (nは正の整数)の時、点Bのx座標は
(a^2)g(a)/(ag(a)+G(a))
= (a^(-98))/(a^(-99)+G(a))
= O(1/(n^96))
となるから、n→+∞の時 0 に収束する。
以上から L(x) は常に正であり liminf(x→+∞)L(x)=0 であるから、L(x)の最小値は存在しない。
0768132人目の素数さん
垢版 |
2019/08/07(水) 03:06:45.67ID:oo9D9QmR
>>760
2・3 x,y が 0≦x≦y≦2 を満たして変化するとき、
 z = 3xx -xy -2x +y +1 の最小値を求めよ。

>>762
z = 3xx-2x+1 + (1-x)y,

(i) 0≦x≦1 なら、
 関数zはyに関して単調増加。
 よって z ≧ 2xx -x +1 = 2(x -1/4)^2 + 7/8 ≧ 7/8,

(ii) 1≦x≦2 なら、
 関数zはyに関して単調減少。
 よって z ≧ 3xx -4x +3 = (x-1)(3x-1) +2 ≧ 2,

よって、 (x,y) = (1/4,1/4) としたときの z=7/8 が最小値。
0769132人目の素数さん
垢版 |
2019/08/07(水) 10:18:32.95ID:0OdoHy4h
n次正方行列Aは、i+j=kのときa_ij=kである(1≤i≤n,1≤j≤n)。
ただし、Aのi行j列成分をa_ijと書く。
Aが正則行列かどうか判定せよ。
0771イナ ◆/7jUdUKiSM
垢版 |
2019/08/07(水) 10:35:58.05ID:J+Sbr8E0
>>766未確認。
>>760
0≦x≦y≦2をxy平面に図示すると、2辺が2の直角二等辺三角形が描ける。
頂点は(0,0),(0,2),(2,2)
z=3x^2-xy-2x+y+1
=(3x^2-2x+1)-(x-1)y
=(3x-1)(x-1)-(x-1)y
=(x-1)(3x-1-y)
xy平面にx=1とy=3x-1のグラフを描き、この2本の直線上の点(x,y)はz=0だ。
直角二等辺三角形内のいくつかの点をにらみながら、zの値を調べた。
(x,y)→z=(x-1)(3x-1-y)
(0,0)→z=1
(0,1)→z=2
(0,2)→z=3
(2,2)→z=3
(1,2)→z=0
(1,1)→z=0
(1/2,1/2)→z=0
(2/3,2/3)→z=-1/9
(3/4,3/4)→z=(-1/4)(9/4-1-3/4)
=(-9+4+3)/16
=-1/8
(4/5,4/5)→z=-3/25>-1/8
最小値の候補z=-1/8だが、
x=y=3/4を
z=3x^2-xy-2x+y+1
に代入すると、
z=3(3/4)^2-(3/4)^2-3/4+1
=2(9/16)+1/4
=11/8>0あわない。
計算間違いの可能性がある。
微分=0のような決定力がほしい。
0772132人目の素数さん
垢版 |
2019/08/07(水) 10:56:35.52ID:baxjFqiL
問題として面白いかどうかとか、質問スレに問題出題してどうなんとかもあるけど、そもそも問題の文章すらホントに数学真面目に勉強した事あるのか問い詰めたくなるよな。>>769のkもどう束縛されてるのかわけわからん。
流石に任意のkについてだろうけど、それならa_ij=i+jと書けばいいだけなのに。
0773132人目の素数さん
垢版 |
2019/08/07(水) 12:26:45.79ID:8Lm802uC
>>767
> 以上から L(x) は常に正であり liminf(x→+∞)L(x)=0 であるから、L(x)の最小値は存在しない。

L(x) って AB の長さなんだから、常に x < L(x) になるのと違うの?
そんなんで liminf(x→+∞)L(x)=0 になる?
0774132人目の素数さん
垢版 |
2019/08/07(水) 12:30:58.08ID:8Lm802uC
今のは取り消す。
勘違いしてた。正の傾きなら原点O近傍に2交点がありえるね。
0775132人目の素数さん
垢版 |
2019/08/07(水) 13:09:50.85ID:rMaz3Ehs
n次正方行列Aのi行j列成分a_ijは、a_ij=i+jを満たす(1≤i≤n,1≤j≤n)。
Aが正則行列かどうか判定せよ。
0776132人目の素数さん
垢版 |
2019/08/07(水) 13:35:03.05ID:8Lm802uC
>>775
n=1, 2 だと正則行列 (簡単に確かめられる)

n≧3 では 非正則行列
a_{i}{j} -2 a_{i+1}{j} + a_{i+2}{j} = (i+j) - 2.(i+j+1) + (i+j+2) = 0
よって連続した3行は必ず線型従属になるから。
0781イナ ◆/7jUdUKiSM
垢版 |
2019/08/07(水) 17:04:04.93ID:J+Sbr8E0
>>771
0≦x≦y≦2――@
をxy平面に図示すると、2辺が2の直角二等辺三角形が描ける。
頂点は(0,0),(0,2),(2,2)
z=3x^2-xy-2x+y+1
=(3x^2-2x+1)-(x-1)y
3x^2-2x+1の因数分解がまちごうてた。
z=3x^2-xy-2x+y+1
=3x^2-(y+2)x+y+1
z=3{x^2-(y+2)x/3}+y+1
=3{x-(y+2)/6}^2+y+1-{(y+2)/6)}^2
放物線の軸x=(y+2)/6が@のどの範囲にあるかで場合分けする。
(y+2)/6≦0のとき、
zの最小値はx=0のとき、
z=y+1
y=0のとき、
z=1――A
0≦(y+2)/6≦xのとき、
-2≦y≦6x-2
zの最小値はx=(y+2)/6のとき、
z=6x-2+1-3{(6x-2+2)/6}^2
=-y^2/12+2y/3+4/3
=-(1/12)(y-4)^2+4/3+4/3
=-(1/12)(y-4)^2+8/3
y=2のとき、
z=-(1/12)(2-4)^2+8/3
=-1/3+8/3
=7/3――B
x≦(y+2)/6≦yのとき、
6x-2≦y,2/5≦y
zの最小値はx=(y+2)/6のとき、
z=y+1-(1/12)(y+2)^2
=-(1/12)(y^2+4y+4-12y-12)
=-(1/12)(y^2-8y)+2/3
=-(1/12)(y-4)^2+2/3+16/12
=-(1/12)(y-4)^2+2
y=2/5のとき、
z=-(1/12)(2/5-4)^2+2
=-(1/12)(-18/5)^2+2
なんかさっきと違う。

y≦(y+2)/6≦2のとき、
と2≦(y+2)/6のとき、
まだあと2範囲あるけど計算が怪しいんで、ここはzの値の分布状況を勘案し、候補として@の範囲を突っ切る直線、
x-(y+2)/6=0すなわちy=6x-2の線上の点を求める。
zの最小値は、
x=y=2/5のとき、
z=3(2/5)^2-(2/5)(2/5)-2(2/5)-2(2/5)+(2/5)+1
=3(4/25)-(4/25)-2/5+1
=8/25-10/25+1
=23/25――C
@ABCより、
zの最小値=23/25
0783イナ ◆/7jUdUKiSM
垢版 |
2019/08/07(水) 17:58:35.26ID:J+Sbr8E0
>>781
(x,y)=(2,2)のとき、
z=7でおそらく最大だと思う。
(x,y)=(0,0),(1/2,1/2)がともにz=1だから、
(x,y)=(1/4,1/4)がたしかに怪しい。
(x,y)=(1/2,1)は、z=5/4
(x,y)=(0,1),(1,1)はともに、z=2
(x,y)=(1,2)のとき、z=4
0784イナ ◆/7jUdUKiSM
垢版 |
2019/08/07(水) 18:11:54.59ID:J+Sbr8E0
>>783
23/25は惜しかった。
7/8のほうがたしかにちょっとだけ小さい。
(x,y)=(1/4,1/4)で来るとはな。
0785132人目の素数さん
垢版 |
2019/08/07(水) 21:48:01.02ID:wVATmZYn
重積分について
√(9-x^2-y^2)をD:x^2+y^2<=3x
を計算する時、極座標変換を用いて0<=r<=3cosθ,-π/2<=θ<=π/2とし計算していくと答えは9πになりました。
しかし、対称性を用いて0<=θ<=2/πとして計算すると答えは9π-12となります。wolframを使って検算するとこちらの方が正しいようです。何故このような差異が生まれるのでしょうか?ただの計算ミスなのでしょうか?
0789イナ ◆/7jUdUKiSM
垢版 |
2019/08/07(水) 23:49:24.98ID:J+Sbr8E0
>>784
そうだ、上に凸か下に凹かで場合分けするとか言ってたな。

ぜんぶ無視してすみませんでした。
0791132人目の素数さん
垢版 |
2019/08/08(木) 00:41:36.97ID:wb6dvUO+
>>788
(1)
これは a[1]=1, a[2]=2, .., a[n]=n 以降は任意の並べ方: 2^n * (2n-n)!
  = (2^n) n! 通り
( 2^n は n枚の色選択肢 )

(2)
(条件なし)全ての並べ方: (2n)! 通り

「a[1]=k, a[2]={k以下}」を満たす並べ方: (2 + 4(k-1) ) * (2n-2)! 通り
(同じ数字2枚並びは異色2通り, 異なる数字2枚は4通り に注意)
よって X=1 となる並べ方: Σ[k=1,n] ... = 2n(n-1)(2n-2)! 通り

「a[1]={k未満}, a[2]=k, a[3]={k以下} 」を満たす並べ方:
  ( 2^2* (k-1) + 2^2 * (k-1) + 2^3 * {(k-1)^2 - (k-1)} ) * (2n-3)! = 8(k-1)^2 (2n-3)! 通り
よって X=2 となる並べ方: Σ[k=1,n] ... = (4/3) n(n-1)(2n-1)(2n-3)! 通り

以上より X≧3 となる並べ方:
(2n)! - 2n(n-1)(2n-2)! - (4/3) n(n-1)(2n-1)(2n-3)!
 = (4/3)n(nn-1)(2n-3)! 通り

もうちょっとマシな解法ありそう
0793132人目の素数さん
垢版 |
2019/08/08(木) 02:10:21.90ID:wb6dvUO+
>>792
そうですね。Σ{k=3,n} (2^3) C{k-1,2}*(2*n-3)! = ... = (4/3)n(n-1)(n-2)(2n-3)!
( n=3 で (1) と一致します )
>>791 の後半は忘れてください...
0794132人目の素数さん
垢版 |
2019/08/08(木) 17:15:44.81ID:tXG8ZJrb
(C1)恒等的にf(x)>0
0795132人目の素数さん
垢版 |
2019/08/08(木) 23:23:06.22ID:+3npddFa
奇数の完全数が存在するかしないか教えてけろ
0796132人目の素数さん
垢版 |
2019/08/09(金) 00:02:46.44ID:3F4yBL2M
奇数の完全数が存在するかどうかまだ分かってないって凄いよね。なんかすぐわかりそうなもんだけど
0797 【大吉】
垢版 |
2019/08/09(金) 00:47:34.30ID:USDKoaPD
奇数の完全数なんてないだろ。前>>789ないな〜ぃ。そんなのあるわけな〜ぃ!! あはははは……
 ̄ ̄]/\______
__/\/ ∩∩∩∩ /|
 ̄ ̄\/ ((^o`^o^))/ |
 ̄ ̄ ̄ っцυ⌒υ |
 ̄ ̄ ̄ ̄~UU~υυ| /|
□ □ □ □ | /|__|__|__|__|__|__|__|__|__|__|__|__|
0799132人目の素数さん
垢版 |
2019/08/09(金) 09:19:55.92ID:7q6wi2Y+
「y=f(x) と y=f⁻¹(x) が異なる2点で交わる」と「y=f⁻¹(x)とy=x が異なる2点で交わる」が同値であるという記述を見かけたのですが、
・連続関数である
・y=x 上に異なる複数(2つ以上)の共有点をもつ
場合に限って同値でしょうか?
0800132人目の素数さん
垢版 |
2019/08/09(金) 10:06:51.57ID:FHE2hgaT
A=Bを示す時に
A=Z(0), B=Z(n) になるような関数 Z(n) を定めてZ(n)=Z(n+1) を示すのって、なんで良いのでしょうか?
0801132人目の素数さん
垢版 |
2019/08/09(金) 10:25:15.84ID:i8gyLeAY
>>799
P=(fとf⁻¹が異なる2つ以上の点で交わる)
Q=(f⁻¹とy=x が異なる2つ以上の点で交わる)
R=(fは連続)
S=(fとf⁻¹がy=x 上に異なる2つ以上の共有点をもつ)
として、主張は「任意のfについて(P⇔Q)⇒(SかつR)」と読めるんですが合ってますか?
0802132人目の素数さん
垢版 |
2019/08/09(金) 14:12:51.36ID:FH4W9/Is
>>768
762です
書き取り助かる、ありがとう
0803132人目の素数さん
垢版 |
2019/08/09(金) 15:03:35.29ID:xtFjtHJo
∫[0 to a] exp(-x^2) dx =I[a]とし、
lim[a to infty] I[a] = Iとおく。
tをI[t] = (π/4)*Iなる実数とするとき、n<t<n+1を満たす自然数nがただ1つ存在する(この事実は示さずともよい)。
nを求めよ。
0806132人目の素数さん
垢版 |
2019/08/09(金) 23:40:41.86ID:ZLWTEHzz
>>803

部分積分により
∫exp(-xx) dx = -(1/2x)exp(-xx) - ∫(1/2xx)exp(-xx) dx
 = -(1/2x)exp(-xx) + {1/(4x^3)}exp(-xx) + ∫{3/(4x^4)}exp(-xx) dx
 = ・・・・

∴ I[a] = I - [1/(2a) - 1/(4a^3) + 3/(8a^5) - ・・・・] exp(-aa)}

t = 0.87752700406522
n = 0,
0807132人目の素数さん
垢版 |
2019/08/10(土) 09:59:40.38ID:sxGno790
数列a[n]=1/nの各項に対して以下の操作(T)を行い、新たな数列b[n]を作る。
Σ[k=1 to n] a[k] = A[n]
Σ[k=1 to n] b[k] = B[n]
とおくとき、極限
lim[n to infty] A[n]/B[n]
が収束することを示せ。

(T)
a[n]=1/nの小数点以下に並ぶ数字のうち、奇数であるものを全て0に置き換えてできる実数をb[n]とする。例えば
a[1]=1=1.00..., b[1]=1
a[2]=1/2=0.500..., b[2]=0.00...=0
a[3]=1/3=0.333..., b[3]=0.00...=0
a[7]=1/7=0.142857..., b[7]=0.042800...
0810132人目の素数さん
垢版 |
2019/08/10(土) 11:32:27.74ID:/OEEP8qd
>>807
b[n] の小数点第m位以外の位を全て0にした数を b_m[n] とおけば、
lim_(m→∞) Σ_(n=1,∞) b_m[n]
= lim_(m→∞) Σ_(n=1,10^m) b_m[n]
= lim_(m→∞) (10^(-m))*Σ_(k=1,(10^m)/2) Σ_( 1/(2k+1)≦n/(10^m)<1/(2k) ) ((2k)mod10)
= Σ_(k=1,∞) ((2k)mod10)/(2k(2k+1))
= ∫_(0,1) (2x^2+4x^4+6x^6+8x^8)(1-x)/(1-x^10) dx
≒ 0.652562.
この値を α とおけば α_m := Σ_(n=1,10^m) b_m[n] は lim_(m→∞) α_m = α を満たすので
lim_(m→∞) Σ_(n=1,m) (α_n)/m = α.
これらから、正の整数 n に対して 10^m≦n<10^(m+1) を満たす整数kをとれば
Σ_(k=1,n) b[k]
= 1 + Σ_(k=1,n) Σ_(j=1,∞) b_j[k]
= 1 + (Σ_(j=1,m-1) α_j) + Σ_(j=m,∞) Σ_(k=1,n) b_j[k]
= 1 + (1+o(1))mα + Σ_(j=m,∞) (θ_j)*8n/(10^j)
(ただし各 j に対して θ_j は 0 以上 1 以下の実数)
= 1 + (1+o(1))mα + 72θn/(10^(m+1))
(θ は 0 以上 1 以下の実数)
= (1+o(1))αlogn.
よって、極限値は存在。ついでにその極限値はα.
0811132人目の素数さん
垢版 |
2019/08/10(土) 12:04:49.63ID:oX3OQU5P
>>803

(π/4)I = 0.7854・I < 0.8427・I = erf(1)・I = I(1) =∫[0,1] exp(-xx) dx,

左辺を ∫[0,t] exp(-xx) dx とすると、
t < 1,
n = 0.
0812132人目の素数さん
垢版 |
2019/08/10(土) 12:06:44.88ID:/OEEP8qd
>>810
二つ訂正

> = ∫_(0,1) (2x^2+4x^4+6x^6+8x^8)(1-x)/(1-x^10) dx
> ≒ 0.652562.

= ∫_(0,1) (2x+4x^3+6x^5+8x^7)(1-x)/(1-x^10) dx
≒ 0.956876.


> = (1+o(1))αlogn.
> よって、極限値は存在。ついでにその極限値はα.

= (1+o(1))αlog_10(n).
よって、極限値は存在。ついでにその極限値はα/ln10. ただしlnは自然対数。
0815132人目の素数さん
垢版 |
2019/08/10(土) 17:24:28.51ID:oX3OQU5P
>>807
nが1桁のとき
 b[1] = 1,    A[1] = 1,   B[1] = 1,
 b[2] = 0,    A[2] = 1.5   B[2] = 1,
 b[3] = 0,    A[3] = 11/6,  B[3] = 1,
 b[4] = 0.2    A[4] = 25/12, B[4] = 1.2
 b[5] = 0.2    A[5] = 137/60, B[5] = 1.4
 b[6] = 0.066667 A[6] = 2.45  B[6] = 1.46667
 b[7] = 0.042800 A[7] = 2.5929 B[7] = 1.50947
 b[8] = 0.02   A[8] = 2.7179 B[8] = 1.52947
 b[9] = 0,    A[9] = 2.8290 B[9] = 1.52947

nがk桁のとき
 (1/9)10^k < n ≦ (1/8)10^k  8 10^(-k) ≦ b[n] < 9 10^(-k)
 (1/7)10^k < n < (1/6)10^k  6 10^(-k) ≦ b[n] < 7 10^(-k)
 (1/5)10^k < n ≦ (1/4)10^k  4 10^(-k) ≦ b[n] < 5 10^(-k)
 (1/3)10^k < n ≦ (1/2)10^k  2 10^(-k) ≦ b[n] < 3 10^(-k)
 その他            0 ≦ b[n] < 10^(-k)

10^(k-1) ≦ n < 10^k の範囲で見ると
 僊 〜 log(10) = 2.302585
 248/315 = 0.78727 < 傳 < 1.68727
0816132人目の素数さん
垢版 |
2019/08/10(土) 21:38:49.55ID:oX3OQU5P
>>812 (上)

(1-x^10)/(1-x) = 1+x+x^2+・・・・+x^9
 = (1+x)(1+x^2+x^4+x^6+x^8)
 = (1+x)(1-x+x^2-x^3+x^4)(1+x+x^2+x^3+x^4),
より
(2x+4x^3+6x^5+8x^7)(1-x)/(1-x^10)
= (2x+4x^3+6x^5+8x^7)/{(1+x^2+x^4+x^6+x^8)(1+x)}
= (4-2x+6x^2-2x^3+6x^4+4x^6+4x^7)/(1+x^2+x^4+x^6+x^8) - 4/(1+x)
= (4-3x+6x^2-4x^3+6x^4-3x^5+4x^6)/(1+x^2+x^4+x-6+x-8)
 + (x+2x^3+3x^5+4x^7)/(1+x^2+x^4+x^6+x^8) + F(x)
= (3xx-4x+3)/2(1-x+x^2-x^3+x^4) + 5(1+xx)/(1+x+x^2+x^3+x^4) + E(x) + F(x)
= (φφ/2)/(xx-φx+1) + {1/(2φφ)}/(xx+x/φ+1) + (1+φ/2)/(xx+φx+1)
+ {(3-φ)/2}/(xx-x/φ+1) + E(x) + F(x)
= A(x) + B(x) + C(x)+ D(x) + E(x) + F(x),

∫ A(x) dx = {(φφ-2)/φ√(4-φφ)}arctan{(2x-φ)/√(4-φφ)},
∫ B(x) dx = {(2+√5)/√(5+2√5)}arctan{(2φx+1)/√(4φφ-1)},
∫ C(x) dx = (φ/2)√{2(5+√5)}arctan{(2x+φ)/√(4-φφ)},
∫ D(x) dx = {(2+φ)/φ√(4φφ-1)}arctan{(2φx-1)/√(4φφ-1)},
∫ E(x) dx = (1/2)log(1+x^2+x^4+x^6+x^8) = (1/2)log{(1-x+x^2-x^3+x^4)(1+x+x^2+x^3+x^4)},
∫ F(x) dx = -∫4/(1+x) dx = -4log(1+x),
0817132人目の素数さん
垢版 |
2019/08/11(日) 02:47:16.52ID:GctloD3X
↑の訂正

= (3-4x+3xx)/2(1-x+x^2-x^3+x^4) + 5(1+xx)/2(1+x+x^2+x^3+x^4) + E(x) + F(x)
= (1-φ/2)/(1-φx+xx) + (φφ/2)/(1+x/φ+xx) + (1+φ/2)/(1+φx+xx)
+ {(3-φ)/2}/(1-x/φ+xx) + E(x) + F(x)
= A(x) + B(x) + C(x)+ D(x) + E(x) + F(x),

A = (π/5)2√(1-2/√5) = 0.408306132277
B = (π/5)√(1+2/√5) = 0.864806265977
C = (π/5)(1/2)√(5+2√5) = 0.966882799046
D = (π/5)(3/2)√(5-2√5) = 0.684750200550
E = (1/2)log(5) = 0.804718956217
F = -4log(2) =-2.772588722240

合計 0.956875631828
0818132人目の素数さん
垢版 |
2019/08/11(日) 03:08:58.57ID:GctloD3X
↑ つまり
A(x) = (1-φ/2)/(1-φx+xx),
B(x) = (φφ/2)/(1+x/φ+xx),
C(x) = (1+φ/2)/(1+φx+xx),
D(x) = {(3-φ)/2}/(1-x/φ+xx),
E(x) = (x+2x^3+3x^5+4x^7)/(1+x^2+x^4+x^6+x^8),
F(x) = -4/(1+x),
φ = (1+√5)/2 = 1.618034・・・・
0819132人目の素数さん
垢版 |
2019/08/11(日) 06:54:43.95ID:3IN5mwD+
>>807
よく見たら A[n]/B[n] の極限かこれ…
>>812 で求めたのは逆数 B[n]/A[n] の極限値であることに注意。

>>817 明示的に表したんか…すごい根気
0821132人目の素数さん
垢版 |
2019/08/11(日) 19:47:44.31ID:24szNkNg
順序で反対称律が成り立てばサイクルが無いって本に書いてあるんだけど
a≦b , b≦aのときa→b→aのサイクルあるよね
おかしくない?
0822132人目の素数さん
垢版 |
2019/08/11(日) 19:52:59.38ID:F9EOH79e
その本は見たことないけど、真に異なる2元についての話じゃないの?
0823132人目の素数さん
垢版 |
2019/08/11(日) 19:54:26.20ID:24szNkNg
0<1<2<...<n-1と順序付け蹴られた{1,...n-1}がn要素の全順序と同型とか
かいてあるんだけど
0<2<3=4<...<n-1と順序づけられた{1,...,n-1}はn要素の全順序だけど同型じゃないよね
この本捨てたほうが良い?
0824132人目の素数さん
垢版 |
2019/08/11(日) 21:51:37.00ID:24Hn+lSJ
辺の長さがa,b,c(0<a<b<c、a^2+b^2+c^2=1)の直方体を、対角線を軸として回転させたときに通過する部分の体積を求めよ。

が分かりません。
0827132人目の素数さん
垢版 |
2019/08/11(日) 23:04:48.03ID:5kZNsF+W
失礼致します。
V : 0次元K-vector空間
T : Vの線形変換 とするとき、
Vの基底∅に関するTの表現行列は
0×0の零行列になるべきだと思うのですが、
これは表現行列のどのように定義すれば正当化できますでしょうか。
0828132人目の素数さん
垢版 |
2019/08/11(日) 23:18:04.17ID:FbwepJo+
Kが1の原始n乗根ζを含むとき、巡回拡大L/Kは冪根拡大である。と、ガロア理論の本に、書いてあるんですが、これってヒルベルトの第12問題の特殊な場合ですか?
0830132人目の素数さん
垢版 |
2019/08/11(日) 23:24:49.82ID:FbwepJo+
すみません。分からない問題スレに書くことではなかったですね。質問スレ行ってきます。
0831132人目の素数さん
垢版 |
2019/08/11(日) 23:28:27.21ID:nJJxU8Jr
>>827
0次元ベクトル空間の基底はφとか言うわけわからないものじゃないはずですよ
もっと身近なものなはずですね
0832132人目の素数さん
垢版 |
2019/08/11(日) 23:39:16.63ID:RSK9lVUi
n,p,q,rは整数でp+q+r=nを満たす
このとき
n!/(p!q!r!)が必ず整数になる理由を教えてください。
0833132人目の素数さん
垢版 |
2019/08/11(日) 23:39:51.08ID:5kZNsF+W
>>831
いえ、0次元vector空間の基底は空集合∅です。
というか、ただ1点からなる基底なので
0836132人目の素数さん
垢版 |
2019/08/11(日) 23:46:41.85ID:5kZNsF+W
>>831
すみません。途中で送信してしまいました。
ただ1点からなるK-vector空間V={0}の基底は空集合∅しかありませんので、
dim(V)=|∅|=0 というのが正しいです。

なお、質問の内容については、
自己解決しました。

ご回答頂きありがとうございました。
0838132人目の素数さん
垢版 |
2019/08/11(日) 23:58:24.46ID:5kZNsF+W
>>835
推敲している途中に送信してしまいましたので、
「ただ1点からなる基底」という意図しない文章を送ってしまいました。
失礼致しました。
0839 【大凶】
垢版 |
2019/08/12(月) 00:39:37.45ID:R6APndVz
>>797
>>824
xyz空間に点(a,b,c)をa<b<cとなるようにとり、点(0,0,0)と結んだ直線を対角線とする直方体をこの直線を軸に一回転させると、直方体の通過部分は、
円錐と円錐台をてれこにして円錐台のちっさい側の底面で張りあわせた鼓のような形。
点(0,0,0)も点(a,b,c)もわりと鈍角にとがった頂点。
オリオン座を天地対称にした形。
直方体をz軸に平行に二等分した断面を、縦c横√(a^2+b^2)の長方形で描くと、対角線の長さは題意より、
√(a^2+b^2+c^2)=1
直角三角形の合同とピタゴラスの定理より、円錐および円錐台の半径と高さを特定する。3辺の比はおっきいほうから、
1:c:√(a^2+b^2)
=1:c:√(1-c^2)
円錐=(1/3)π{c^2/(1-c^2)}(1-c^2)
=πc^2/3
鼓形物体の最大半径
=c/√(1-c^2)
対角線中央の底面積(鼓形物体のくびれてる部分の最小断面)
=(1/2){c/√(1-c^2)}(1/c^2)
=1/2c√(1-c^2)
円錐台=(1/3)π{c^2/(1-c^2)}-(1/3)π{1/4c^2(1-c^2)}(1/2)
∴直方体の通過部分の体積
=2(円錐+円錐台)
=2πc^2/3
+2(1/3)π{c^2/(1-c^2)}-2(1/3)π{1/4c^2(1-c^2)}(1/2)
=2πc^2/(1-c^2)-π/12c^2(1-c^2)
=(8c^2-1)π/12c^2(1-c^2)
0842イナ ◆/7jUdUKiSM
垢版 |
2019/08/12(月) 01:53:22.78ID:R6APndVz
>>839
>>840
a<b<cだから、
c=1/√3と仮定したら、aもbも存在しないだろう。
a^2+b^2=1-c^2=1-1/3=2/3
aもbもcより短い。つまりcは、
1/√3<c
1/√3よりおっきいはずで、その仮定に矛盾する。
0843132人目の素数さん
垢版 |
2019/08/12(月) 02:23:34.08ID:yp85yo3k
コレも円錐とかの組み合わせにはならないやつで積分しないと無理だろ?
よってイナには無理です。
0844132人目の素数さん
垢版 |
2019/08/12(月) 02:40:19.08ID:Nl95aGGH
>>842
<ではなく<=でした申し訳ないです泣
a=b=cのときはπ/9になるはずなんです

>>843
そうですね、円柱2つと回転一葉双曲面とあと2つの物体になるはずです
0845132人目の素数さん
垢版 |
2019/08/12(月) 06:04:32.09ID:PDjbyBy2
>>832
二項係数
nCr = (n!)/((n-r)!)(r!)
が整数であることを使い、この積で表す。

(p+q+r)!/(p!)(q!)(r!)
= (p+q)!/(p!)(q!)・(p+q+r)!/((p+q)!)(r!)
= (p+q)Cq・(p+q+r)Cr
2つの二項係数は整数だから、与式は整数。
0846イナ ◆/7jUdUKiSM
垢版 |
2019/08/12(月) 10:20:23.27ID:R6APndVz
>>842
>>844
a≦b≦cってことか。立方体も直方体の1つというわけだ。
>>843体積1の直方体を対角線を軸に一回転させると、対角線に対してねじれの位置にある長さcの長辺が通過する部分はなめらかな曲面になるね。
側面が双曲面の鼓の両面に円錐の底面を張りあわせた形になる。
/\  /\
) (  〉〈
\/  \/
.○   ×
円盤を足し集めるか。さっきより少しおっきなりそうだ。
0847イナ ◆/7jUdUKiSM
垢版 |
2019/08/12(月) 11:43:16.15ID:R6APndVz
>>846鼓部分を増改修。
xyz空間に点(a,b,c)をa≦b≦cとなるようにとり、点(0,0,0)と結んだ直線を対角線とする直方体をこの直線を軸に一回転させると、直方体の通過部分は、
側面が双曲面の鼓の両面に円錐をてれこにして底面を張りあわせた形。
点(0,0,0)も点(a,b,c)もわりと鈍角にとがった頂点。
オリオン座を天地対称にして側面をなめらかにした形。
直方体を対角線とz軸がなす平面で二等分した断面を、縦c横√(a^2+b^2)の長方形で描くと、対角線の長さは題意より、
√(a^2+b^2+c^2)=1
直角三角形の3辺の比はおっきいほうから、
1:c:√(a^2+b^2)
=1:c:√(1-c^2)
円錐=(1/3)π{c^2/(1-c^2)}(1-c^2)
=πc^2/3
鼓形物体の最大半径
=c/√(1-c^2)
直角三角形の合同とピタゴラスの定理より、
円錐の底面の半径は、
c/√(1-c^2)
円錐の高さは、
1-c^2
双曲面の高さは、
1-2(1-c^2)=2c^2-1
双曲面間の円盤の半径を鼓の底面からの距離t(0≦t≦c^2-1/2)で表すと、

積分する。
∴直方体の通過部分の体積
=2(円錐+側面が双曲面の円台)
=2πc^2/3
+2π∫[t=0→c^2-1/2](双曲面と対角線の距離)^2dt
=
双曲面と対角線の距離すなわち双曲線と直線の距離がtの関数で表せれば積分関数がわかるはず。
0848イナ ◆/7jUdUKiSM
垢版 |
2019/08/12(月) 19:05:46.19ID:R6APndVz
>>847
たばこの箱の8つの頂点のうちいちばん遠い2つの頂点を親指と中指か薬指ぐらいで挟むように支え、対角線を軸に箱をはじき、超高速で回転させつづけると通過部分がわかる。
できればソフトタイプよりボックスタイプがいい。未開封であればなおよい。
対称軸に対する半径の極大値は上3つ、下3つ。指が触れてない6つの頂点のそれぞれにある。
半径の極大値は上から、
a√(b^2+c^2)=a√(1-a^2)――@
b√(c^2+a^2)=b√(1-b^2)――A
c√(a^2+b^2)=c√(1-c^2)――B
c√(a^2+b^2)=c√(1-c^2)――C
b√(c^2+a^2)=b√(1-b^2)――D
a√(b^2+c^2)=a√(1-a^2)――E
たばこの箱の場合、
a≦b≦√(a^2+b^2)≦c≦√(c^2+a^2)≦√(b^2+c^2)
だが、√(a^2+b^2)とcの大小関係は決まってない。
大小が変わればA〜Dは順番が入れ替わる。
たばこの12本ある辺のうち指が触れてない6本の辺はねじれの位置にある。
回転軸となる対角線に対してねじれの位置にある6本の辺が描く軌跡は双曲面。
指が触れてる6本の辺とその内側が描く軌跡は円錘で上下あわせて12個。
0849132人目の素数さん
垢版 |
2019/08/12(月) 22:39:10.09ID:uLwjs1DH
>>845
二項係数 nCr = n!/((n-r)!・r!) が自然数であることは
n についての帰納法で示せる。

パスカルの規則
 nCr = (n-1)Cr + (n-1)C(r-1)   (0<r<n)
 nC0 = (n-1)C0 = 1,
 nCn = (n-1)C(n-1) = 1,
を使う。
0850132人目の素数さん
垢版 |
2019/08/12(月) 22:45:27.51ID:8kN9J226
図書館で読んだ微積分の本で、
∫[0→ki] f(x) dx
のように積分区間に虚数単位iが出てきましたが、これはどう解釈したら良いでしょうか。

問題としては確か、実数の積分
∫[0→∞] g(x)*exp(-x^2) dx
を重積分に依らず技巧的に求めるもので、その過程で複素数を導入していました。g(x)はすみませんが失念しました。
なお、読者が複素関数論を学んでいることは想定してない本だと思います。

高校の積分では、実数値関数とx軸との間の面積として積分を扱っていたと思います。虚数単位が出てくると、面積は考えられなくなると思うのです。

長くなりましたがよろしくお願いいたします。
0851132人目の素数さん
垢版 |
2019/08/12(月) 23:09:36.50ID:kGEWDH2n
>>850
複素積分を勉強してね
0852132人目の素数さん
垢版 |
2019/08/13(火) 00:13:20.51ID:kM02sybU
>>851
読め
複素積分が既習であることを考慮してない本にいきなり登場したって書いただろ
0853132人目の素数さん
垢版 |
2019/08/13(火) 00:36:25.47ID:L5RmY8hi
>>852
積分は面積だなんだ言ってるようなアホが
解釈()なんて考えたところで意味無いから
もうちょっと広い世界の勉強をしてから戻っておいでよ

何も知らないアホが一々解釈なんてしていったところで
何の意味も無いどころか、却って害にしかならんことも多いからな
0854132人目の素数さん
垢版 |
2019/08/13(火) 00:48:00.36ID:kM02sybU
>>853
高校生の積分なんて、区分求積を微積分の基本定理で簡単に計算できます、程度でしかない
たかが高校卒業して数ヶ月の工学部生が何の説明もなく積分区間にi入ってる数式見たら、戸惑うのは当然と思わない?
ついでに、その参考書見てもどこにも積分区間にi入れていい説明載ってなかったぞ
0855132人目の素数さん
垢版 |
2019/08/13(火) 00:54:41.64ID:ekC7hCRN
>>850
ふむ。学んでいないことを前提としているのであれば、高校生相手に裏技で検算の一手段を教えるつもりなのだろう。
つまり、考えている十分広い領域で正則であることが明らかなような関数だけを考えて説明しているのだろう。
f(x) の原始関数を F(x) とするとき
F(ki)-F(0) で正しい値が与えられる筈。(積分経路は問題にならない筈なので)
0858132人目の素数さん
垢版 |
2019/08/13(火) 01:26:24.43ID:wcTxNf5z
https://imgur.com/cooKcRK
(1) 次の連立方程式の解を持つ条件を求めよ。その時の解を求めよ。
3x+4y+5z = a
2x+3y+4z = b
x+2y+3z = c

(2)次の関数f(x)をx=1/2を中心にテイラー展開せよ。その時の収束半径を求めよ。
f(x) = 1/(x^2-x+1)


この2題が解けないんですが、どうしたらいいでしょうか・・・?
自分的にやってみたんですが、(1)はrankA = rankAb にならないと解が一つに求まらないから、zを適当に決めてあげる?という感じで纏めました。
(2)に関しては、とりあえず二階微分してみたんですが、何かが違う感が出たので諦めました。。。。
0859132人目の素数さん
垢版 |
2019/08/13(火) 01:49:09.58ID:L5RmY8hi
>>854
工学部生なら道具として使えればいいだけだから
猶更、戸惑う必要無い
こうすれば求まるってだけで十分だろう
工学部逝きの落ちこぼれが無理に考えても仕方ねえんだからよ

勉強していてどうしても知りたいことがあるなら
いろんな本に当たったり、調べるのは普通の事
それができないから落ちこぼれたのでは
0860132人目の素数さん
垢版 |
2019/08/13(火) 01:52:43.43ID:Ef6DrBrl
>>656
(1)もともと解が一意に定まるはずない。
解一個v0もとめて、Av=0の非自明解v1求めてv=v0+kv1
(2)部分分数分解
0862132人目の素数さん
垢版 |
2019/08/13(火) 04:42:41.89ID:TamC0u4p
>>861
あ、分かりました。繰り下げのとき+10-1して考えるけどmodで考えるから関係ないのですね。
0863132人目の素数さん
垢版 |
2019/08/13(火) 09:33:10.61ID:kQEu2nzR
>>858
rank((3, 4, 5), (2, 3, 4), (1, 2, 3)) = 2
なので、この方程式は事実上
px + qy + rz = s
tx + uy + vz = w
(1次式2本の方程式)と同等。
つまり、3本目の式が他の2本の式と
矛盾しなければよい。
x + y + z = a - b = b - c であるから、
a + c = 2b(a, b, c がこの順に等差数列)
であることが、解を持つための必要十分条件。
0864132人目の素数さん
垢版 |
2019/08/13(火) 09:51:09.60ID:SJPyKl4N
>>858
(つづき)
x + y + z = a - b = b - c = d とおくと
a = c + 2d、b = c + d
また -x + z = c - 2d、y + 2z = c - d なので、
x = z - c + 2d、y = -2z + c - d
綺麗にするために z = t + c - d とおけば、
(x, y, z)
= (t + d, -2t - c + d, t + c - d)
= (
0865132人目の素数さん
垢版 |
2019/08/13(火) 10:06:09.79ID:SJPyKl4N
>>858
x^2 - x + 1 = (x - 1/2)^2 + 3/4 であるから、
t = (x - 1/2)^2、a = 3/4 とおいて、
t = 0 での Taylor 展開をすればよい。

f(x) = 1/(t + a)
= (1/a)(1/(1 + u)) (u = t/a とおいた)
= (1/a)(1 - u + u^2 - u^3 + ……)
= (4/3) 納k=0, ∞] (-1)^k (x - 1/2)^(2k)
0866132人目の素数さん
垢版 |
2019/08/13(火) 11:03:32.71ID:gccQR1zi
>>858
 (2)
f(x) = 1/{(3/4) + (1/2 - x)^2}
  = (4/3)/{1 + (4/3)(1/2 - x)^2}
  = (4/3)Σ[k=0,∞] (-4/3)^k・(1/2 - x)^2k  ・・・・ 等比級数
収束条件は
 |公比| = (4/3)(1/2 - x)^2 < 1
 |1/2 - x| < (1/2)√3 = R,
0867132人目の素数さん
垢版 |
2019/08/13(火) 11:20:20.36ID:Il4ex80V
ベクトル(1,1,1)をnとして、nに垂直な平面x+y+z=tを新たに(今の)xy平面のように考えた時の空間の基底ベクトルのうち(1,1,1)以外のものを求めたいのですが、どうしたらいいでしょうか?
0869132人目の素数さん
垢版 |
2019/08/13(火) 12:10:26.35ID:gccQR1zi
>>861

6. 7ア4281イ071 が 99 で割りきれるように、1けたの数 (ア)、(イ) を求めよ。

 7042810071 + (10^8)(ア) + 1000(イ) ≡ 66 + (ア) + 10(イ) (mod 99)

 (ア) = 3, (イ) = 3, 7342813071 / 99 = 74169829.

7. 1998の倍数のうち、各位の数がすべて等しい最小の数を求めよ。

 1998 = 2(10^3 -1),
 333667 = (10^6 +10^3 +1)/3,
 辺々掛けて 6 (10^9 -1)/9.
0873132人目の素数さん
垢版 |
2019/08/13(火) 15:15:24.75ID:TLRsURuN
>>869
上の問題まで回答ありがとうございます!

333667 = (10^6 +10^3 +1)/3
これはどのようにして発見するのですか?
0874132人目の素数さん
垢版 |
2019/08/13(火) 15:46:51.92ID:TLRsURuN
>>869
何度もすみません。
333667 = (10^6 +10^3 +1)/3これはだんだん分かってきたのですが。
更に疑問が、最小性は(10^6 +10^3 +1)/9、(10^3 +1)/3、 (10^3 +1)/9、(10^3 +1)/2がそれぞれ整数でないことをいえば十分なのですか?
0875イナ ◆/7jUdUKiSM
垢版 |
2019/08/13(火) 16:52:40.46ID:LC7aWG7e
>>848
>>824たばこの箱を対称軸が最長になるように持ち、一回転させると、上下対称な円錘2個と円錘台10個を積み重ねた立体になる。
回転軸である対角線とねじれの位置にある箱の最長辺が作りだす双曲面と対角線の最小距離≦1/2なら、双曲面は円錐台の内部。
通過部分は円錐と円錐台のみ。
かなりギザギザな三連屋根となる。
対称軸0〜1のうち、
0〜a^2――@は円錘、
a^2〜a√(1-b^2)/{ab+√(1-a^2)(1-b^2)}――A,
a√(1-b^2)/{ab+√(1-a^2)(1-b^2)}〜b^2――B,
b^2〜b√(1-c^2)/bc+√(1-b^2)(1-c^2)――C,
b√(1-c^2)/bc+√(1-b^2)(1-c^2)〜c^2――D,
c^2〜1/2――Eの5区分は円錘台5つ、
1/2〜1-a^2の5区分も円錘台5つ、
1-a^2〜1は円錐。
円錐2個と円錐台10個の和は、円錐1個と円錐台5個の和の2倍。
@(1/3)πa^2(1-a)^2a^2
=πa^4(1-a^2)/3
Aπ/3a^2-πa^(1-b^2)[1-a(1-b^2)/{ab^2+b√(1-a^2)(1-b^2)]/3{ab+√(1-a^2)(1-b^2)}^2
Bπb^4(1-b^2)/3-πa^3(1-b^2)^2/{ab^2+b√(1-a^2)(1-b^2)}/3{ab+√(1-a^2)(1-b^2)}^2
Cπ/3b^2-πb^(1-c^2)[1-b(1-c^2)/{bc^2+c√(1-b^2)(1-c^2)]/3{bc+√(1-b^2)(1-c^2)}^2
Dπc^4(1-c^2)/3-πb^3(1-c^2)^2/{bc^2+c√(1-b^2)(1-c^2)}/3{bc+√(1-b^2)(1-c^2)}^2
Eπ{8c^2(1-c^2)^2-1}/24
通過部分の体積=2(@+A+B+C+D+E)
=2πa^4(1-a^2)/3
+2π/3a^2-πa^(1-b^2)[1-a(1-b^2)/{ab^2+b√(1-a^2)(1-b^2)]/3{ab+√(1-a^2)(1-b^2)}^2
+2πb^4(1-b^2)/3-πa^3(1-b^2)^2/{ab^2+b√(1-a^2)(1-b^2)}/3{ab+√(1-a^2)(1-b^2)}^2
+2π/3b^2-πb^(1-c^2)[1-b(1-c^2)/{bc^2+c√(1-b^2)(1-c^2)]/3{bc+√(1-b^2)(1-c^2)}^2
+2πc^4(1-c^2)/3-πb^3(1-c^2)^2/{bc^2+c√(1-b^2)(1-c^2)}/3{bc+√(1-b^2)(1-c^2)}^2
π{8c^2(1-c^2)^2-1}/12
0876イナ ◆/7jUdUKiSM
垢版 |
2019/08/13(火) 16:59:48.45ID:LC7aWG7e
>>875最終行、πの前に+が抜けた。
>>824

あってると思う。
a^2+b^2+c^2=1を使って文字を3文字から2文字に減らすことは可能だが、与えられたa,b,cが答えに影響することを思えば、3つとも使っていいと思います。
もう少し簡単になる可能性はある。
0879132人目の素数さん
垢版 |
2019/08/14(水) 12:29:40.39ID:jiacTcOi
x,yについての連立方程式
sx-(1-t)y=1
(1-t)x+sy=a
について、以下の問に答えよ。

(1)s,tが-1≤s≤1かつ-1≤t≤1である実数の定数で、a=0のとき、解x,yについて|x+2y|の最小値を求めよ。

(2)s,tが-1≤s≤1かつ-1≤t≤1を満たしながら変化し、aが実数の定数であるとき、-1≤x≤1かつ-1≤y≤1の解を持つようなs,tの条件式を求めよ。
0886132人目の素数さん
垢版 |
2019/08/14(水) 18:29:12.97ID:HQlvV6Mq
>>879
s=0, t=1 のとき … 解なし。
それ以外のとき
 x = {a(1-t)+s}/{ss+(1-t)^2},
 y = {as-(1-t)}/{ss+(1-t)^2},

(1) a=0 のとき
 |x+2y| = |s-2(1-t)|/{ss+(1-s)^2},
 s=2(1-t), 0<s≦1, 1/2≦t<1 のとき最小値 0
0887132人目の素数さん
垢版 |
2019/08/15(木) 01:22:05.78ID:RxBWT0Y0
>>879

A(1/2, a/2)  B(-a/2, 1/2)  C(-1/2, -a/2)  D(a/2, -1/2)
は 原点O(0,0) を中心とする正方形で
 OA = OB = OC = OD = (1/2)√(1+aa), これをrとおく。

(2)
-1≦x≦1, -1≦y≦1 となる条件は
(s,1-t) と A,B,C,D の距離がいずれも r 以上であること。
(s,1-t) は A,B,C,Dを中心とする半径rの円の外側。
0888132人目の素数さん
垢版 |
2019/08/15(木) 01:52:43.08ID:RxBWT0Y0
線形代数っぽく云えば、

一次変換
 X = sx - (1-t)y,
 Y = (1-t)x + sy,
は θ=arctan((1-t)/s) の回転と √{ss+(1-t)^2} 倍を行うもの。

逆変換          >>886
 x = {sX + (1-t)Y}/{ss+(1-t)^2},
 y = {-(1-t)X + sY}/{ss+(1-t)^2},
は -θ の回転と 1/√{ss+(1-t)^2} 倍を行うもの。
0890132人目の素数さん
垢版 |
2019/08/15(木) 05:11:15.45ID:NPux4hZm
「自刃しろ。」と聞こえてきたが、たかが論文をリジェクトされたからと言って
そうするわけないだろう

馬鹿じゃねーの?
0891132人目の素数さん
垢版 |
2019/08/15(木) 05:27:23.62ID:NPux4hZm
正しいか正しくないかは優秀な数学者は判断可能だろうから、数か月の間には
問題になるのであろうか?
0892132人目の素数さん
垢版 |
2019/08/15(木) 11:20:51.71ID:Fy0P+UNL
質問させてください。a、b、c、dの4チームがクジを引いてトーナメントで試合をする場合、aとBが1回戦で対戦する確率は3分の1?2分の1?どっちですか?
0893132人目の素数さん
垢版 |
2019/08/15(木) 11:27:56.29ID:clgnH51z
どちらでもなく0だと思いますよ
0894132人目の素数さん
垢版 |
2019/08/15(木) 11:38:16.49ID:Fy0P+UNL
>>893ヤクザ
aとBという揚げ足取るなよドアホ
0895132人目の素数さん
垢版 |
2019/08/15(木) 11:57:27.62ID:Fy0P+UNL
質問させてください。a、b、c、dの4チームがクジを引いてトーナメントで試合をする場合、aとbが1回戦で対戦する確率は3分の1?2分の1?どっちですか?
0897132人目の素数さん
垢版 |
2019/08/15(木) 12:43:57.44ID:Fy0P+UNL
>>896
そんな遠回しに書かんでええから、どっちか書けやタコ
0898132人目の素数さん
垢版 |
2019/08/15(木) 12:59:18.32ID:LKxW09nX
>>895
意味不明な逆切れをした挙句、何事もなかったかのように問題チェンジ
お前エラだろ
0901132人目の素数さん
垢版 |
2019/08/15(木) 13:20:33.58ID:Na35gbME
一試合で十本のホームランが出たチームに
入る最小限の得点は?
0902イナ ◆/7jUdUKiSM
垢版 |
2019/08/15(木) 13:40:51.09ID:Lu+2Gj09
>>876
>>901
ホームランを打ってホームベースを踏まなかった選手がアウトになることがあった。
長嶋選手は一塁ベースを踏み忘れてピッチャーゴロになった。
∴最小限の点数は0点
0903132人目の素数さん
垢版 |
2019/08/15(木) 16:20:28.94ID:Yo4OPtk5
A,Bは2次正方行列であり、
AB=BA=P^(-1)AP…(*)
を満たすn次正方行列Pが存在するという。

(1)A,Bに逆行列が存在するか、それぞれ判定せよ。
(2)Bが与えられており、Aが変化するとき、B=APとなるためのAの条件をBで表わせ。
0905132人目の素数さん
垢版 |
2019/08/15(木) 17:06:30.08ID:Yo4OPtk5
すみません写し間違えました

A,Bは2次正方行列であり、
AB=BA=P^(-1)AP…(*)
を満たす2次正方行列Pが存在するという。

(1)A,Bに逆行列が存在するか、それぞれ判定せよ。
(2)Bが与えられており、Aが変化するとき、B=APとなるためのAの条件をBで表わせ。
0906132人目の素数さん
垢版 |
2019/08/15(木) 17:35:09.96ID:BJH0L0Wg
ポエマーは 写し間違えのじゅもんを おぼえた!
0907132人目の素数さん
垢版 |
2019/08/15(木) 17:47:05.82ID:NPux4hZm
ゴミは妄想の電波を放出するのをやめてくれ、私に上司がいたのは10年以上前だ
0908132人目の素数さん
垢版 |
2019/08/15(木) 17:48:11.16ID:NPux4hZm
右折でつっこんでくるのやめてね、正面衝突で自殺したいのか分からないが
0913132人目の素数さん
垢版 |
2019/08/15(木) 19:47:18.06ID:Na35gbME
正解
0915132人目の素数さん
垢版 |
2019/08/16(金) 01:04:05.31ID:jhC+gulh
野球のルールには詳しくないのだが、
ベースを踏み忘れた時もホームランと言うのかい?
0919132人目の素数さん
垢版 |
2019/08/16(金) 02:54:25.36ID:jhC+gulh
いやいや、>>901の問題記述と>>902を正解とした.>>913への疑義ゆえ、まだまだこの数学のスレのうち。

>>902,917から敷衍するに、打球がフェア領域を飛翔のまま観客席に落下した場合、
打者が全塁を踏んでホームに到達した場合のみホームランとして得点換算。
それ以外の場合は、記録上は踏んだ塁までの塁打で、得点は無し(つまりホームランではない)
故に、>>901への答は、問題記述でホームランとしている以上全塁を踏んでいることになるので最小得点は10、というkとになるな。
0920132人目の素数さん
垢版 |
2019/08/16(金) 05:22:25.98ID:ZMLcuGhl
AB+BA=E
A^2+B^2=O
を満たす2×2行列A,Bの例をあげよ。
ただしEは単位行列、Oは零行列である。
0922132人目の素数さん
垢版 |
2019/08/16(金) 11:48:17.20ID:sdM1emAh
#919
不正解
0923132人目の素数さん
垢版 |
2019/08/16(金) 15:55:31.82ID:Tq2l7gt9
有限次元のベクトル空間が与えられたとき、その全ての基底を見つけることはできますか?
0925132人目の素数さん
垢版 |
2019/08/16(金) 16:31:57.68ID:oNtuWoss
>>920
(A+B)^2 = E,
(A-B)^2 = -E,
より
 A+B = [ p, 0 ]
     [ 0, r ]
ここに p=±1, r=±1,

 A-B = [ 0, q ]
     [-q, 0 ]
ここに q=±1,

よって
 A = (1/2)[ p, q ]
       [-q, r ]

 B = (1/2)[ p, -q ]
       [ q, r ]
0926132人目の素数さん
垢版 |
2019/08/16(金) 16:35:06.54ID:JLFT4W5r
N次元の運動の影をM次元(M<N)に投影した場合、
その影から元の次元Nを推測or計算するする方法はありますか
0929132人目の素数さん
垢版 |
2019/08/16(金) 17:16:21.22ID:AMv1eF+R
>>928
多分認めてもらえないでしょう。
ある二変数のみを動かして残りの変数を固定して考えたときにある点で最小となることは、全ての変数を自由に全部動かしたとき、その点で最小となる事の必要条件ではありますが、十分性は確認しないと自明とは認めてもらえないと思います。
0931132人目の素数さん
垢版 |
2019/08/16(金) 17:54:20.11ID:tnsB+xEg
>>929
この方針でやりたいなら
xの全ての値が等しくない場合、等しくない2つを取って平均を取り等しくすると、Fはより小さくなる
これを繰り返すとxが全て等しくなる

これが言えないとダメってことですね。ありがとうございます

そもそもこの操作繰り返すと等しくなるかったらならないですもんねこれ…
3つしかxない時考えると、全てが等しくなる最後の操作の直前、平均からのズレが0,α,-αになってないといけないから
初期配置がこれじゃないと無理ってことですもんね
ありがとうございました
0934132人目の素数さん
垢版 |
2019/08/16(金) 18:31:07.56ID:ggiAunhW
二次方程式の解をa,bとしたとき
(a-b),(b-a)を解とする二次方程式を求める問題で
塾の先生が簡単だけどこれはもっと深い話につながる
と言っていたのですが何か意味あるのでしょうか

やめたのか担当が新しい先生なってしまって聞いても
深いというのは人によるから気にするなとしか言ってくれませんでした
0935132人目の素数さん
垢版 |
2019/08/16(金) 18:32:06.09ID:oNtuWoss
>>920
(A+B)^2 = E,
より
 A+B = [ 0, s ]
     [ 1/s, 0 ]
ここに s≠0
よって
 A = (1/2)[ 0, s+q ]
       [1/s -q, 0 ]

 B = (1/2)[ 0, s-q ]
       [ 1/s +q, 0 ]
0936352
垢版 |
2019/08/16(金) 18:38:54.05ID:ivsWmPiu
>>934
4次元で考えたら何か見えるかもしれん
0937132人目の素数さん
垢版 |
2019/08/16(金) 18:43:04.19ID:oNtuWoss
>>920
 (A+B)^2 = E,
 (A-B)^2 = -E,
より
 A+B = [ 0, s ]
     [ 1/s, 0 ]

 A-B = [ 0, t ]
     [ -1/t, 0]
ここに st≠0,
よって
 A = (1/2)[ 0, s+t ]
       [1/s -1/t, 0 ]

 B = (1/2)[ 0, s-t ]
       [ 1/s +1/t, 0 ]
0938132人目の素数さん
垢版 |
2019/08/16(金) 18:47:38.83ID:ggiAunhW
>>936
ありがとうございます
四次元とは四次方程式ということでしょうか
もしそうなら四次方程式につながる深い話ということですか?
0940132人目の素数さん
垢版 |
2019/08/16(金) 19:49:15.84ID:tnsB+xEg
>>928
と思ったけど、「全てのxが等しくない限り、絶対にFは最大でない」ということはこれから言えるはずなので、こう書けばokでしょうか?

間違えてxa+xb=k-Sと書いてしまってますが、xa+xb=Sに読み替えて下さい。
0942132人目の素数さん
垢版 |
2019/08/16(金) 19:52:13.55ID:zX6dA1oW
>>931
数学的帰納法で
Σ[i=1,j-1](x_i*ln(x_i))≧Σ[i=1,j-1](x_i)*ln(Σ[i=1,j-1](x_i)/(j-1))
⇒Σ[i=1,j](x_i*ln(x_i))≧Σ[i=1,j](x_i)*ln(Σ[i=1,j](x_i)/j)
が言えないかな?

それで
Σ[i=1,n](x_i*ln(x_i))≧Σ[i=1,n](x_i)*ln(Σ[i=1,n](x_i)/n)=k*ln(k/n)
が言えれば、Σ[i=1,n](x_i)=kとなる任意の{x_i≧0}の組で言えるけれど
0943132人目の素数さん
垢版 |
2019/08/16(金) 20:05:49.42ID:zX6dA1oW
>>940

> >>928
> と思ったけど、「全てのxが等しくない限り、絶対にFは最大でない」ということはこれから言えるはずなので、こう書けばokでしょうか?

Fに極小値があることと、全てのxが等しくない時Fより小さいF'が存在することは
言えるけれど、
F'が極小値より小さい可能性を否定できない
0944132人目の素数さん
垢版 |
2019/08/17(土) 02:16:46.36ID:BxLWuOXn
Lie群の普遍被覆群がまたLie群となることはどのように示したら良いですか?
0945132人目の素数さん
垢版 |
2019/08/17(土) 05:53:09.37ID:CeaLmZxZ
>>928
n≧2,
 x_i を正の数とし、Σ[i=1,n] x_i = k をみたすとする。
Π[i=1,n] (x_i)^(x_i) の最小値を求めよ。

(略解)
 log(x) - log(k/n) = - log(k/(nx)) ≧ - {k/(nx) -1},
 x log(x) - x log(k/n) ≧ x - k/n,
i=1〜n でたす。
 Σ[i=1,n] (x_i)log(x_i) - k log(k/n) ≧ k - k = 0,
 Π[i=1,n] (x^i)^(x^i) ≧ (k/n)^k,
∴すべての {x_i} を自由に動かしても、これが最小値。

あるいは
 f(x) = x log(x) とおくと f "(x) = 1/x >0 (下に凸)
 Jensen で・・・・
0946132人目の素数さん
垢版 |
2019/08/17(土) 08:34:17.04ID:4NKDQA0A
>>943
アホですいません……難しくてよくわかりません

@違う2つのxを含む時は絶対に最小でない
A違う2つのxを含まないxの組は全てが等しい1組しかない
Bなので最小の候補はこれしかない、よってこれが最小

これって論理としてやっぱり弱いですか?書いてても自信無いですが
0947132人目の素数さん
垢版 |
2019/08/17(土) 08:35:24.91ID:4NKDQA0A
>>942
模範解答ではxlogxが上に凸なことを利用して帰納法で示していました。jensen不等式と言うやつだと思います
0948132人目の素数さん
垢版 |
2019/08/17(土) 08:43:04.33ID:4NKDQA0A
>>945
ありがとうございます。
正の数に対してlogx≧1-(1/x)なのですね。これ知りませんでした…

いやーこれはきれいな解き方ですね。ありがとうございます。
0949132人目の素数さん
垢版 |
2019/08/17(土) 11:25:06.40ID:qlT1SKHA
直接、分からないけど問題ではないんですが…
現代数学の最高点をレベル100とすると、
高校数学のレベルってどの辺りなんですか?
レベル15くらい?
0956132人目の素数さん
垢版 |
2019/08/17(土) 16:00:14.88ID:z3U6yrga
整数nの奇数桁目の数字の合計をA[n]、偶数桁目の数字の合計をB[n]とおく。
例えば
n=5のときA[n]=5、B[n]=0
n=39のときA[n]=9、B[n]=3
n=19855720のとき、A[n]=0+7+5+9=21、B[n]=2+5+8+1=16
である。
このとき、以下を証明せよ。

lim[n to infty] {Σ[k=1 to n] A[n]}/{Σ[k=1 to n] B[n]} = 1
0957132人目の素数さん
垢版 |
2019/08/17(土) 17:25:43.89ID:Xzt+q1zF
数学の最高が100なら高校数学は2とか3だろうな
ちなみに100はあくまで最高点であって研究レベルが2とか3の数学者もゴロゴロいる
0959132人目の素数さん
垢版 |
2019/08/17(土) 20:37:12.20ID:6UIOHwOk
>>946
> @違う2つのxを含む時は絶対に最小でない
> A違う2つのxを含まないxの組は全てが等しい1組しかない
> Bなので最小の候補はこれしかない、よってこれが最小

> Bなので最小の候補はこれしかない
これは、最小が存在するとしたらこれである、ということを言っているだけで、最小値が存在しない場合を否定できない

例えば-∞までいくらでも小さい値をとれる場合や、
Aの極小値より小さい数αを下限として、いくらでも近い数に値をとれるが、αの値をとる組が存在せず、αは極小値にならない場合。
これらは@、Aを満たしうるけれどAの極小値は最小値にはならない
0960132人目の素数さん
垢版 |
2019/08/17(土) 20:53:29.33ID:6UIOHwOk
> 例えば-∞までいくらでも小さい値をとれる場合や、
> Aの極小値より小さい数αを下限として、いくらでも近い数に値をとれるが、αの値をとる組が存在せず、αは極小値にならない場合。
> これらは@、Aを満たしうるけれどAの極小値は最小値にはならない
ついでに、
> Bなので最小の候補はこれしかない、
も満たしうるけれど、
> よってこれが最小
は満たさない
0961132人目の素数さん
垢版 |
2019/08/18(日) 01:45:05.44ID:/5MJgSP2
>>948

x<1 のとき 1/t ≦ 1/tt,
 ∫[x,1] (1/t) dt ≦ ∫[x,1] (1/tt) dt,

x>1 のとき 1/t ≧ 1/tt,
 ∫[1,x] (1/t) dt ≧ ∫[1,x] (1/tt) dt,
0962132人目の素数さん
垢版 |
2019/08/18(日) 03:11:37.40ID:0ZO8yZ7C
>>920
(A+B)^2 = E より
A+B = E, -E, [a, b] (ただし bc=1-a^2)
        [c, -a]
(A-B)^2 = -E より
A-B = iE, -iE, [p, q] (ただし qr=-1-p^2, i^2=-1)
        [r, -p]
これらより適当に A+B, A-B を選んで
A=(1/2)((A+B)+(A-B)), B=(1/2)((A+B)-(A-B)) とすることにより解を得る
0963132人目の素数さん
垢版 |
2019/08/18(日) 04:04:58.39ID:IMOErfx4
まあその関数に最小値がある事は殆ど自明にわかるからそれを断って議論すれば証明としては成り立つな
何故最小値の存在を確認しなければいけないかさえ理解してれば
0964132人目の素数さん
垢版 |
2019/08/18(日) 07:27:10.11ID:lCg3PZJU
>>959
ありがとうございます!
そもそも最小値が存在することを証明しないとダメってことですね…
これを簡単に示す方法あったりしませんかね
0967132人目の素数さん
垢版 |
2019/08/18(日) 15:30:35.73ID:GrIIhMQ8
そうすると GL(n,R)の全ての元を列挙することと同値だから、

そこから、君の意味で見つけることが可能かどうかを判断してくれ。
0968132人目の素数さん
垢版 |
2019/08/18(日) 15:32:52.85ID:MOsS1arh
1次元ベクトル空間の場合を考えれば良いですね

実数全てを列挙ってどうするんでしょうね
0972132人目の素数さん
垢版 |
2019/08/18(日) 19:40:56.40ID:Wo+eu6OS
50
右下に伸ばす補助線で
同一弧からのなんかの角が共通とか
合同とか小学生の知識で習うやつで解ける
0974132人目の素数さん
垢版 |
2019/08/18(日) 19:45:05.52ID:bm3QqEJw
解き方は右側の折り返してくっそゴリ押した
多分もっといい解き方があるんやろうけど

そっか円周角か……
0977132人目の素数さん
垢版 |
2019/08/18(日) 20:02:57.02ID:PHZC80XN
円周角か!!!!なるほど気がつかんかった
作図までありがとう
おまいらサンクス
0978132人目の素数さん
垢版 |
2019/08/18(日) 20:09:40.68ID:5I0VE581
>>973の方針で○=90°とすると、
左右の三角形の上の頂点が重なるから、
左の三角形の上の頂点は40°、x=90°-40°=50°
0980132人目の素数さん
垢版 |
2019/08/18(日) 20:24:58.75ID:Wo+eu6OS
対称性で円周角の定理が使えるパターンになる
中学受験だとこの手の同じ角から補助線引くパターンやりこむことになる
0982132人目の素数さん
垢版 |
2019/08/18(日) 20:33:29.04ID:Wo+eu6OS
50~40度ぐらいな見た目だが
人間の目は縦横の精度に比べて斜めは見えにくい見切りにくい
弾幕シューティングしてると斜めの難易度の高さが実感できる
人間の目は分度器のような精度していない
0983132人目の素数さん
垢版 |
2019/08/18(日) 20:37:50.61ID:IMOErfx4
二等辺三角形を作って、とか外角が、とかで細々計算をする分の情報が円周角の定理に入っている
円周角の定理がそういう証明をするわけだから
0985132人目の素数さん
垢版 |
2019/08/18(日) 23:11:16.28ID:UHfTMwnf
>>979
情報が増えているわけではなく、与えられた情報が使えるようになるってことじゃないかな
元の図だと角度が同じと示されている情報をどう使えばいいのかわからんけど
折り返すと直線が出来上がることがわかる
0986イナ ◆/7jUdUKiSM
垢版 |
2019/08/18(日) 23:26:34.73ID:GxyUo5yh
>>981
ぱっと見x=40°
まずは図にアルファベットの文字をつける。いちばん高い位置にある頂点をAとして左回りにBCDと直径経由で低いほうの頂点Dから直径上のEに戻るのが自然かと。
DEとACの交点をFとする。
∠BCA=40°=∠ECF
∠OAB=50°
∠BOA=80°
∠AOC=100°
必要ないかもしれないけどEを通ってBCに垂直な直線を引く。
∠EAO=10°とすると、
∠OEA=70°となり、
∠DEC=70°
∠CAO=40°だから、
∠CAE=∠CAO-∠EAO=30°
x=40°なら、
∠CDE+∠DCA=∠CFE
x+30°=70°となってちょうどいい。
AE//DCとなるとわかって図が歪んでたとわかる。
もろもろ直しいれて、
x=40°で矛盾なし。
0987イナ ◆/7jUdUKiSM
垢版 |
2019/08/18(日) 23:49:38.66ID:GxyUo5yh
>>986訂正。
∠BCD=65°
∠EAO=15°
∠CAE=25°
x=50°のほうが自然だった。
0989132人目の素数さん
垢版 |
2019/08/19(月) 19:17:40.18ID:j2qBBzTu
xyz+x+y+z-xy-yz-zx-1=(x-1)(y-1)(z-1)
なのは実際に展開すれば分かりますが、どういう発想で左辺から右辺を導けばいいでしょうか
0992132人目の素数さん
垢版 |
2019/08/19(月) 19:35:42.05ID:Q3ufC9fD
対称性からぱっと見でわかると思うが

例えば左辺をxについてまとめ

x(yz-y-z+1)-(yz-y-z+1)
=(x-1)(yz-y-z+1)

yz-y-z-1をyについてまとめ

y(z-1)-(z-1)=(y-1)(z-1)

実際には対称式だから(x-1)で割り切れるのがわかった時点で(y-1)(z-1)でも割り切れるのが分かるけど
0993132人目の素数さん
垢版 |
2019/08/20(火) 04:44:12.47ID:tL4LcjDy
>>956 >>991

Σ[k=01,99] A[k] = Σ[k=01,99] B[k] = 450,

f(n) = Σ[k=1,n] (A[k] - B[k])
とおく。
00≦n≦99 に対して
 0 ≦ f(n) ≦ 225 - 100 = 125,
n を 100進数で表わす。
 n = Σ[i=0,m] n_i・100^i
  00 ≦ n_i ≦ 99,
  01 ≦ n_m ≦ 99,

0 ≦ f(n) = Σ[i=0,m] f(n_i)・100^i
 ≦ 125 Σ[i=0,m] 100^i
 ≒ (125/99)100^(m+1)
 = 126.26 100^m,

Σ[k=1,n] A[k] ≧ (4.5m)(n_m 100^m) ≧ (4.5m) 100^m
Σ[k=1,n] B[k] ≧ (4.5m)(n_m 100^m) ≧ (4.5m) 100^m

n→∞ のとき m→∞
0998132人目の素数さん
垢版 |
2019/08/22(木) 15:10:36.91ID:M2m672lJ
ペアノ曲線はR→R^2の単射ではない
空間充填曲線は自己交差し、連続全単射は存在しない
10011001
垢版 |
Over 1000Thread
このスレッドは1000を超えました。
新しいスレッドを立ててください。
life time: 46日 20時間 35分 51秒
10021002
垢版 |
Over 1000Thread
5ちゃんねるの運営はプレミアム会員の皆さまに支えられています。
運営にご協力お願いいたします。


───────────────────
《プレミアム会員の主な特典》
★ 5ちゃんねる専用ブラウザからの広告除去
★ 5ちゃんねるの過去ログを取得
★ 書き込み規制の緩和
───────────────────

会員登録には個人情報は一切必要ありません。
月300円から匿名でご購入いただけます。

▼ プレミアム会員登録はこちら ▼
https://premium.5ch.net/

▼ 浪人ログインはこちら ▼
https://login.5ch.net/login.php
レス数が1000を超えています。これ以上書き込みはできません。

ニューススポーツなんでも実況